NBME27

Pataasin ang iyong marka sa homework at exams ngayon gamit ang Quizwiz!

88 Exam Section 2: Item 39 of 50 National Board of Medical Examiners' Comprehensive Basic Science Self-Assessment 3' 39. An investigator creates Drug Z that interferes with protein synthesis by binding to the anticodon on the human tRNA shown in the diagram. Drug Z is most likely binding in the place of which of the following codons? A A) AUG 5' A B) CCG C) GCC A D) GCU A E) GUA G A O F) UCG G G. A A,

A. AUG is the codon to which this TRNA anticodon would normally bind in the absence of Drug Z. RNA is a specialized RNA that recognizes a codon on mRNA molecules, which consists of three nucleotides in a specific sequence. It carries the associated amino acid for a given mRNA codon and is therefore critical in protein synthesis. There are several primary domains of the tRNA molecule, which is read in the 3'-to-5' direction. The vertical sequence at the top starting with the 3' end is called the acceptor stem, which covalently binds the specific amino acid. The loop on the right is called the T-arm, which is required for the RNA to bind to the ribosome. The D-arm lies opposite to the T-arm. The D-arm is necessary for the accurate recognition of the tRNA by the appropriate aminoacyl-tRNA synthetase. The vertically oriented loop pointing downwards is the anticodon arm, and the anticodon sequence refers to the three-base pair sequence at the bottom. In this instance, the anticodon sequence is 3'-UAC-5'. This corresponds to an mRNA sequence of 5'-AUG-3'. Incorrect Answers: B, C, D, E, and F. Choices B through F all represent mRNA codon sequences that have corresponding base pairs on the above tRNA molecule, but none of them are the anticodon for this specific 1RNA molecule. The anticodon is located at the bottom of the vertically oriented, downward-directed arm of the tRNA, which in this particular example is 3'-UAC-5'. Educational Objective: The anticodon of a tRNA molecule is the three-base pair sequence at the bottom of the anticodon arm that recognizes a corresponding MRNA codon. This is the location where tRNA binds to mRNA in the transfer of a specific amino acid during protein synthesis. A drug that binds to the anticodon sequence would prevent the tRNA from interacting with MRNA. Previous Next Score Report Lab Value

61 Exam Section 2: Item 12 of 50 National Board of Medical Examiners' Comprehensive Basic Science Self-Assessment 12. A3-year-old boy is brought to the physician because of developmental delay. He has a history of a cleft palate that was repaired surgically at the age of 1 year. He did not walk until the age of 18 months and did not speak any words until the age of 20 months. His height and weight are at the 5th percentile; his head circumference is below the 5th percentile. Examination shows ptosis and a smooth philtrum. Maternal use of which of the following substances during pregnancy is the most likely cause of this patient's condition? A) Alcohol B) Cocaine C) Heroin D) Marijuana E) Nicotine

A. Alcohol is a proven teratogen, as alcohol use during pregnancy leads to irreversible fetal neurological effects and physical malformations in a cluster of symptoms called fetal alcohol syndrome. Fetal alcohol syndrome is the leading cause of intellectual disability in the United States and would explain this patient's failure to meet developmental milestones. As a result of the failure of cell migration, fetal alcohol syndrome also presents with physical abnormalities such as microcephaly, facial abnormalities (small palpebral fissures, thin vermilion border, cleft lip/palate, smooth philtrum), limb deformities, and congenital heart disease. Though alcohol is associated with obstetric complications such as preterm birth that commonly result from the use of other substances during pregnancy, these neonatal malformations are specific to alcohol use. Incorrect Answer: B, C, D, and E. Cocaine (Choice B) and nicotine products (Choice E) cause vasoconstriction of the placental vasculature, which can lead to preterm birth, intrauterine growth restriction, low birth weight, and placental abruption. Maternal hypertension and preeclampsia can result from the use of cocaine or cigarettes. Smoking cigarettes can additionally lead to respiratory complications in the mother (eg, asthma) and poor fetal oxygen delivery. Heroin (Choice C) use during pregnancy may result in obstetric complications such as intrauterine growth restriction, placental abruption, preterm birth, fetal passage of meconium, and fetal death. Heroin withdrawal in the fetus can present as neonatal abstinence syndrome, which features irritability, a high-pitched cry, poor sleep, and uncoordinated sucking reflexes that can lead to poor feeding and longer postpartum hospital stays. Marijuana (Choice D) use during pregnancy leads to an increased risk for developmen

1 Exam Section 1: Item 1 of 50 National Board of Medical Examiners' Comprehensive Basic Science Self-Assessment 1. A 40-year-old woman at 5 months' gestation comes to the physician for amniocentesis. Results show a normal 46,XY karyotype of the fetus. Four months later, the newborn is delivered and examination shows a female phenotype with mild clitoral enlargement. Ultrasonography shows the presence of male but not female genital ducts. This newborn most likely has a mutation of the gene of which of the following factors? A) Anti-paramesonephric (müllerian) hormone B) 5a-Reductase C) SRY protein D) Steroid sulfatase E) Zinc finger transcription factor Wilms tumor 1

B. 5a-Reductase is an enzyme that catalyzes the metabolism of testosterone to dihydrotestosterone (DHT). Testosterone promotes the development of the mesonephric (wolffian) duct that develops into the seminal vesicles, epididymis, vas deferens, and ejaculatory duct. DHT promotes the development of male external genitalia and the prostate from the genital tubercle and urogenital sinus. Individuals with 5a-reductase deficiency have defective conversion of testosterone to DHT, leading to decreased concentrations of DHT and impaired virilization of the male urogenital tract. Individuals with 5a-reductase deficiency therefore appear phenotypically female externally or may have ambiguous external genitalia. Individuals have normal male internal genitalia as a result of normal concentrations of testosterone. Incorrect Answers: A, C, D, and E. Anti-paramesonephric (müllerian) hormone (Choice A) or müllerian inhibiting factor (MIF) is secreted by Sertoli cells and suppresses the development of the paramesonephric (müllerian) duct that would have developed into female internal genital structures such as the fallopian tubes, uterus, and upper vagina. The SRY gene is located on the Y chromosome and is responsible for producing testis-determining factor, also known as the SRY protein (Choice C), for testes development. Following testes development, hormones secreted by Sertoli cells (MIF) and Leydig cells (testosterone and DHT) promote the development of male internal and external genitalia and suppress the development of female structures. SRY gene translocation can occur during recombination in which the SRY gene on the Y chromosome becomes part of the X chromosome, leading to an XX embryo developing male characteristics. Steroid sulfatase (Choice D) is an enzyme that catalyzes the conversion of sulfated steroid precursors to

85 Exam Section 2: Item 36 of 50 National Board of Medical Examiners' Comprehensive Basic Science Self-Assessment 36. A 3-year-old boy is brought to the physician because of a 4-hour history of abdominal pain and vomiting. His mother says that she also has occasionally noticed several 20-cm, white worms in his stool during the past month. He recently immigrated to the USA from India with his family. Abdominal examination shows marked distention and tympany. Bowel sounds are high pitched. Which of the following is the most likely causal organism? A) Ancylostoma caninum B) Ascaris lumbricoides C) Enterobius vermicularis D) Strongyloides stercoralis E) Trichuris trichiura

B. Ascaris lumbricoides infection likely explains this patient's abdominal pain, vomiting, and presence of large white worms in the stool. Infection with Ascaris lumbricoides is called ascariasis. This worm is endemic to India and other parts of Asia, Africa, and South America, with transmission occurring via food or water contaminated with eggs, which must embryonate in the soil before becoming infectious. Ingested eggs hatch within days in the small intestine, releasing larvae that travel into the colon, followed by larval infiltration of the intestinal wall with subsequent hematogenous spread within the portal venous system to the liver. From here larvae can move to the heart and finally to the lungs where they mature over a period of two weeks, ascend the bronchi, and travel back down the esophagus after patients cough them up and swallow them. At this point, the larvae mature into adult worms that can measure up to 35 centimeters in length. Adult worms produce eggs that are released in the stool and begin the infectious cycle again. Adult worms live up to two years, and autoinfection does not occur, although reinfection is common. Clinical manifestations are related to the number of worms and the stage of the life cycle. Patients with a high volume of worms can present with bowel obstruction, which is likely the case in this patient presenting with abdominal pain and distension, vomiting, and borborygmi (high-pitched, hyperactive bowel sounds). Pulmonary manifestations include cough and eosinophilic pneumonitis. Some patients are entirely asymptomatic as a result of low worm burden. Incorrect Answers: A, C, D, and E. Ancylostoma caninum (Choice A) is a hookworm species that causes intestinal infection in dogs, not in humans. Enterobius vermicularis (Choice C) is an intestinal roundworm that causes perianal pruritu

58 Exam Section 2: Item 9 of 50 National Board of Medical Examiners' Comprehensive Basic Science Self-Assessment 9. A70-year-old man comes to the office because of a 6-day history of abdominal pain. He has no history of major medical illness and takes no medications. His vital signs are within normal limits. Physical examination shows epigastric tenderness. A CT scan of the abdomen shows a mass in the liver. As part of a study, electron microscopy of a biopsy specimen of the mass is done. Results show nuclear chromatin margination under the nuclear membrane, intact cell membranes, and cells that are smaller in size than normal cells in the tissue. The mechanism causing the cell size anomaly in this patient was most likely triggered by the release of which of the following substances into the cytoplasm? A) BAX protein B) BCL2 protein C) Cytochrome c OD) Procaspase-8 E) Procaspase-9

C. Apoptosis may be triggered through the intrinsic pathway by cellular damage such as radiation, oxidative damage, ischemia, or toxin exposure, or through the extrinsic pathway by the activity of perforin, granzyme, tumor necrosis factor, or the interaction between Fas and Fas ligand. Natural killer cells induce apoptosis in cells that no longer express major histocompatibility complex class I, the loss of which is a marker of malignant transformation. Apoptosis initiated in this manner occurs via the extrinsic pathway and involves caspases 8, 10, and 3. Both pathways converge upon the release of proapoptotic factors from the mitochondria. Regulatory proteins, including BAX and BAK, interact with the mitochondrial outer membrane and induce permeability that leads to the release of cytochrome c into the cytoplasm. Once in the cytoplasm, cytochrome c activates caspase enzymes. Cellular features of apoptosis include cell shrinkage caused by degradation of the cytoskeleton, nuclear pyknosis (condensation of nuclear chromatin), karyorrhexis (fragmentation of the nucleus), and blebbing of the cellular membrane. Incorrect Answers: A, B, D, and E. BAX protein (Choice A) is found in the cytoplasm under normal conditions. Upon initiation of apoptosis, BAX inserts itself into the outer mitochondrial membrane, allowing for the release of cytochrome c. BCL2 protein (Choice B) is found in the cytoplasm under normal conditions, where it is bound to the outer mitochondrial membrane. BCL2 inhibits the action of proapoptotic factors such as BAX and BÁK, and promotes cell survival. Overexpression of BCL2 is a feature of malignancies such as B-cell lymphoma, from which BCL2 protein derives its name. Procaspase-8 (Choice D) and procaspase-9 (Choice E) are found in the cytoplasm and are inactive precursors to caspase enzymes. Caspases exi

67 Exam Section 2: Item 18 of 50 National Board of Medical Examiners' Comprehensive Basic Science Self-Assessment 18. A 45-year-old man is undergoing a surgical procedure. Which of the following maneuvers by the anesthesiologist would result in an increase in intracranial pressure? A) Decreasing airway pressures B) Decreasing fractional inspired oxygen (F102) C) Decreasing respiratory rate D) Increasing airway pressures E) Increasing Fio2 F) Increasing respiratory rate

C. Intracranial pressure (ICP) is dependent on the volumes of the brain, cerebrospinal fluid (CSF), and blood as a result of the fixed space of the skull. Brain volume can be increased with a mass or cerebral edema secondary to ischemic injury. CSF volume can be increased by increased production, decreased reabsorption, or an obstruction that prevents drainage. Blood volume can be increased in the extravascular or intravascular space. Extravascular blood accumulation occurs in the setting of intracerebral hemorrhage, epidural hematoma, subdural hematoma, and subarachnoid hemorrhage. However, the primary way an anesthesiologist or other provider can alter ICP is through alterations in cerebral blood flow (CBF) or intravascular blood volume present in the cranial vault. As a result of autoregulation, the CBF is maintained at a nearly constant level between blood pressures of 60 to 160 mm Hg. CBF, however, increases with increased metabolic demand, hypercarbia, hypoxemia, hyperthermia, and increased central venous pressure (CVP). The most marked effect on CBF is seen with hypercarbia, with an approximate increase of 1 to 2 milliliters per 100 grams per minute per millimeter of mercury in the arterial pressure of carbon dioxide (PaCO,). Therefore, decreasing the respiratory rate or the tidal volume, and thereby increasing PaCO, would result in an increase in CBF and a consequent increase in ICP. Incorrect Answer: A, B, D, E, and F. Decreasing airway pressures (Choice A) would result in a multitude of effects such as preventing barotrauma, potentially decreasing the arterial pressure of oxygen (Pao,) if the positive end- expiratory pressure (PEEP) were decreased, and lowering CVP by decreasing intrathoracic pressure. However, hypoxemia does not cause increases in ICP unless severe and the resultant decreases in CVP would de

2 Exam Section 1: Item 2 of 50 National Board of Medical Examiners' Comprehensive Basic Science Self-Assessment 2. A 16-year-old girl with cystic fibrosis is brought to the physician because of a 3-week history of generalized weakness, numbness and tingling of her arms and legs, and difficulty walking. She has not adhered to her medication regimen during the past 6 months. She appears alert and oriented. Her vital signs are within normal limits. Physical examination shows bilateral weakness and decreased deep tendon reflexes in the upper and lower extremities. She walks with an ataxic gait. The most likely cause of these findings is a deficiency of which of the following? A) Folic acid B) Vitamin A C) Vitamin B, (pyridoxine) D) Vitamin D E) Vitamin E

E. Vitamin E is an antioxidant that protects erythrocytes and cells from free radical damage. Deficiency may present with hemolytic anemia and generalized muscle weakness. It can have a similar presentation to vitamin B12 (cobalamin) deficiency with posterior column and spinocerebellar tract demyelination. In contrast to vitamin B12 deficiency, patients do not have megaloblastic anemia, hypersegmented neutrophils, or increased serum methylmalonic acid concentrations. On peripheral smear, patients with vitamin E deficiency may have acanthocytosis. Incorrect Answers: A, B, C, and D. Folic acid (Choice A) is converted to tetrahydrofolic acid and used as a coenzyme in the synthesis of nucleotides and nucleosides. Folate is contained in leafy vegetables and absorbed in the jejunum. Folate deficiency is often seen in patients with malnutrition, alcoholism, and patients taking anti-folate medications (eg, phenytoin, methotrexate). Megaloblastic anemia occurs in the setting of impaired DNA synthesis. Vitamin A (Choice B) is an antioxidant necessary for differentiation of epithelial cells into specialized tissue. Deficiency is characterized by ocular manifestations including night blindness, corneal degeneration, Bitot spots on the conjunctiva, dry skin, and immunosuppression. Vitamin B6 (pyridoxine) (Choice C) deficiency limits synthesis of histamine, hemoglobin, and neurotransmitters including epinephrine, norepinephrine, dopamine, serotonin, and GABA. Deficiency commonly presents with peripheral neuropathy, dermatitis, sideroblastic anemia, glossitis, and seizures (especially in the setting of isoniazid use). Vitamin D (Choice D) deficiency commonly comes from low exposure to UV radiation and low dietary vitamin D intake. Fat malabsorption syndromes such as celiac disease or cystic fibrosis can impair absorption of vitamin D

19 Exam Section 1: Item 19 of 50 National Board of Medical Examiners' Comprehensive Basic Science Self-Assessment 19. A 19-year-old woman comes to the physician because of a 3-day history of fever, shortness of breath, cough productive of blood-tinged sputum, rash on her legs, and pain of her groin. She lives on a small farm in rural New Mexico and says that all three of her cats have been infested with fleas. She is in respiratory distress. Her temperature is 39°C (102.2°F), pulse is 104/min, respirations are 28/min, and blood pressure is 100/60 mm Hg. Physical examination shows a 1-cm ulcer on the right lower extremity and numerous erythematous macular lesions over both lower extremities. There is a fluctuant 3 x 5-cm mass in the femoral lymph node chain. A chest x-ray shows bilateral infiltrates. A Gram stain of purulent material aspirated from the femoral mass shows numerous neutrophils and few gram-negative ba

E. Yersinia pestis is the causal organism responsible for this patient's sepsis and ulceroglandular disease, which is consistent with the bubonic plague. Yersinia pestis is a gram-negative bacillus that is most often transmitted by infected fleas, but can also be acquired via direct handling of infected animals (eg, cats), consumption, or aerosolized droplets from other infected humans. This patient's right lower extremity ulcer likely represents the site of an initial flea bite, and the femoral lymph node mass is known as a buboe, which is a large inflammatory infiltrate containing Y. pestis in a lymph node that drains the infected leg. Pneumonia, which this patient also demonstrates, can either occur as a primary syndrome or can develop as a complication (termed secondary pneumonic plague) of disseminated disease and bacteremia. Disseminated intravascular coagulation can occur with disseminated disease, and meningitis is an additional complication. Leukocytosis and thrombocytopenia are common laboratory findings. Treatment is primarily with aminoglycosides although doxycycline is a valid alternative therapy. Incorrect Answers: A, B, C, and D. Coxiella burnetii (Choice A) is an intracellular gram-negative bacterium that causes Q fever. Exposure occurs through contact with contaminated livestock. It is found throughout the United States. Acute infection presents with high fevers, myalgias, malaise, and gastrointestinal symptoms; ulceroglandular disease is not a feature. Chronic Q fever can result in endocarditis. Ehrlichia chaffeensis (Choice B) is an intracellular bacterium that causes ehrlichiosis, which is characterized by fevers, myalgias, headache, and occasionally neurologic symptoms. It is endemic in the southeast, mid-Atlantic, and central United States, but the vector is a tick, not a flea. Common laboratory f

89 Exam Section 2: Item 40 of 50 National Board of Medical Examiners' Comprehensive Basic Science Self-Assessment 40. A 70-year-old woman comes to the physician for advice after her husband of 45 years suddenly dies of cardiac arrest. She wants to donate $1,000,000 to a small city's department of public health to decrease the number of sudden cardiac deaths. She asks the physician to recommend the most effective way to put this money to use. She says, "I want to see the results of my donation over the next 1 to 2 years." The patient is most likely to accomplish her goal if she contributes to which of the following? A) Creation of an antismoking campaign B) Creation of a citywide aerobic exercise program C) Fortification of the water supply with aspirin D) Placement of external automatic defibrillators in public spaces throughout the city E) Provision of free HMG-COA reductase inhibitor drugs to everyone in town over

D. Cardiac arrests in public places (nonhealthcare settings) affect thousands of persons in the United States every year. The effectiveness of defibrillation is greatest at the earliest possible time after cardiac arrest. Easy to use, battery-powered automatic external defibrillators (AEDS) placed in public areas have been shown to be highly effective in preventing sudden cardiac death from ventricular fibrillation or ventricular tachycardia. Thus, if the individual wishes to support an effective public health intervention to decrease the incidence of sudden cardiac death over the next 1 to 2 years, placement of AEDS in public places throughout the city should be considered a viable option. This treatment is on-target with respect to the proximate cause of sudden cardiac death. Incorrect Answers: A, B, C, E, and F. Creation of an antismoking campaign (Choice A) may be an effective public health intervention; however, the effects of smoking occur over a long period of time and the impact of such an intervention would likely not be measurable for many years. Further, there are many factors that influence smoking such as advertising campaigns promoting smoking and the addictive properties of nicotine. Smoking is an indirect cause of sudden cardiac death, leading to it through increasing the risk for atherosclerotic cardiovascular disease and structural and ischemic heart disease. Creation of a citywide aerobic exercise program (Choice B) would provide numerous health benefits to the community. Exercise has been shown to decrease the risk for cardiac disease, obesity, and depression. However, the effects of this intervention would be difficult to measure in the short term. Fortification of the water supply with aspirin (Choice C) is inappropriate, as aspirin has been shown to have benefits in patients at high risk for acut

49 Exam Section 1: Item 50 of 50 National Board of Medical Examiners' Comprehensive Basic Science Self-Assessment 50. A 75-year-old woman is brought to the physician because of a 1-day history of fever and back pain. Her temperature is 39.5°C (103.1°F), pulse is 111/min, respirations are 32/min, and blood pressure is 115/79 mm Hg. Physical examination shows left-sided costovertebral angle tenderness. Laboratory studies show a leukocyte count of 17,000/mm3 (with 9% bands) and pyuria. Urine cultures grow Escherichia coli. It is determined that the patient's fever is partially due to interleukin-6 (IL-6), which was induced by the IL-1 produced during the immune response. Which of the following best describes the role of IkB in the nuclear factor-kappa B (NF-KB) signal transduction pathway from IL-1 binding to IL-6 induction in this patient? A) Attaches to cytokine receptor B) Facilitates proteolytic cleavage of NF-KB

D. NF-KB (nuclear factor kappa-light-chain-enhancer of activated B cells) comprises a family of transcription factors that induce the expression of several proinflammatory genes, promoting the production of cytokines, chemokines, and other inflammatory mediators in response to infection, as in this patient's case of pyelonephritis. The NF-KB pathway may be activated by several stimuli, including the presence of IL-1, reactive oxygen species, tumor necrosis factor-alpha, and binding of pathogen-associated molecular patterns (PAMPS) such as bacterial lipopolysaccharides to toll-like receptors on leukocytes. NF-KB is normally present in the cytoplasm bound to the inhibitor protein IKB that renders it inactive. In the signaling pathway, a ligand (such as IL-1) binding to the appropriate receptor results in activation of the IKB kinase (IKK) complex. Activated IKK phosphorylates IKB, which releases NF-KB after undergoing phosphorylation. NF-KB is then able to translocate to the nucleus and upregulate transcription and translation of proinflammatory genes. Increased synthesis of IL-6 is induced by this pathway. Incorrect Answers: A, B, C, and E. IL-1 attaches to cytokine receptors (Choice A) on inflammatory cells, which is one of the signals that initiates the NF-KB pathway. Conversely, IkB is located within the cytoplasm, bound to NF-KB. IKB does not bind a cytokine receptor. There are bacterial proteases that facilitate proteolytic cleavage of NF-KB dimers (Choice B), which act as virulence factors expressed by some strains of Escherichia coli and Chlamydia species. Cleavage of the NF-KB dimer renders it ineffective in promoting gene expression. This is not the mechanism of IkB. Phosphorylates NF-KB (Choice C) is not the role of IKKB. IKB is phosphorylated by IKK leading to release of the NF-KB dimer, which then translocat

92 Exam Section 2: Item 43 of 50 National Board of Medical Examiners' Comprehensive Basic Science Self-Assessment 43. A 64-year-old woman comes to the physician because of a 6-month history of heartburn and difficulty swallowing solid food and liquid. She has not had nausea or vomiting. She also has a 2-year history of pain and swelling of her wrists and hands. She has not seen a physician in 5 years. She takes no medications. She does not smoke. Physical examination shows no wrinkles on the face and neck. Her vital signs are within normal limits. There is synovial thickening of the wrists bilaterally. A photograph of the left hand is shown. Abdominal examination shows no abnormalities. Which of the following sets of findings is most likely on esophageal manometry? Esophageal Peristalsis Lower Esophageal Sphincter Tone A) Increased increased B) Increased decreased C) Normal increased D) Normal decreased E) Decreased

F. Localized scleroderma, also called CREST syndrome, is characterized by calcinosis cutis, Raynaud phenomenon, esophageal dysmotility, sclerosis, and telangiectasias. Esophageal dysmotility, evidenced by heartburn and dysphagia, along with tightening of the skin on the face, around the mouth, and on the fingers is suggestive of this diagnosis. The esophageal dysmotility is caused by an infiltration of the esophageal muscular layer with fibrous tissue, which substantially decreases normal peristaltic activity. Infiltration of the lower esophageal sphincter by fibrosis prevents it from contracting and generating adequate tone to prevent reflux of gastric contents into the esophagus. Esophageal manometry measures the pressure generated along the length of the esophagus and lower esophageal sphincter and can be utilized in the evaluation of suspected esophageal dysmotility disorders such as CREST syndrome, achalasia, and diffuse esophageal spasm. Incorrect Answers: A, B, C, D, and E. Esophageal peristalsis relies on contraction of the muscular layer of the esophageal wall. Decreased, not increased (Choices A and B), contraction of this muscle layer characterizes CREST syndrome. Though it does not generate successful peristalsis, diffuse esophageal spasm is a condition in which the muscular layer of the esophagus is overactive. These spasms cause severe dysphagia and chest pain, often acutely in the setting of swallowing. Lower esophageal sphincter tone is increased (Choices C and E) in achalasia, a condition in which there is impaired relaxation of the lower esophageal sphincter. It manifests as dysphagia, odynophagia, weight loss, halitosis, and regurgitation of undigested food. It is diagnosed through a barium swallow plus esophageal manometry. The other findings of CREST syndrome are not present. Normal peristalsis and

9 Exam Section 1: Item 9 of 50 National Board of Medical Examiners' Comprehensive Basic Science Self-Assessment 9. A 3-year-old girl is brought to the emergency department by her mother 1 hour after she was found with a half-empty bottle of her grandmother's diabetes medication. The mother tells the physician that the child consumed approximately 25 metformin tablets. Physical examination shows no abnormalities. This patient is at greatest risk for which of the following serum abnormalities? A) Decreased calcium concentration B) Decreased glucose concentration C) Decreased sodium concentration D) Increased AST and ALT activities E) Increased creatinine concentration F) Increased lactic acid concentration

F. Metformin is an oral biguanide agent used in the management of type 2 diabetes mellitus. It decreases gluconeogenesis, increases peripheral tissue glucose uptake, and decreases serum free fatty acid concentration. Because of its efficacy, tolerability, safety, and low cost, it serves as the first-line treatment for type 2 diabetes mellitus. Although it is generally safe and well-tolerated, there are a few contraindications to metformin, including impaired renal function (caused by its excretion in the urine) and active liver disease. Metformin may cause several adverse effects, including nausea, abdominal discomfort, diarrhea, and (rarely) increased lactic acid concentration resulting in lactic acidosis. Although the incidence is rare, metformin-associated lactic acidosis has a high mortality rate. Treatment is supportive, and patients with severe lactic acidosis may require hemodialysis. Metformin does not generally cause hypoglycemia. Incorrect Answers: A, B, C, D, and E. Decreased calcium concentration (Choice A) is not a common side effect of metformin. Since metformin does not stimulate insulin release, hypoglycemia and decreased glucose concentration (Choice B) are not common. Hypoglycemia can be an adverse effect in the setting of an insulin or sulfonylurea overdose. Decreased sodium concentration (Choice C) can occur in syndrome of inappropriate antidiuretic hormone secretion (SIADH) resulting in excessive free water retention and euvolemic hyponatremia. SIADH is a common side effect of many medications (eg, cyclophosphamide) but not metformin. Increased AST and ALT activities (Choice D), seen in medication-induced hepatotoxicity, can be caused by oral hypoglycemic drugs such as glitazones (eg, pioglitazone, rosiglitazone). They increase peripheral tissue insulin sensitivity and can be hepatotoxic. Increased

24 Exam Section 1: Item 24 of 50 National Board of Medical Examiners' Comprehensive Basic Science Self-Assessment 24. A 60-year-old man comes to the physician because of a 1-month history of progressive shortness of breath with exertion. Examination of the chest shows: Breath sounds Percussion note Tactile fremitus Adventitious sounds Right Lung Base decreased dullness decreased Left Lung Base normal normal normal crackles none Which of the following is the most likely diagnosis? A) Asthmatic bronchitis O B) Bronchiectasis C) Bullous emphysema D) Chronic bronchitis E) Lobar pneumonia F) Pleural effusion G) Pneumothorax H) Pulmonary embolism

F. Unilateral decreased breath sounds, dullness to percussion, and decreased tactile fremitus are most suggestive of a pleural effusion. Fluid occupying the space between the lung and the body wall dampens transmission of sound waves and mechanical vibrations (in contrast to consolidated lung, which presents with decreased breath sounds as a result of decreased air movement into the region of consolidation and increased tactile fremitus as dense, fluid-filled lung tissue transmits mechanical vibrations better than ventilated lung). Unilateral crackles (rales) are nonspecific. Pleural effusions occur as a result of multiple underlying conditions, including infection such as pneumonia, malignancy such as lung cancer or mesothelioma, and increased hydrostatic pressure such as left-sided heart failure. They can be visualized on imaging including chest x-ray, CT scan, or ultrasound. A diagnostic thoracentesis should be performed whenever the etiology of the effusion is unknown. A therapeutic thoracentesis can be used to relieve clinical symptoms. Incorrect Answers: A, B, C, D, E, G, and H. Asthmatic bronchitis (Choice A) is caused by short-term inflammation of the bronchi and is commonly provoked by environmental triggers such as pollen, molds, smoke, cold, and exertion. Auscultation typically discloses wheezes. Bronchiectasis (Choice B) is abnormal dilatation of the bronchi that may result from recurrent necrotizing infections and chronic inflammation. Risk factors for bronchiectasis are cystic fibrosis, Kartagener syndrome, and allergic bronchopulmonary aspergillosis. Bullous emphysema (Choice C) describes structural changes to the lung which result in abnormal and enlarged air spaces distal to the terminal bronchioles with loss of individual alveoli. There is noted absence of fibrotic changes. Emphysema is associated wit

4 Exam Section 1: Item 4 of 50 National Board of Medical Examiners' Comprehensive Basic Science Self-Assessment 4. A 55-year-old man with long-standing type 2 diabetes mellitus comes to the physician because of a 2-day history of fever, chills, nausea, and swelling of his left leg. His temperature is 38.9°C (102°F), and pulse is 110/min. Examination of the left lower extremity shows a warm, tender, erythematous, blanching rash. A photograph of the left lower extremity is shown. A photomicrograph of a Gram stain of a blood culture is shown. Which of the following is the most likely causal organism? A) Clostridium perfringens B) Enterococcus faecalis C) Haemophilus influenzae D) Pasteurella multocida E) Pseudomonas aeruginosa F) Staphylococcus aureus G) Streptococcus pyogenes (group A)

G. Cellulitis presents with cutaneous erythema, warmth, and induration, often following inoculation from an injury such as an abrasion. Systemic symptoms, including fever, tachycardia, and leukocytosis are often present. Most commonly, it is acute and results from a bacterial infection of the skin. The two most common pathogens to cause cellulitis are Staphylococcus aureus and Streptococcus pyogenes (group A). S. pyogenes is a gram-positive bacterium that forms chains. It is part of the B-hemolytic group of Streptococcus species and is bacitracin sensitive. Clinically, the cellulitis caused by S. aureus is typically purulent while that caused by S. pyogenes is not. In addition to cellulitis, infection with S. pyogenes can result in various diseases such as pharyngitis, scarlet fever, necrotizing fasciitis, glomerulonephritis, and rheumatic fever. Incorrect Answers: A, B, C, D, E, and F. Clostridium perfringens (Choice A) is an anaerobic gram-positive bacillus. It often infects an open wound and causes necrotizing fasciitis. Clostridium perfringens secretes alpha toxin, a phospholipase that degrades cell membranes to cause gas gangrene. Enterococcus faecalis (Choice B) belongs to Group D Streptococcus, which are gamma hemolytic. Gamma-hemolytic species are identified by their ability to grow in bile and 6.5% sodium chloride solution. Clinically, Enterococcus faecalis is a common cause of nosocomial urinary tract infections. Haemophilus influenzae (Choice C) is an encapsulated gram-negative bacterium that can result in various mucosal infections such as conjunctivitis, otitis media, bronchitis, and pneumonia. It was the leading cause of epiglottitis in children prior to widespread immunization. It is an uncommon cause of cutaneous infections. Pasteurella multocida (Choice D) is a gram-negative coccobacillus, which is fre

32 Exam Section 1: Item 33 of 50 National Board of Medical Examiners' Comprehensive Basic Science Self-Assessment 33. A healthy 25-year-old man participates in a study of muscle function. The electrophysiologic observations made on a muscle biopsy specimen are shown. Via iontophoresis, 1 µM acetylcholine (ACh) was applied to the muscle surface. Extracellular Ca2+ concentration was decreased to prevent end-plate potentials from acting as a suprathreshold for muscle action potentials. epp Amplitude (in mV) 10 mepp Amplitude (in mV) 1 Response (in mV) to ACh ( 1 μΜ) 1 Normal muscle epp=end-plate potential; mepp= miniature epp Based on these findings, which of the following electrophysiologic characteristics is expected in a muscle biopsy specimen from a patient with acute botulism? epp Amplitude (in mV) mepp Amplitude (in mV) Response (in mV) to ACh (1 μΜ) A) 1 0.5 0.5 B) 1 1 C) 2 0.2 0.2 D) 0.2 0.1 E) 15 1 1

B. Skeletal muscle activity is controlled by neurons which synapse on muscles at the neuromuscular junction. At this junction, the action potential from the neuron signals for the presynaptic release of acetylcholine leading to endplate potentials on the myocyte membrane. These endplate potentials then stimulate the dihydropyridine receptors and subsequently the ryanodine receptors which cause the release of calcium from the sarcoplasmic reticulum. This calcium then activates the binding of myosin and actin leading to muscle contraction. Endplate potentials are the depolarization events that occur at the neuromuscular junction which lead to muscle fiber depolarization and subsequent contraction. Miniature endplate potentials are the small depolarization events that occur from the release of a single vesicle of acetylcholine into the neuromuscular junction. These potentials are not sufficient for reaching the action potential threshold on the muscle cell membrane. These miniature potentials occur sporadically when no depolarization signal is given to the neuromuscular junction as there is a small amount of baseline vesicle release. Given that the extracellular calcium concentration was significantly decreased from physiologic values in this experiment, less acetylcholine will be released from the axon terminal when an action potential occurs as this is a calcium-dependent process. Miniature endplate potentials are caused by ligand-gated nonspecific cation channels and are more dependent on the concentrations of potassium and sodium as well as calcium. Thus, miniature endplate potentials will not vary significantly based on calcium concentration. Similarly, the response to 1 µM acetylcholine placed in solution will be relatively unchanged. The major change as a result of botulinum toxin will be failure of the release of

77 Exam Section 2: Item 28 of 50 National Board of Medical Examiners' Comprehensive Basic Science Self-Assessment 72% 100% Superior vena cava Pulmonary veins Right atrium 84% 95% Left atrium Right ventricle 84% 95% Left ventricle Pulmonary artery Aorta 84% 95% 28. A 10-year-old girl is referred to a cardiologist because of a 6-year history of a murmur. She is asymptomatic. Physical examination shows a grade 2/6, systolic ejection murmur heard over the left sternal border. An ECG shows right axis deviation and right ventricular hypertrophy. She undergoes cardiac catheterization. Oxygen saturation findings are shown in the block diagram. This patient most likely has which of the following congenital heart defects? A) Abnormal pulmonary venous return B) Atrial septal defect C) Membranous ventricular septal defect D) Tetralogy of Fallot

B. Atrial septal defect is a common congenital malformation of the interatrial septum. The most common type is an ostium secundum defect, although ostium primum defects are commonly associated with trisomy 21. The atrial septal defect results in a left-to-right shunt with abnormal flow of blood from the left atrium to the right atrium, resulting in relative volume overload of the right atrium and ventricle. The presence of a channel between the left and right atrium leads to mixing of oxygenated blood from the left atrium with returning venous blood from the superior vena cava, resulting in a higher oxygen saturation in the right atrium than expected (as seen in the patient's cardiac catheterization findings). Atrial septal defects are also associated with a fixed, split S2 as a result of increased blood flow across the pulmonic valve, and a low-grade ejection murmur on cardiac auscultation. The increased right heart volumes also result in a prominent right ventricular impulse on physical examination and may present an increased risk for the development of a right bundle branch block. Chest x-rays characteristically demonstrate increased caliber of the main pulmonary artery and increased pulmonary vascular markings. If the atrial septal defect remains uncorrected, it can result in the development of Eisenmenger syndrome secondary to prolonged pulmonary vasculature remodeling with consequent pulmonary arterial hypertension and shunt reversal leading to cyanosis. Incorrect Answers: A, C, D, and E. Abnormal pulmonary venous return (Choice A) occurs in total or partial anomalous pulmonary venous return, a rare congenital malformation in which one or more of the four pulmonary veins do not insert appropriately into the left atrium and instead drain into the right atrium. Increased right atrial and decreased left atrial oxyg

81 Exam Section 2: Item 32 of 50 National Board of Medical Examiners' Comprehensive Basic Science Self-Assessment 32. A 57-year-old man comes to the physician because of increasingly severe joint pain and weakness during the past 2 months. Physical examination shows pallor. Laboratory studies show: 8 g/dL 25% Hemoglobin Hematocrit Leukocyte count Myeloid cells (immature and mature) Platelet count 150,000/mm3 90% 50,000/mm3 present Philadelphia (Ph') chromosome A chimeric growth-promoting protein with which of the following activities is most likely encoded in this patient due to the presence of this chromosome? A) CAMP B) Dihydrofolate reductase C) MAP kinase D) Platelet-derived growth factor E) Tyrosine kinase

E. This patient carries a diagnosis of chronic myelogenous leukemia (CML), which is defined by the Philadelphia chromosome (Ph). The Ph chromosome is created by a translocation between chromosomes 9 and 22, leading to constitutive activation of the ABL1 tyrosine kinase. Activation of the ABL1 tyrosine kinase leads to subsequent activation of the JAK/STAT and Ras/MAPK/ERK pathways, with the downstream consequences of increased cellular proliferation, loss of normal checkpoint inhibition, and resistance to apoptosis. While the Ph chromosome is required for the diagnosis of CML, it can also be found in acute lymphoblastic leukemia (ALL) and acute myeloid leukemia (AML). Typical laboratory findings in CML include hyperleukocytosis, which is present in this case, with an increase in nearly all of the myeloid cell lines. Basophilia and eosinophilia are characteristic. Leukostasis, a condition in which hyperleukocytosis leads to end organ damage from occlusion of capillaries by malignant, nondistensible cells, is uncommon in CML because malignant cells are well differentiated and pliable. This is in contrast to conditions such as AML and ALL in which circulating blasts, which are large and not particularly pliable, easily lodge in capillaries to cause pulmonary and neurologic complications. Treatment of CML is primarily with tyrosine kinase inhibitors such as imatinib or desatinib. They are remarkably effective and often result in complete molecular response within several months of initiation. Incorrect Answers: A, B, C, and D. CAMP (Choice A) is a ubiquitous signaling molecule critical in a variety of cellular signaling pathways. Mutations in pathways that utilize CAMP are found in many types of cancer, including breast, liver, skin, and lung cancer. CAMP is not necessarily implicated in the pathogenesis of CML. Dihydrofola

14 Exam Section 1: Item 14 of 50 National Board of Medical Examiners' Comprehensive Basic Science Self-Assessment 14. In the hypothetical distribution of laboratory values for a particular laboratory test in "normal" and "diseased" populations, respectively, which of the following lettered decision points will produce the maximum sensitivity for the detection of the disease? Normal Number of Diseased subjects tested D Range of values for a particular test- A) B) C) D) E)

A. Any population of patients will include those who have the disease and those who do not, and overlap may exist between these groups when considering testing with a range of possible values. For example, one diagnostic criteria for type 2 diabetes mellitus is two fasting serum blood glucose concentrations greater than 126 mg/dL. However, not all patients with fasting serum blood glucose greater than 126 mg/dL will truly have diabetes mellitus. These patients may receive positive tests but do not have the underlying condition (false-positive result). Therefore, the set points for diagnostic tests will result in some true positives (patients who have the disease and who the tests identifies as having the disease), true negatives (patients who do not have the disease who the test identifies as not having the disease), false positives (patients who do not have the disease who the test identifies as having the disease), and false negatives (patients who have the disease who the test identifies as not having the disease). The set point will determine the fraction of patients who are true- or false-positive and true- or false-negative and should be chosen to optimize the sensitivity and specificity of the test. If the graph were being used to determine the sensitivity and specificity of the fasting serum blood glucose test in diagnosing diabetes mellitus, Point A would result in all persons with type 2 diabetes mellitus being identified as positive. This would mean the number of false negatives would be zero, and the number of true positives would be high. Moving the cut point from point À to point B (raising the threshold for a positive test result) would cause fewer people with the disease to have a positive test, therefore increasing the number of false negatives and decreasing the number of false positives. Sensitivity

53 Exam Section 2: Item 4 of 50 National Board of Medical Examiners' Comprehensive Basic Science Self-Assessment 4. A 20-year-old woman is brought to the emergency department by her roommate 30 minutes after she ingested a large quantity of acetaminophen tablets during a suicide attempt. The physician asks her why she tried to kill herself. She replies tearfully, "My boyfriend told me that he doesn't want to see me again, and he won't return any of my phone calls. I loved him more than I've loved anyone else in my entire life. I was going to marry him! Now I hate his guts for what he's done to me. I just wanted to die." On further questioning, the physician learns that she had only two dates with this man. She tells the physician, "I just can't bear being alone. But I can tell that you understand. You're the only doctor who's ever understood how I feel." This patient most likely has which of the following types of pe

A. Borderline personality disorder (BPD) is a cluster B personality disorder, the emotional or dramatic cluster, that features an unstable sense of self and tumultuous relationships. Likely caused by a combination of genetic polymorphisms in serotonin and dopamine receptors and emotional invalidation during childhood, patients with BPD unconsciously learn to make impulsive and dramatic gestures, including self-harm, to obtain attention and emotional fulfilment from others. Chronic invalidation also leads to poor self-esteem and the consequent reliance on others for self-esteem needs, resulting in an intense fear of abandonment and severe distress when abandonment happens. As a result of inadequate emotional attunement from caregivers, patients with BPD do not learn to label, understand, or regulate their emotions. Their negative emotional experiences are so intensely painful that people with BPD must separate these experiences from their consciousness. This splitting leads people with BPD to black-and-white thinking that includes seeing people as all good and others as all bad, as in this patient. Incorrect Answers: B, C, D, and E. Dependent personality disorder (Choice B) is a cluster C personality disorder, which is the anxious cluster. Dependent personality disorder presents with an excessive need to be cared for by others that manifests as severe separation anxiety and clinging behavior. Though this patient does illustrate clinging behavior, she also illustrates difficulty with emotional regulation, black-and- white thinking, and suicidal gestures that are more typical of BPD. Histrionic personality disorder (Choice C), a cluster B personality disorder, is characterized by theatrical, superficial expressions of emotion that unconsciously serve to garner attention from others to fulfill self-esteem needs. These pati

70 Exam Section 2: Item 21 of 50 National Board of Medical Examiners' Comprehensive Basic Science Self-Assessment 21. A 60-year-old man comes to the emergency department because of swelling of his right elbow that began after he struck it against the edge of a file cabinet earlier that day. On physical examination, the tip of the olecranon is obscured by a golf ball-sized fluctuant mass with well-defined borders. The mass is swollen, mildly tender, and smooth. Range of motion of the elbow is normal. Which of the following structures was most likely injured in this patient? A) Bursa B) Cartilage C) Joint capsule D) Ligament E) Synovium OF) Tendon

A. Bursae are thin, sac-like, fluid-filled structures that decrease friction and permit motion between layers of connective tissue. Bursae are frequently implicated in pathology and are often associated with pain and swelling, such as the subacromial bursa which can cause pain in the shoulder, the trochanteric bursa which can cause pain over the lateral aspect of the hip, and the olecranon bursa which can cause pain and swelling posterior to the elbow. When inflamed or injured, fluid can collect inside the potential space of the bursa, causing swelling that can be appreciated on examination as a fluctuant, tender, circumscribed mass. Olecranon bursitis demonstrates such findings posterior to the elbow and may be painful to the touch. If erythema and warmth are present, these findings can signify infection of the bursa (septic bursitis). Incorrect Answers: B, C, D, E, and F. Cartilage (Choice B) is a specialized connective tissue that provides both structure and flexibility. Hyaline cartilage provides the smooth surface of joints; fibrocartilage provides the flexible structure of the intervertebral discs and pubic symphysis. Joint capsule (Choice C) is the collagen-rich ligamentous tissue that surrounds joints. It provides stability to the joint as well as contains synovial fluid within the joint space. Infection inside the joint capsule is known as septic arthritis and presents with fever, significant pain with range of motion, swelling of the joint, erythema, and inability to bear weight on the joint. Ligaments (Choice D) are organized structures of collagen, type I, that provide stability to joints and define specific ranges of motion. For example, the anterior cruciate ligament of the knee provides resistance to anterior translation of the tibia on the femur and limits hyperextension of the knee. Synovium (Choice E)

18 Exam Section 1: Item 18 of 50 National Board of Medical Examiners' Comprehensive Basic Science Self-Assessment 18. A 56-year-old man with a history of alcohol dependence comes to the emergency department because of fever, night sweats, and a cough productive of bloody sputum. Laboratory studies of his sputum show numerous acid-fast rods arranged in small arrays. Which of the following components of the causal organism may be directly cytotoxic to macrophages? A) Cord factor B) Cytoplasmic membrane C) Endotoxin D) Polysaccharide capsule E) Teichoic acid

A. Cord factor is a glycolipid found as part of the cell wall of Mycobacterium tuberculosis (MTB) and is responsible for much of its virulence. Ingestion of MTB by phagocytes should lead to the intracellular killing of the organism, but the presence of cord factor prevents fusion of the vesicles containing MTB with the lysosome, thereby preventing intracellular killing. Additionally, cord factor is directly cytotoxic to macrophages. It also plays a role in the organization of MTB in various tissues, and this ability to organize MTB into straight, closely grouped arrays gives cord factor its name. Finaly, cord factor is thought to be responsible for inducing secretion of many of the cytokines that are important in granuloma formation. The virulence of MTB is complex and includes the presence of cell wall lipids, cytoplasmic transport proteins, and certain enzymes, but cord factor is known to be one of the most important virulence factors associated with bacterial growth and survival. Incorrect Answers: B, C, D, and E. The cytoplasmic membrane (Choice B) is a lipid bilayer containing various cellular proteins that encapsulates the cytoplasm. It houses several transporter proteins, including ESX and TAT proteins, involved in iron and zinc transport, but none of these contribute as much to the virulence of MTB as cord factor. Endotoxin (Choice C) production is not a feature of MTB but is commonly seen in infections with gram-negative bacteria, which contain lipopolysaccharide (LPS) in their cell wall. LPS is highly immunogenic and is involved in the pathophysiology of many of the symptoms of septic shock (eg, rigors, fevers, vasodilation). Polysaccharide capsule (Choice D) is not directly cytotoxic to macrophages, although it does play an important role in the interaction of MTB with phagocytes. There are four primary laye

62 Exam Section 2: Item 13 of 50 National Board of Medical Examiners' Comprehensive Basic Science Self-Assessment 13. A study is conducted to assess body mass index (BMI) in a group of 100 patients with type 2 diabetes mellitus. Results show a mean (± standard error of the mean) BMI of 31 kg/m2 (+ 4 kg/m2). The 99% confidence intervals for this measurement are 20.7 to 41.3. The 95% confidence intervals are 23.3 to 38.8. Based on this information, which of the following best represents the number of individuals who have a BMI between 38.8 kg/m2 and 41.3 kg/m2? A) 2 B) 4 C) 6 D) 8 E) 10

A. In a gaussian (normal) distribution without skew, data points center around the mean value. Standard deviation is a measure of the degree of dispersion of data points about the mean and is defined as the square root of the variance. In a gaussian distribution, one standard deviation above and below the mean will include approximately 68% of data points, while two standard deviations above and below the mean will include approximately 95% of data points. Three standard deviations above and below the mean will include approximately 99.7% of data points. In a population with a mean BMI of 31 kg/m2 and a standard deviation of 4 kg/m2, individuals with a BMI less than 23.3 kg/m2 therefore fall more than two standard deviations below the mean. Of the 5% of data points found outside of the second standard deviation (95% lie within two standard deviations, therefore 5% lie beyond two standard deviations), half of these (2.5%) are found below the second standard deviation from the mean (less than 23.3 kg/m2) and half (2.5%) are found above the second standard deviation from the mean (greater than 38.8 kg/m2). The number of individuals who fall between two standard deviations higher than the mean and three standard deviations higher than the mean will be half of 4.7% (99.7% minus 95%), or roughly 2%. In a population of 100 individuals, this represents two individuals. Incorrect answers: B, C, D, and E. Four individuals (Choice B), or 4% of the population corresponds approximately to the amount of data points found in total outside the range of two standard deviations but within three standard deviations. This includes those individuals who fall both above and below the mean. Six (Choice C), eight (Choice D), and 10 (Choice E) do not correspond with a number of individuals outside the range of one, two, or three standard devia

42 Exam Section 1: Item 43 of 50 National Board of Medical Examiners' Comprehensive Basic Science Self-Assessment 43. A 4-day-old male newborn who was born at home is brought to the emergency department because of respiratory distress and cyanosis. The mother reports that she found him in his crib not breathing. He began to breathe again after she picked him up. Examination shows a narrow thorax. His ears have periauricular skin tags. He also has micrognathia, glossoptosis, a mandibular cleft, and a short palate. Tracheostomy relieves his respiratory distress. Defects such as these are consistent with altered development of which of the following pharyngeal arches? A) First B) Second C) Third D) Fourth E) Sixth

A. Pierre Robin syndrome is a condition present at birth and is characterized by a small lower jaw (micrognathia), downward displacement or retraction of the tongue (glossoptosis), and cleft palate. The downward displacement of the tongue can lead to airway obstruction, hypoxia, difficulty feeding, and respiratory distress. The syndrome is caused by a defect in the development of the first pharyngeal arch, which forms the maxilla, mandible, masseter, pterygoids, and mylohyoid, along with cranial nerves V2 and V3. In Pierre Robin syndrome, abnormalities resulting in hypoplasia of the mandible and micrognathia lead to the posterior displacement of the tongue. Incorrect Answers: B, C, D, and E. The second pharyngeal arch (Choice B) contributes to the development of the middle ear, hyoid bone, temporal styloid process, and muscles including the facial muscles, stapedius, platysma, and stylohyoid. It gives rise to cranial nerve VII. The third pharyngeal arch (Choice C) is involved in the development of the stylopharyngeus muscle and the glossopharyngeal nerve (cranial nerve IX). The fourth pharyngeal arch (Choice D) contributes to the development of the cricothyroid muscle and muscles of the soft palate, as well as the thyroid cartilage, and it is innervated by the superior laryngeal branch of the vagus nerve. The sixth pharyngeal arch (Choice E) contributes to the development of all intrinsic muscles of the larynx except the cricothyroid muscle, the cricoid, arytenoid, corniculate, and cuneiform cartilages, and it is innervated by the recurrent laryngeal branch of the vagus nerve. Educational Objective: In Pierre Robin syndrome, abnormalities in the development of the first pharyngeal arch result in hypoplasia of the mandible and micrognathia leading to posterior displacement of the tongue. The downward displacement of the

52 Exam Section 2: Item 3 of 50 National Board of Medical Examiners' Comprehensive Basic Science Self-Assessment 3. A 48-year-old man comes to the emergency department because of a 3-day history of fever, chills, headache, and nonproductive cough. He underwent renal transplantation 5 years ago. Current medications include corticosteroids. His temperature is 40°C (104°F). Coarse rhonchi are heard over the right lower lobe on auscultation. A chest x-ray shows right lobar pneumonia with small pleural effusions. The patient is treated with an antibiotic that inhibits DNA gyrase. This drug is most likely which of the following? A) Ciprofloxacin B) Metronidazole C) Rifampin D) Tetracycline E) Vancomycin

A. Pneumonia commonly presents with cough, shortness of breath, pleuritic chest pain, fever, and abnormal breath sounds. It often follows an upper respiratory infection or aspiration of oropharyngeal microbes and is more common in immunocompromised persons. Corticosteroid use is a risk factor for infection with atypical organisms, such as Legionella pneumophila, which is a gram-negative rod that is transmitted primarily through aerosols from water sources (eg, air conditioning systems). The atypical pneumonia may appear as diffuse, patchy alveolar and interstitial opacities on lung imaging or as a lobar consolidation. It can also be associated with hyponatremia. First-line therapy for Legionella pneumophila includes administration of either macrolides or fluoroquinolones. Ciprofloxacin is a fluoroquinolone antibiotic that was likely prescribed for this patient. Fluoroquinolones impair DNA synthesis by the direct inhibition of DNA gyrase (prokaryotic topoisomerase II). Resistance to fluoroquinolones is frequently encountered and mediated by structural variants of DNA gyrase and production of membrane efflux pumps. Adverse effects of fluoroquinolones include gastrointestinal distress, tendonitis with risk for tendon rupture, and QT prolongation. Incorrect Answers: B, C, D, and E. Metronidazole (Choice B) is an antibacterial and antiprotozoal agent that diffuses into target organisms and alters microbial protein function, resulting in the generation of free radicals and subsequent DNA damage. It is particularly effective against anaerobes, especially infections involving the abdomen or pelvis, as well as the facultative anaerobes Helicobacter pylori and Gardnerella vaginalis. These organisms are not common causes of pneumonia. Rifampin (Choice C) is a rifamycin antibiotic that inhibits DNA-dependent RNA polymerase. Resist

76 Exam Section 2: Item 27 of 50 National Board of Medical Examiners' Comprehensive Basic Science Self-Assessment 27. A 10-year-old girl is brought to the physician by her parents 30 minutes after she sustained a red ant bite to her right foot. The parents report that within 15 minutes, redness and swelling occurred at the site of the bite. Physical examination shows edema and erythema at the wound site. The action of which of the following chemical mediators on postcapillary venules is the most likely cause of these findings? OA) Bradykinin В) СЗЬ C) Interleukin-1 D) Phospholipase C E) Thromboxane A2 F) Tumor necrosis factor

A. The body responds to harmful stimuli such as infectious agents, mechanical disruption, and chemical irritants with a localized inflammatory response, which is mediated by the innate immune system. The cardinal signs of inflammation are rubor (redness or erythema), calor (warmth), tumor (swelling or edema), dolor (pain), and functio laesa (loss of function). Erythema, warmth, and edema are caused by localized vasodilation of the postcapillary venules, resulting in increased blood flow to the affected area, which occurs within minutes of an insult. The primary inflammatory mediators of vasodilation are histamine, prostaglandin-E2, and bradykinin. Prostaglandin-E2 and bradykinin are also involved in sensitizing nociceptive nerve endings resulting in the increased pain of inflammation. Incorrect Answers: B, C, D, E, and F. C3b (Choice B) is a core component of the complement cascade and is formed by the cleavage of C3. The C3b fragment may then participate in one of three of pathways. It may bind with factor B to form C3 convertase and amplify C3 cleavage in a positive feedback loop. It may bind with C4b and C2b fragments to form C5 convertase. Or, it may opsonize the microbe by binding directly to the pathogen surface. It is not a vasoactive mediator. Interleukin-1 (Choice C) is a proinflammatory cytokine secreted by macrophages, endothelial cells, dendritic cells, and B lymphocytes. It has numerous effects, including the promotion of fever, vasodilation, and stimulation of endothelial cells to express adhesion proteins for leukocyte recruitment. Interleukin-1 is produced in the inflammatory response after the recruitment and migration of initial innate immune cells. Phospholipase C (Choice D) enzymatically cleaves the lipid molecule phosphatidylinositol 4,5-bisphosphate (PIP2) into inositol triphosphate (IP3) and diac

43 Exam Section 1: Item 44 of 50 National Board of Medical Examiners' Comprehensive Basic Science Self-Assessment 44. A 48-year-old woman comes to the physician because of increasingly severe right flank pain during the past 3 weeks. Pelvic examination shows a normal-appearing vagina and cervix. A mass is palpated in the right pelvis during bimanual examination. Ultrasonography of the abdomen shows a mass associated with the right ovary. An MRI of the abdomen is suggestive of ovarian cancer. Which of the following is the location of the lymph nodes to which the ovaries first drain? A) Along the ovarian vessels B) Immediately lateral to the rectum C) Inside the broad ligament D) On the anterior surface of the sacrum E) On the fundus of the uterus

A. The para-aortic lymph nodes are a component of the lymphatic system, a network of vessels which follow a predictable pattern of drainage to lymph node beds. Lymph is generated by hydrostatic pressure in the tissues causing fluid to leak out of vascular structures and into the interstitium. It is then collected by the lymphatics along with lymphocytes and any malignant cells exiting the tissues. The para-aortic lymph nodes drain the ovaries, testes, uterus, and kidneys and would be the most likely first recipient of lymph from the ovaries. Lymphatic vessels from the ovaries are carried in the suspensory, or infundibulopelvic, ligament which extends laterally from the ovary to the wall of the pelvis and runs adjacent to the ovarian artery and vein. Thus, the lymphatics draining the ovaries run along the ovarian vessels, which is a common pairing in many anatomical locations of the body. After reaching the para-aortic lymph nodes, the lymph will be channeled through the cisterna chyli and thoracic duct, and ultimately drain into the left subclavian vein. This is the same final pathway for lymph from either lower extremity, the pelvis, or the left upper extremity. În contrast, the lymphatic network of the right side of the body above the diaphragm is drained by the right lymphatic duct, which enters the right subclavian vein. Incorrect Answers: B, C, D, and E. The pelvic lymph nodes include the pararectal lymph nodes, which are immediately lateral to the rectum (Choice B), and the sacral lymph nodes, which are on the anterior surface of the sacrum (Choice D). It is unlikely that the sentinel node of an ovarian malignancy will be located in either of these lymph node beds. The lymphatic vessels draining lymph from the ovaries are carried within the suspensory ligament, along with the ovarian vessels, toward the para-aor

17 Exam Section 1: Item 17 of 50 National Board of Medical Examiners' Comprehensive Basic Science Self-Assessment 17. A 40-year-old woman who is a construction worker comes to the physician because of pain in her shoulder that developed after she lifted a heavy beam. Physical examination shows weakness and pain only during external rotation of the arm at the shoulder. There is muscle wasting inferior to the spine of the scapula. Which of the following muscles is most likely injured? A) Infraspinatus B) Latissimus dorsi C) Subscapularis D) Supraspinatus E) Teres major

A. The rotator cuff is the group of muscles and tendons that act to seat the humeral head directly into the center of the glenoid during motion of the shoulder. These muscles also act, in concert with other muscles about the shoulder, to internally and externally rotate, abduct, and adduct the shoulder. These muscles include the subscapularis, teres minor, supraspinatus, and infraspinatus. The prime movements of these muscles at the glenohumeral joint include internal rotation (subscapularis), external rotation when in an adducted position (teres minor), external rotation (infraspinatus), and abduction (supraspinatus). The coupling and antagonistic action of these muscles seats the humeral head in the glenoid and provides a stable pivot upon which the larger, more powerful muscles of the shoulder can act, such as the latissimus dorsi, deltoid, and pectoralis major. A rotator cuff tear typically occurs at the insertion of the tendons of the rotator cuff on the humeral head. Commonly, this occurs in the supraspinatus tendon; however, tears can occur in any of the rotator cuff tendons or at the myotendinous junction. Risk factors for rotator cuff tears include heavy labor, repetitive overhead lifting, diabetes mellitus, and vascular disease. Chronic disease leading to tendon degeneration as well as acute or repetitive microtrauma are thought to contribute to eventual tendon failure. The classic presentation of a rotator cuff tear is with pain and weakness in one of the axes of motion corresponding with the associated rotator cuff muscle. Late manifestations of a tear include clinically appreciable atrophy of the corresponding muscle. In this case, the patient's weakness in external rotation and loss of muscle bulk inferior to the scapular spine are consistent with an injury to the infraspinatus. Incorrect Answers: B, C, D

87 Exam Section 2: Item 38 of 50 National Board of Medical Examiners' Comprehensive Basic Science Self-Assessment 38. A7-year-old boy is brought to the physician by his mother because of a 3-hour history of pain and stiffness of his neck. Two days ago, he fell off his bike and hurt his neck. Physical examination shows decreased range of motion of the neck. Neurologic examination shows no abnormalities. AP and lateral x-rays of the cervical spine show congenital fusion of the atlas to the occipital bone associated with C2-3 vertebral fusion. Flexion and extension of the neck are most likely placing additional strain on which of the following structures in this patient because of his congenital abnormalities? A) Atlantoaxial joint B) Interventricular foramen of Monroe C) Sternocleidomastoid muscles D) Vertebral artery

A. The spinal column is comprised of bony vertebrae, intervertebral discs made of fibrocartilage and gel-like material (nucleus pulposus), and ligaments that maintain the structure of the column. Function of the spinal column is based upon having numerous mobile segments; the facet joints and intervertebral discs between each vertebra provide for a small amount of motion in multiple directions. The summation of small motions allows for the composite range of motion of the spine, including flexion, extension, medial and lateral deviation, rotation and torsion, and combinations of these together. When segments are fused together from a congenital disorder, ankylosing spondylitis, or from surgery, no motion can occur at these segments. Because of this, the adjacent segments (facets, intervertebral discs, ligaments) will experience increased strain. This may lead to accelerated degeneration of the spinal column at such locations. The patient in this case demonstrates fusion of C1 to the occipital bone and fusion of C2 with C3. The segment with intact motion between these areas of fusion that will endure increased strain will be the C1-C2 joint, also known as the atlantoaxial joint. Incorrect Answers: B, C, and D. The interventricular foramen of Monro (Choice B) connects the lateral ventricles with the third ventricle of the brain. These structures do not see increased strain or pressure from fusion of spinal segments. Increased intracranial pressure (eg, secondary to hydrocephalus) would cause increased pressure in these regions. The sternocleidomastoid muscles (Choice C) originate at the manubrium and medial portions of the clavicle and insert at the mastoid process of the temporal bone. Spasms in these muscles can cause torticollis. These muscles do not undergo increased stress with the fusion of spinal segments. The ver

37 Exam Section 1: Item 38 of 50 National Board of Medical Examiners' Comprehensive Basic Science Self-Assessment 38. A 66-year-old right-handed man has a thrombotic stroke. A CT scan of the head is shown. Which of the following is the most likely neurologic finding in this patient? A) Global aphasia B) Left hemiparesis OC) Spatial neglect D) Weakness of all muscles of facial expression on the right E) Weakness of the lower two thirds of the face on the left

A. This axial, noncontrast CT image shows hypodensity (darkness) of most of the left hemisphere (sparing the anterior and posterior cerebrum) and is consistent with an ischemic infarction involving the territory of the middle cerebral artery (MCA). The MCA supplies large portions of the lateral frontal, parietal, and temporal lobes. Occlusion of the MCA in the dominant hemisphere, which is frequently the left hemisphere, can lead to global aphasia caused by involvement of the Broca (inferior frontal lobe) and Wernicke (superoposterior temporal lobe) areas, along with the arcuate fasciculus, the bundle of axons that connects these two areas. Occlusion of the left MCA also leads to right-sided paralysis and sensory loss of the face and upper limb (caused by involvement of the superior and lateral precentral and postcentral gyri). Cerebrovascular accidents (CVAS, also known as strokes) occur as a result of ischemic or hemorrhagic loss of blood supply to the brain. Approximately 80%-85% of CVAS are ischemic, commonly arising from thromboembolic disease (eg, MCA occlusion from a thrombus), whereas 15%-20% of CVAS are hemorrhagic as a result of blood vessel rupture (eg, hypertension-related intraparenchymal hemorrhage from a perforating artery). Risk factors for CVAS include smoking, hypertension, diabetes, carotid or intracranial atherosclerotic disease, history of hypercoagulability, atrial fibrillation, and advanced age. Classically, CVAS manifest as a neurologic deficit related to loss of function of the affected part of the brain. Incorrect Answers: B, C, D, and E. Left-sided hemiparesis (Choice B) would be caused by right-sided lesions of the lateral corticospinal tract. Occlusion of the right MCA could lead to hemineglect, paralysis, and sensory deficits of the left-sided face and arm, and lacunar stroke affecting the

8 Exam Section 1: Item 8 of 50 National Board of Medical Examiners' Comprehensive Basic Science Self-Assessment 8. A 60-year-old woman comes to the physician because of vaginal bleeding for 6 weeks. Menarche occurred at the age of 12 years, and menses had occurred at regular 30-day intervals. She had three term pregnancies by the age of 25 years. She has been receiving conjugated equine estrogen therapy since menopause 10 years ago. She has smoked 1 pack of cigarettes daily for 40 years. She drinks alcoholic beverages five times weekly. Physical examination and laboratory studies show no abnormalities. An endometrial biopsy specimen shows endometrial adenocarcinoma. Which of the following is the strongest predisposing risk factor for the biopsy findings? A) Alcohol use B) Cigarette smoking C) Early menarche D) Estrogen therapy E) Multiparity

D. Vaginal bleeding in a postmenopausal woman should raise suspicion for malignancy, including uterine or cervical carcinoma. Estrogen excess without adequate opposition by progestin is a strong risk factor for atypical endometrial hyperplasia and endometrial carcinoma, as this malignancy is estrogen-sensitive. This is one reason that estrogen therapy without progestin is no longer generally recommended for long-term treatment of menopausal symptoms or prevention of osteoporosis. Additionally, estrogen therapy also increases the risk for thromboembolic events in postmenopausal women. Increased exposure to estrogen can come from an exogenous supplement as in this case, or an endogenous source, such as increased peripheral production of estrogen by adipose tissue or from an increased number of lifetime menstrual cycles. Thus, risk factors for endometrial carcinoma include estrogen supplementation, obesity, early menarche or late menopause, nulliparity, advanced age, and family history. Endometrial biopsy is used to confirm the diagnosis. With biopsy and imaging, the stage of endometrial carcinoma is determined and follows TNM (tumor, nodes, metastases) staging. If the disease is confined to the uterus, surgical excision is the first-line therapy. This is generally achieved through hysterectomy with bilateral salpingo-oophorectomy. Incorrect Answers: A, B, C, and E. While alcohol use (Choice A) and cigarette smoking (Choice B) are risk factors for cancers such as hepatocellular carcinoma and squamous cell carcinoma, respectively, they have not been shown to increase the risk for endometrial carcinoma. Early menarche (Choice C) is a risk factor for endometrial carcinoma as it increases the total number of ovulatory cycles and thus endometrial estrogen exposure over a lifetime. However, this patient's use of estrogen supple

12 Exam Section 1: Item 12 of 50 National Board of Medical Examiners' Comprehensive Basic Science Self-Assessment 12. A73-year-old woman has a 3-month history of increasing fatigue, weakness, and a 9.0-kg (20-lb) weight loss. Examination shows hepatosplenomegaly. An M-protein spike is observed on serum electrophoresis. Which of the following is the most likely diagnosis? A) Acute myelogenous leukemia B) B-lymphocyte neoplasia C) Chronic myelogenous leukemia D) Leukemoid reaction OE) T-lymphocyte neoplasia

B. B-lymphocyte neoplasia likely accounts for this patient's presentation with an M-protein spike (M-spike). While an M-spike may occur in a variety of B cell malignancies including Waldenstrom macroglobulinemia, plasma cell leukemia, smoldering myeloma, amyloidosis, or plasmacytoma, it is most commonly recognized in the setting of multiple myeloma (MM). Other signs and symptoms of MM include bone pain, anemia, hypercalcemia, renal dysfunction, and lytic bone lesions. Hepatosplenomegaly is an occasional finding. B cells are precursors to plasma cells, which secrete specific immunoglobulins of different classes. MM is a malignancy caused by the neoplastic proliferation of a single plasma cell clone, which overproduces monoclonal immunoglobulin and light or heavy chains. These clonal immunoglobulins are secreted in high numbers and appear as a monoclonal spike on protein electrophoresis in the gamma region. Diagnosis of multiple myeloma is suspected based on the presence of an M-protein spike in the presence of concerning symptoms, but is confirmed by bone marrow biopsy, which must demonstrate at least 10% clonal plasma cells. As a result of the hypersecretion of a single immunoglobulin, these patients often have a relative immunodeficiency as the production of other immunoglobulins is impaired. Incorrect Answers: A, C, D, and E. Acute myelogenous leukemia (Choice A) involves abnormal cells of the myeloid lineage, not the lymphocyte lineage, with a variety of defining mutations, each of which portends its own prognosis. Secretion of immunoglobulins is not a feature of this disease; therefore, an M-spike would not be expected on protein electrophoresis. Chronic myelogenous leukemia (Choice C) is defined by the presence of the Philadelphia (Ph) chromosome, which is created by a translocation between chromosomes 9 and 22, r

83 Exam Section 2: Item 34 of 50 National Board of Medical Examiners' Comprehensive Basic Science Self-Assessment 34. During a clinical study, a researcher finds that as children approach puberty, the consumption of milk in the diet is partially replaced by the consumption of soft drinks. This results in a decrease in calcium intake. This dietary change is most likely to have which of the following effects on bone mineralization or metabolism? A) Decreased parathyroid hormone secretion because of increased phosphorus intake B) Decreased peak bone accretion during adolescence C) Hypophosphatasia D) Rickets resulting from calcium deficiency E) Rickets resulting from vitamin D deficiency

B. Bone tissue is constantly remodeling throughout human life. During childhood and adolescence, bone mineral density increases until the early third decade of life in females and the late third decade in males. According to Wolff's law, bone density will increase in response to stress applied. In turn, weightbearing exercise or physically strenuous work will increase bone mineral density. In addition, availability of calcium and vitamin D are required for increased bone mineral density. Estrogen and testosterone are key regulators of ossification and bone mineral density as well; because of this, bone mineral density significantly increases during puberty. Deficits in any of these factors during the first three decades of life will lead to decreased peak bone mineral density. After peak bone mineral density is reached, increasing bone mineralization seldom occurs. The normal aging process biases bone remodeling toward greater osteoclast activity and less osteoblast activity, which leads to progressive losses of bone mineral density over time. In this study, decreased calcium intake will lead to decreased mineralization of bone, leading to a decreased peak bone accretion during adolescence. Incorrect Answers: A, C, D, and E. Parathyroid hormone increases the secretion of phosphorus at the kidney. Increased phosphorus intake, reflected as serum hyperphosphatemia, leads to increased (not decreased) concentrations of parathyroid hormone secretion through normal feedback mechanisms (Choice A). Hypophosphatasia (Choice C) is a rare inherited genetic disorder caused by mutations in the ALPL gene, which codes for alkaline phosphatase. Deficiency of alkaline phosphatase leads to the accumulation of pyrophosphate and decreased serum concentrations of phosphate ion, which results in impaired bone mineralization. Rickets resultin

97 Exam Section 2: Item 48 of 50 National Board of Medical Examiners' Comprehensive Basic Science Self-Assessment 48. A 37-year-old woman is brought to the emergency department after her husband found her unconscious. Her temperature is 36°C (96.8°F), pulse is 128/min, and blood pressure is 70/40 mm Hg. Physical examination shows cool, pale extremities, jugular venous distention, faint peripheral pulses, and crackles over the bottom two thirds of both lung fields. Heart sounds are normal, and there are no murmurs. She withdraws to painful stimuli in all four extremities. This patient is most likely experiencing which of the following types of shock? A) Anaphylactic B) Cardiogenic C) Hypovolemic D) Neurogenic E) Septic

B. Cardiogenic shock can occur in the setting of acute myocardial infarction, severe valvular dysfunction, severe heart failure, heart block, or arrhythmia. Patients may experience cardiogenic pulmonary edema resulting in shortness of breath, tachypnea, hypoxia, and audible rales on examination. Physical examination demonstrates hypotension, cool and pale extremities, and jugular venous distention. Pulmonary artery catheterization typically demonstrates decreased cardiac index with increased pulmonary capillary wedge pressure if there is left-sided heart dysfunction and increased systemic vascular resistance (SVR). In cardiogenic shock, dysfunction of cardiac myocytes results in decreased chronotropy, lusitropy, or inotropy. Decreased cardiac output and increased pulmonary capillary wedge pressure result from such dysfunction. Systemic vascular resistance is increased as a physiologic attempt to maintain mean arterial pressure in the setting of decreased cardiac output and peripheral perfusion. Incorrect Answers: A, C, D, and E. Anaphylactic (Choice A) and septic (Choice E) shock are forms of distributive shock, which are characterized by a primary decrease in SVR. This decrease in SVR is secondary to IgE-mediated histamine and cytokine release in anaphylactic shock, and cytokine or toxin-mediated endothelial dysfunction in septic shock. Cardiac output increases to compensate for low SVR in the early stages of distributive shock. If the inciting state is not appropriately treated, further decompensation can occur resulting in progressive hypotension and insufficient perfusion with end-organ damage. Pulmonary artery catheterization would demonstrate an increased cardiac index and decreased SVR in cases of distributive shock. Hypovolemic (Choice C) shock occurs secondary to a significant decrease in intravascular volume,

86 Exam Section 2: Item 37 of 50 National Board of Medical Examiners' Comprehensive Basic Science Self-Assessment 37. A 67-year-old man comes to the physician because of a 2-month history of unexplained weight loss. He has a 50-year history of type 1 diabetes mellitus. He underwent gallstone removal 12 years ago. He has smoked 1 pack of cigarettes daily for 45 years. He is 188 cm (6 ft 2 in) tall and weighs 120 kg (265 lb); BMI is 34 kg/m2. Physical examination shows no other abnormalities. Serum studies show a calcium concentration of 11 mg/dL. An abdominal CT scan shows a pancreatic mass. A biopsy specimen of the mass shows pancreatic adenocarcinoma. Which of the following is the strongest predisposing risk factor for pancreatic cancer in this patient? A) BMI of 34 kg/m2 B) Cigarette smoking C) History of gallstone disease O D) Hypercalcemia E) Type 1 diabetes mellitus

B. Cigarette smoking is the most important preventable risk factor for pancreatic cancer. Increased risk for pancreatic cancer is observed in smokers, especially heavy smokers. This excess risk slowly declines after smoking cessation. Other important risk factors for pancreatic cancer include obesity and type 2 diabetes mellitus. In addition to pancreatic cancer, cigarette smoking is an important, modifiable risk factor for numerous other cancers, including those affecting the larynx, esophagus, lung, kidney, bladder, and cervix. A hypothesized mechanism of carcinogenesis from smoking involves the promotion of malignancy through DNA mutation secondary to inhaled polycyclic aromatic hydrocarbons. Incorrect Answers: A, C, D, and E. BMI of 34 kg/m2 (Choice A) is suggestive of obesity. While obesity is associated with an increased risk for pancreatic cancer, smoking is a stronger and well-established risk factor. History of gallstone disease (Choice C) may increase the patient's risk for acute pancreatitis. A prior history of gallstone disease has not, however, been established to contribute to pancreatic cancer. Hypercalcemia (Choice D) is a rare cause of acute pancreatitis. It has not been established to contribute to pancreatic cancer. Type 1 diabetes mellitus (Choice E) has not been conclusively demonstrated to contribute to pancreatic cancer. While there is evidence of an association between type 2 diabetes mellitus and pancreatic cancer, this relationship has not been consistently demonstrated for patients with type 1 diabetes mellitus. Educational Objective: Cigarette smoking is the most important preventable risk factor for pancreatic cancer. The excessive risk for pancreatic cancer caused by cigarette smoking decreases slowly after smoking cessation. Previous Next Score Report Lab Values Calculator Help Pause

50 Exam Section 2: Item 1 of 50 National Board of Medical Examiners' Comprehensive Basic Science Self-Assessment 1. A 52-year-old man comes to the physician because of inability to achieve an erection 6 weeks after undergoing operative removal of a tumor on the distal portion of the rectum. During the excision, the tumor was found to be extensively attached to the posterior aspect of the prostate. Damage to which of the following nerves is the most likely cause of this patient's symptoms? A) Gray rami from the sympathetic chain B) Pelvic splanchnics C) Pudendal D) Superior hypogastric E) Ventral rami of S2-4

B. Damage to the pelvic splanchnic nerves is the most likely cause of this patient's erectile dysfunction. The pelvic splanchnic nerves are efferent parasympathetic preganglionic nerves that arise from the intermediolateral column of spinal cord levels S2 to S4. These nerves innervate the smooth muscle of the distal anterior rectum, located immediately posterior to the prostate, explaining this patient's pelvic splanchnic nerve damage. The pelvic splanchnic nerves then synapse with the postganglionic parasympathetic cavernous nerve in the inferior hypogastric plexus. The cavernous nerve travels to the corpora cavernosa of the penis and releases acetylcholine to promote vasodilation of the helicine arteries, resulting in penile erection. Damage to the pelvic splanchnic nerves results in impaired parasympathetic innervation to the erectile tissue of the penis, which manifests as erectile dysfunction. Incorrect Answers: A, C, D, and E. The gray rami from the sympathetic chain (Choice A) are located immediately dorsal to the paravertebral sympathetic chain. The gray rami are located too posteriorly to be affected by this patient's tumor excision. As well, sympathetic innervation plays a role in detumescence. The pudendal nerve (Choice C) carries sensory fibers that originate from the penile skin, glans, and urethra as well as motor neurons innervating the ischiocavernosus and bulbocavernosus muscles, leading to the contraction of these muscles and thereby mediating the sensorimotor aspect of erections. Though damage to the pudendal nerve would lead to erectile dysfunction, the pudendal nerve is located near the posterior aspect of the rectum and is therefore less likely to be damaged by surgical manipulation of the periprostatic region. The superior hypogastric nerves (Choice D) are located just anterior to the bifurcation

46 Exam Section 1: Item 47 of 50 National Board of Medical Examiners' Comprehensive Basic Science Self-Assessment 47. Agroup of men between the ages of 25 and 30 years and a group of men between the ages of 75 and 80 years were deprived of water for 24 hours. Renal function measurements for both groups are shown: 25-30 Years Urine specific gravity Urine osmolality (mOsmol/kg) 1.030 1100 75-80 Years 1.020 850 Which of the following changes associated with aging is the most likely cause of the difference in urine concentrating ability between the two groups? A) Decreased renal blood flow B) Decreased renal tubule responsiveness to ADH (vasopressin) OC) Decreased single nephron filtration rate D) Increased release of atrial natriuretic peptide E) Increased sodium transport in the thick ascending loop of Henle

B. Decreased renal tubule responsiveness to ADH (vasopressin) most likely explains the disparities in urine concentrating abilities between younger and older men. ADH is a peptide hormone made in the hypothalamus and released from the posterior pituitary in response to decreased effective circulating volume or states of hyperosmolarity. It acts on vascular smooth muscle via the V, receptor to cause vasoconstriction, thereby increasing mean blood pressure. It also acts on the principal cells of the renal collecting duct to increase the expression of aquaporin channels on the luminal surface of these cells. Increased expression of aquaporin channels leads to a greater ability to resorb water from the lumen of the collecting duct to regulate serum osmolality. This free water resorption consequently increases the urine osmolality and specific gravity. In the setting of water deprivation, serum osmolality rises, which signals the hypothalamic osmoreceptors to secrete increased amounts of ADH. This subsequently results in increased free water absorption from the collecting tubule to maintain homeostatic serum osmolality. On the basis of the data provided, this ability to concentrate urine through the action of ADH during water deprivation appears to wane with age. ADH can also be secreted in response to myriad other stimuli including pain and nausea and can cause an inappropriate resorption of water leading to a condition known as the syndrome of inappropriate ADH release. Incorrect Answers: A, C, D, and E. Decreased renal blood flow (Choice A) occurs in states of hypovolemia or hypotension, each of which have many potential causes. This stimulates the release of renin from the juxtaglomerular apparatus of the kidney, which is necessary for the formation of angiotensin I from angiotensinogen. This is converted to angiotensin

30 Exam Section 1: Item 31 of 50 National Board of Medical Examiners' Comprehensive Basic Science Self-Assessment 31. An investigator is testing the hypothesis that advanced glycosylation end products (AGES) lead to an increased synthesis of extracellular matrix by human renal mesangial cells. The cells are plated on two media: type I collagen treated extensively with glucose 6-phosphate to produce AGES (first medium), and a control medium of untreated type I collagen. To confirm this hypothesis, MRNA that encodes for which of the following is most likely to be increased in cells incubated in the first medium compared with cells in the control medium? A) Endothelial nitric oxide synthase B) Fibronectin C) NADPH oxidase D) Nuclear factor-KB E) Vascular endothelial growth factor

B. Fibronectin MRNA would most likely be increased in renal mesangial cells incubated in a medium containing glucose-6-phosphate, as fibronectin is a critical component of the extracellular matrix (ECM). Soluble fibronectin is secreted from cells-in this case, renal mesangial cells-into the extracellular space where it binds to cell-surface receptors such as a5B1 integrin. Binding is thought to introduce conformational changes in the fibronectin strand leading to the exposure of previously hidden binding sites. Additional fibronectin molecules bind at these sites and allow the previously soluble fibronectin molecule to become an insoluble matrix of multiple branching fibronectin units. As fibronectin is a protein that must be translated from MRNA, cells that secrete more fibronectin would be expected to have higher levels of MRNA encoding fibronectin. Incorrect Answers: A, C, D, and E. Endothelial nitric oxide synthase (Choice A) is an enzyme that synthesizes nitric oxide from L-arginine and oxygen. It is a vasoactive compound that results in the relaxation of vascular smooth muscle. It is not implicated in the formation of the ECM. NADPH oxidase (Choice C) is an enzyme important in the synthesis of hypochlorous acid and reactive oxygen species in the respiratory burst of phagocytes. It plays a role in phagocyte- mediated killing of pathogens but not in the formation of the ECM. Deficiency of NADPH oxidase in phagocytes is characteristic of chronic granulomatous disease, an immunodeficiency that results in recurrent infections with catalase-positive organisms. Nuclear factor-KB (Choice D) is a transcription factor that has broad functionality but primarily serves to increase expression of genes implicated in immune response, lymph tissue development, B-cell development, and stress response. It is activated by tumor nec

95 Exam Section 2: Item 46 of 50 National Board of Medical Examiners' Comprehensive Basic Science Self-Assessment 46. A 64-year-old homeless man with alcoholism is brought to the emergency department by police after he was found wandering aimlessly in a city park. He has weakness, fatigue, and hunger. He appears disheveled and cachectic. His temperature is 38.2°C (100.8°F), pulse is 110/min, respirations are 20/min, and blood pressure is 134/82 mm Hg. Physical examination shows mild scleral icterus, numerous skin abrasions, and 1+ edema of the lower extremities. He is oriented to person but not to place or time. Laboratory studies show: 6.8 g/dL 19% 111 um3 3500/mm3 Hemoglobin Hematocrit Mean corpuscular volume Leukocyte count Platelet count 95,000/mm3 A photomicrograph of a peripheral blood smear is shown. A deficiency of which of the following best accounts for the hematologic findings in this patient? A) Copper

B. Folic acid deficiency most likely accounts for this patient's megaloblastic anemia. Megaloblastic anemia occurs in the setting of impaired DNA synthesis, commonly related to folate or vitamin B12 (cobalamin) deficiency, and is characterized by erythrocyte macrocytosis and hypersegmented neutrophils. Laboratory evaluation shows anemia, increased mean corpuscular volume, and normal white cell and platelet indices. The presence of hypersegmented neutrophils on peripheral smear is a characteristic finding in megaloblastic anemia. Since folate and vitamin B 12 are needed for the conversion of homocysteine to methionine, deficiency is associated with increased homocysteine concentrations. However, vitamin B12 also acts as a cofactor for the conversion of methylmalonyl-COA to succinyl-CoA, and its deficiency is associated with both high methylmalonic acid and homocysteine concentrations. Common risk factors include chronic alcohol abuse, malnutrition, and high cell turnover conditions such as pregnancy and sickle cell disease. Deficiency of folate and/or B12 can be seen in patients with chronic alcohol use disorder, although folate deficiency is more common as the body maintains larger stores of vitamin B 12, thus depletion takes much longer. Incorrect Answers: A, C, D, and E. Copper deficiency (Choice A) results in sideroblastic anemia and is often seen in the setting of malabsorptive syndromes. Neutropenia is also a common finding. Iron deficiency (Choice C) anemia results in microcytosis, not megaloblastic anemia. It is common in menstruating women, those with diets low in iron, and in patients with chronic blood loss such as in colorectal carcinoma. Other cell lines are not typically affected, and treatment is with oral iron supplementation. Vitamin B, (thiamine) deficiency (Choice D) results in wet or dry beriberi. Dr

38 Exam Section 1: Item 39 of 50 National Board of Medical Examiners' Comprehensive Basic Science Self-Assessment 39. A 33-year-old African American woman comes to the physician because of dry eyes for 3 weeks. Physical examination shows decreased lacrimation and uveitis. There is mild hepatosplenomegaly. A chest x-ray shows prominent bilateral hilar adenopathy. Microscopic examination of a hilar lymph node is most likely to show which of the following pathologic conditions? A) Abscess formation B) Granuloma formation C) Hodgkin disease D) Langerhans histiocytosis E) Metastatic melanoma F) Non-Hodgkin lymphoma G) Plasmacytoma H) Sinus histiocytosis

B. Sarcoidosis is a noncaseating granulomatous disease that often involves the lung, and classically presents with shortness of breath and cough in a younger, of characterized by the presence of noncaseating granuloma formation on histology. The clinical presentation ranges from asymptomatic, to mild shortness of breath and cough, to diffuse granulomatous infiltration of multiple organ systems. Manifestations include uveitis, lymphadenopathy, hepatosplenomegaly, hypercalcemia, and erythema nodosum. On diagnostic imaging, patients often present with bilateral hilar lymphadenopathy, with or without bilateral upper lobe predominant interstitial infiltrates depending on the stage of the disease. The diagnosis is presumed on the basis of clinical history and imaging features, although tissue sampling, most commonly from involved lymph nodes, is required to definitively diagnose a patient. Treatment typically requires steroids. Untreated chronic disease can progress to pulmonary fibrosis and restrictive lung physiology. African American female. It is Incorrect Answers: A, C, D, E, F, G, and H. Abscess formation (Choice A) occurs in the setting of an acute inflammatory response to an infectious organism, resulting in an organized collection of pus, necrotic cellular debris, and infectious organisms present on histology. Tuberculosis is commonly implicated in lymph node abscess formation. Šarcoidosis forms noncaseating granulomas without evidence of necrosis. Hodgkin disease (Choice C) is a B-cell malignancy of the lymphatic system. Patients present with constitutional symptoms such as fever, night sweats, and unintentional weight loss. Biopsy may disclose atypical cells with bilobed nuclei, a large inflammatory infiltrate with eosinophil predominance, and/or diffuse fibrosis forming bands with scattered open spaces. Langerha

72 Exam Section 2: Item 23 of 50 National Board of Medical Examiners' Comprehensive Basic Science Self-Assessment 23. A previously healthy 6-year-old boy is brought to the physician by his mother because of a 1-week history of an itchy rash in his armpits, both hands and feet, and groin. The itching is most intense at night and keeps him awake. His vital signs are within normal limits. Examination of the skin shows multiple erythematous papules, some with burrows and many of which are excoriated, similar to the ones shown in the photograph. Which of the following questions to the mother will be most helpful in establishing the diagnosis? A) "Do you have any pets at home?" B) "Does anyone else in the family have an itchy rash like this?" OC) "Does your child take any medications?" D) "Has there been any recent travel?" E) "Have you changed soaps or detergents lately?"

B. Scabies is a skin infestation by the mite Sarcoptes scabiei that is transmissible by skin-to-skin contact, as in family members or sexual partners. The skin infestation typically occurs on the sides or webs of the fingers, wrists, axillae, areolae, or genitalia and leads to intense pruritus, as in this patient. Physical examination commonly demonstrates several small, erythematous papules that are frequently excoriated from scratching and may show mite burrows. In some cases, lesions demonstrate a crusted appearance. Diagnosis is confirmed by microscopic examination disclosing mites, eggs, or feces. Scabies is managed with topical or oral antiparasitic agents such as permethrin or lindane (less frequently used). Incorrect Answers: A, C, D, and E. Skin infections transmitted by household pets (Choice A) may include cutaneous larva migrans (twisting linear skin lesions caused by burrowing of larvae), tinea corporis (pruritic, erythematous circular or oval plaques), and cellulitis. Scabies lesions appear distinct from these zoonotic skin infections. Household pets may transmit scabies to humans; however, human-to-human transmission is more common. Rashes related to medications (Choice C) include type I (urticarial lesions) and type III (urticarial or purpuric rashes) hypersensitivity reactions. Intensely pruritic papules with burrows are atypical for medication-related eruptions. Rashes related to recent travel (Choice D) include a broad differential diagnosis depending on the region of travel including infectious causes (parasites, fungi, bacteria), which typically produce nodular or ulcerative lesions. Noninfectious causes include insect bites, which also appear nodular. This patient's erythematous papules and burrows are more typical of scabies. Contact dermatitis may arise from irritation of the skin from soaps or

99 Exam Section 2: Item 50 of 50 National Board of Medical Examiners' Comprehensive Basic Science Self-Assessment 50. A 16-year-old boy with intellectual disability is admitted to the hospital because of severe respiratory distress. Physical examination shows dislocation of the ocular lenses bilaterally, dyspnea, and tenderness and edema of the left lower extremity. Serum concentrations of methionine and homocysteine are increased, and serum cystine concentration is decreased. This patient most likely has a defect of which of the following enzymes? A) y-Cystathionase B) Cystathionine B-synthase C) Fumarylacetoacetate hydrolase D) Hepatic adenosyltransferase E) Methylmalonyl-CoA mutase

B. Homocystinuria is an autosomal recessive metabolic disorder most commonly caused by a defect in cystathionine B-synthase, leading to increased serum and urine concentrations of homocysteine. Additional deficits that cause homocystinuria include decreased affinity of cystathionine synthase for its cofactors and deficiency in homocysteine methyltransferase. Increased serum concentrations of methionine indicate that the defective enzyme is cystathionine B-synthase, rather than methionine synthase, another enzyme that causes homocystinuria when deficient. Patients with homocystinuria present with characteristic signs and symptoms including intellectual disability, a tall Marfanoid habitus with long limbs, pectus excavatum and pes cavus, ocular lens subluxation (with the lens typically subluxated inferomedially), and vascular disease such as thrombosis and atherosclerosis. Patients with homocystinuria are at significantly increased risk for intravascular thrombosis as illustrated by the tender, edematous left lower extremity in this case. Myocardial infarction at an early age is well-described. Homocystinuria caused by cystathionine B-synthase deficiency is treated with supplementation of cysteine, vitamins B6 (pyridoxine) and B12 (cyanocobalamin), folate, and a diet restricted in methionine. Incorrect Answers: A, C, D, and E. y-Cystathionase (Choice A) catalyzes the conversion of cystathionine to cysteine. Defects in this enzyme cause cystathioninuria, rather than homocystinuria. Fumarylacetoacetate hydrolase (Choice C) is an enzyme involved in the metabolism of phenylalanine and tyrosine. It cleaves fumarylacetoacetate, a metabolite of homogentisic acid, to acetoacetate and fumarate. Hepatic adenosyltransferase (Choice D) catalyzes the conversion of methionine to S-adenosylmethionine. Deficiency of this enzyme results

71 Exam Section 2: Item 22 of 50 National Board of Medical Examiners' Comprehensive Basic Science Self-Assessment 22. A 10-month-old female infant is brought to the physician because of a 1-day history of a rash. Her mother says that the infant also had a 3-day history of high fever and irritability, which spontaneously resolved 2 days ago. The patient is alert and playful. Her temperature is 37°C (98.6°F). Physical examination shows the findings in the photograph. Which of the following is the most likely causal agent? A) Epstein-Barr virus B) Human herpesvirus 6 C) Parvovirus B19 D) Rubeola virus O E) Varicella-zoster virus

B. Human herpesvirus 6 (HHV-6) is the cause of roseola infantum, or exanthem subitum. In a minority of cases, this viral syndrome may also be caused by human herpesvirus 7, enteroviruses, and adenoviruses, though HHV-6 is the most frequent cause. This viral syndrome is characterized by several days of high fevers sometimes accompanied by seizures; as the fevers subside, a characteristic, blanching, macular eruption forms on the neck and trunk, which then spreads outward to the face and extremities. The appearance of the rash indicates resolution of the viremia, so the diagnosis is made clinically rather than by viral serologies. Roseola infantum commonly affects infants but may also occur later in childhood. Other exanthems of childhood may be considered in the differential diagnosis of roseola infantum such as rubella, measles, erythema infectiosum, and enterovirus infections. The characteristic high fevers followed by a rash helps to identify roseola infantum as the etiology. Treatment of the syndrome is supportive; in most cases, it is a benign condition with a limited course. Incorrect Answers: A, C, D, and E. Epstein-Barr virus (Choice A) is a common cause of infectious mononucleosis. EBV is transmitted through respiratory secretions and saliva, causing theillness to be commonly acquired by teenagers and young adults rather than infants. Clinically, the syndrome commonly presents with fever, lymphadenopathy (typically involving the posterior cervical lymph nodes), and hepatosplenomegaly along with pharyngitis. Unless amoxicillin is administered for treatment of a misdiagnosed pharyngitis thought to be from Streptococcus pyogenes, patients do not typically develop a macular eruption as a feature of this syndrome. Parvovirus B19 (Choice C) is the cause of erythema infectiosum. This presents with pink patches on the

94 Exam Section 2: Item 45 of 50 National Board of Medical Examiners' Comprehensive Basic Science Self-Assessment 45. A 55-year-old woman with a history of alcohol dependence has severe epigastric pain and is vomiting large amounts of blood. She has slightly yellow skin and conjunctivae. Her abdomen is large and distended. The bleeding vessels are most likely enlarged due to increased blood flow directly supplied by which of the following veins? О А) Нерatic B) Left gastric C) Right gastro-omental D) Splenic E) Superior mesenteric

B. Left gastric vein pressure from portal hypertension leads to the development of esophageal varices that may rupture and lead to massive, life-threatening hematemesis. Portal hypertension (PH) is often a consequence of liver cirrhosis but may also occur in patients with schistosomiasis or portal venous thrombosis. The portal venous system describes a series of interconnected veins that drain blood from the colon, small intestines, spleen, liver, and stomach. While this blood eventually makes its way to the systemic circulation via the inferior vena cava, it must first pass through the liver. In cirrhosis, obliteration of the hepatic sinusoids through progressive fibrosis increases the resistance to blood flow through the liver, which is transmitted to the portal venous system causing PH. Anatomically, the portal vein starts at the confluence of the left gastric, splenic, and superior mesenteric veins. The left gastric vein drains the inferior esophageal veins. When engorged secondary to portal hypertension, esophageal veins can be visualized on esophagogastroduodenoscopy (EGD) as large vertical columns underneath the esophageal mucosa, representing dilated submucosal veins, and are termed esophageal varices (EVs). EVs are usually treated endoscopically by band ligation, and patients with cirrhosis must be screened for EVs, as the risk for life-threatening upper gastrointestinal bleeding is high. This patient presents with hematemesis likely from ruptured esophageal varices. Incorrect Answers: A, C, D, and E. Hepatic veins (Choice A) are not part of the portal system, as they drain blood from the liver to the inferior vena cava. Increased pressure in this system from right-sided heart failure can result in hepatic sinusoidal congestion and potential cardiac cirrhosis. Additionally, a stent placed between the hepatic v

3 Exam Section 1: Item 3 of 50 National Board of Medical Examiners' Comprehensive Basic Science Self-Assessment 3. A 55-year-old woman comes to the physician because of a 6-month history of progressive shortness of breath while climbing stairs and constant fatigue. She is 163 cm (5 ft 4 in) tall and weighs 107 kg (235 lb); BMI is 40 kg/m2. Her blood pressure is 170/110 mm Hg on three separate readings. Physical examination shows no other abnormalities. Echocardiography shows an ejection fraction of 45%. There are no regional wall motion or valvular abnormalities. An ECG shows that the sum of the magnitudes of the S wave in lead V, and the R wave in lead Vs is 45 mm (scale=10 mm/mV). Which of the following cellular changes is most likely in this patient's left ventricle compared with normal? A) Decreased sarcomeres and increased fibroblasts B) Increased sarcomeres in an array in parallel with each other C) Increased s

B. Left ventricular hypertrophy (LVH) is a common disorder with numerous causes, including chronic systemic hypertension, aortic stenosis, coarctation of the aorta, chronic aortic or mitral insufficiency, ventricular septal defect, infiltrative cardiomyopathy (eg, amyloidosis, sarcoidosis, Fabry disease, hereditary hemochromatosis), or hypertrophic cardiomyopathy. Hypertension and aortic stenosis are the most common causes. In these conditions, hypertrophy develops secondary to chronic contraction against increased afterload. This stimulates growth and remodeling of cardiomyocytes, resulting in increased sarcomeres in an array in parallel with each other in order to generate greater contractile force to overcome the afterload. The condition can progress to left-sided heart failure if untreated. Diagnostic testing includes ECG; one of the criteria suggestive of LVH is if the sum of the magnitude of the S wave in lead V2 and the R wave in lead V5 is greater than 35 mm as in this patient. Echocardiography is more sensitive than ECG for the presence of LVH. Cardiac MRI is the gold standard for diagnosis, though availability, cost, and logistics limit its use as a first-line screening tool. Incorrect Answers: A, C, D, and E. Decreased sarcomeres and increased fibroblasts (Choice A) occurs in myocardial scar formation, which may result after a myocardial infarction or acute inflammatory injury. It is also seen in Fabry disease as a result of dysfunctional metabolism of sphingolipids. Increased sarcomeres in an array in series with each other (Choice C) occurs in dilated cardiomyopathy as the cardiac myofibrils lengthen. Dilated cardiomyopathy may be caused by an underlying genetic predisposition, toxin exposure (eg, chronic alcohol use, chronic cocaine use, doxorubicin chemotherapy), infections (eg, Chagas disease, viral myo

57 Exam Section 2: Item 8 of 50 National Board of Medical Examiners' Comprehensive Basic Science Self-Assessment 8. A 3685-g (8-lb 2-oz) male newborn is delivered vaginally at term to a 26-year-old primigravid woman. Pregnancy and delivery were uncomplicated. Apgar scores are 9 and 9 at 1 and 5 minutes, respectively. Physical examination shows no abnormalities.. An antibiotic ointment is applied to his eyes to prevent ocular infections. Inhibition of which of the following is most likely to occur with this treatment? A) Bacterial cell-wall synthesis B) Bacterial protein synthesis C) DNA gyrase D) Folic acid synthesis E) Mycolic acid synthesis

B. Neonatal conjunctivitis is defined as conjunctival inflammation during the first 30 days of life. The three most common causal organisms are Neisseria gonorrhoeae, Chlamydia trachomatis, and herpes simplex virus. Gonococcal conjunctivitis presents within three to five days after birth and is characterized by purulent discharge. Gonococcal infections are aggressive and can rapidly progress to keratitis, corneal perforation, or endophthalmitis. Chlamydial conjunctivitis presents within five to 14 days of birth and is characterized by watery discharge. Herpetic conjunctivitis presents more than 14 days after birth. All neonates should be given topical antibiotic prophylaxis to prevent gonococcal conjunctivitis. Historically, topical silver nitrate was a common prophylactic agent that was also a frequent cause of chemical conjunctivitis, which presents within the first three days after birth. The use of silver nitrate has more recently been supplanted by the use of topical erythromycin, which binds to bacterial ribosomal 50s subunits and prevents protein synthesis. Incorrect Answers: A, C, D, and E. Inhibition of bacterial cell-wall synthesis (Choice A) is the mechanism of B-lactams such as penicillins and cephalosporins. Penicillin resistance is relatively prevalent in N. gonorrhoeae, and topical agents from this drug class are not routinely used for the prophylaxis of neonatal conjunctivitis. Inhibition of DNA gyrase (Choice C) is the mechanism of quinolone antibiotics. The side effect profile of quinolones, including arthropathy and tendinopathy, precludes their use in children and infants. Inhibition of folic acid synthesis (Choice D) is the mechanism of trimethoprim-sulfamethoxazole. Trimethoprim-sulfamethoxazole may contribute to neonatal kernicterus and is not commonly used for the prophylaxis of neonatal conjunc

26 Exam Section 1: Item 26 of 50 National Board of Medical Examiners' Comprehensive Basic Science Self-Assessment 26. A 55-year-old man is found to have gastroesophageal reflux disease. In addition to recommending lifestyle changes, the physician prescribes an agent that binds irreversibly to its site of action to inhibit gastric acid formation. Which of the following agents was most likely prescribed? A) Clarithromycin B) Esomeprazole C) Famotidine D) Misoprostol E) Sucralfate

B. Proton-pump inhibitors, such as esomeprazole, are first-line pharmacotherapy for patients presenting with gastroesophageal reflux (GERD) and associated reflux esophagitis. GERD typically presents with burning epigastric and lower chest pain, often in association with consumption of a large meal or trigger food. It is often worse with supine positioning. Acid reflux in the esophagus causes mucosal irritation and inflammation, which can present as mucosal erythema and multiple erosions (reflux esophagitis). Over time, if the GERD remains untreated, metaplasia can occur leading to Barrett esophagus, a state of premalignancy marked by intestinal metaplasia of the distal esophagus. Untreated, this can subsequently lead to the development of esophageal adenocarcinoma. Proton pump inhibitors decrease gastric acid secretion by parietal cells through irreversible inhibition of the H*/K* ATPase located on the luminal surface, which is the primary exporter of hydrogen ions into the gastric lumen. Incorrect Answers: A, C, D, and E. Clarithromycin (Choice A) is a macrolide antibiotic that may be used in the treatment of Helicobacter pylori infection. H. pylori is a spiral-shaped, gram-negative bacterium that is associated with chronic gastritis and peptic ulcer formation. Famotidine (Choice C) is a second-generation histamine type 2 (H2) receptor blocker, which is also used in the treatment of GERD. H2 receptors on gastric parietal cells are G protein-coupled receptors that stimulate increased intracellular CAMP concentrations, which leads to increased H*/K* ATPase activity. H2 receptor blockers are reversible competitive antagonists. Misoprostol (Choice D) is a prostaglandin-E, analog that may be used to prevent peptic ulcer formation in the setting of NSAID use (NSAIDS cause decreased prostaglandin concentrations as a result o

16 Exam Section 1: Item 16 of 50 National Board of Medical Examiners' Comprehensive Basic Science Self-Assessment H+ (nmol/L) 16. The graph shows a normal (Y) and abnormal (Z) relationship of bicarbonate to pH in the blood. Which of the following is the most likely cause of a shift from point Y to point Z in a healthy subject? 100 90 80 7O 60 50 40 30 25 20 15 44 100 80 A) Breathing a gas mixture with a high Pco2 40 60 B) Breathing a gas mixture with a low Po, 36 40 mm Hg C) Breathing air at 2 atmospheres of pressure 32- D) Ingestion of an acid 28 20 E) Metabolic alkalosis 24- 20- 16 12 8. 7.0 7.1 7.2 7.3 7.4 7.5 7.6 7.7 7.8 pH

B. The bicarbonate buffer system is an important regulator of acid-base balance in the blood. The reaction involves the conversion of carbon dioxide plus water to carbonic acid, which then dissociates into a bicarbonate ion and a hydrogen ion. To maintain a physiologic pH of 7.4 at normal body temperature, this system has a 20:1 bicarbonate to carbonic acid ratio. This relationship is determined by the Henderson-Hasselbalch equation, which is a model used to estimate the pH of a buffer solution. Blood acid-base balance in the body is coupled to the lungs and alveolar gas exchange. Breathing a gas mixture with a low Po2 stimulates hyperventilation in response to hypoxemia; this results in decreased alveolar Pco2. The alveolar-capillary interface is a thin membrane and Pco2 readily equalizes between the blood and the alveoli. The decrease in Pco2 shifts the carbonic acid buffer reaction to the left by the Le Chatelier principle, with increased conversion of bicarbonate ions and hydrogen ions into carbonic acid. The result of acute hyperventilation is an alkalotic shift in pH, causing the carbonic acid reaction to shift towards carbon dioxide and water production to account for losing carbon dioxide to the atmosphere. This process is referred to as a respiratory alkalosis. Incorrect Answers: A, C, D, and E. Breathing a gas mixture with a high Pco2 (Choice A) and breathing air at 2 atmospheres of pressure (Choice C) would result in high carbon dioxide concentrations in the alveoli and thus in the blood, shifting the carbonic acid buffer reaction to the right with increased generation of bicarbonate and hydrogen ions. This would result in a decreased blood pH from respiratory acidosis. Ingestion of an acid (Choice D) would result in an acute increase in hydrogen ions in the serum with a shift to the left in the buffer react

96 Exam Section 2: Item 47 of 50 National Board of Medical Examiners' Comprehensive Basic Science Self-Assessment 47. A 35-year-old woman with leiomyomata uteri undergoes an experimental treatment that involves instillation of an embolizing agent directly into the vessels that feed the leiomyomata. During this procedure, in order to reach the involved vessels, which of the following is the most direct course of the catheter after entering the femoral artery? A) Aorta gonadal artery uterine artery B) External iliac artery - internal iliac artery → uterine artery C) External iliac artery superior gluteal artery uterine artery D) Internal iliac artery - inferior vesical artery → uterine artery E) Internal iliac artery superior vesical artery → uterine artery

B. The main vessels supplying blood to the uterus are the uterine arteries, which have smaller subdivisions that supply the different portions of the uterus, cervix, and vagina. The uterine artery is a branch of the internal iliac artery, and travels along the broad ligament anterior to the ureter before dividing into the ascending and descending branches. The ascending branch joins the ovarian artery and further divides in the myometrium into the arcuate artery, radial artery, spiral artery, and basal artery. The descending branch is the major blood supply to the cervix and vagina. The internal iliac artery is a branch of the common iliac artery, which also gives rise to the external iliac artery. The femoral artery, in turn, is a continuation of the external iliac artery as it courses below the inguinal ligament. Therefore, to reach the patient's leiomyomata and uterine arteries in this case, the most direct course to travel is from the femoral artery to the external iliac artery and then ascend proximally until the common iliac artery bifurcation. Further advancements should occur down the internal iliac artery and, subsequently, the uterine artery. Incorrect Answer: A, C, D, and E. Entering the femoral artery and proceeding to the aorta to descend along the gonadal artery into the uterine artery (Choice A) is not feasible. The origin of the gonadal arteries occur from the upper abdominal aorta, near the level of the second lumbar vertebra. The gonadal artery, termed the ovarian artery in women, descends in the abdomen to supply arterial blood to the ovary. While the ovarian artery demonstrates an anastomotic connection with the uterine artery, this would not permit the passage of the embolizing agent to treat the patient's leiomyomata. The superior gluteal artery (Choice C) is a branch of the internal iliac artery

75 Exam Section 2: Item 26 of 50 National Board of Medical Examiners' Comprehensive Basic Science Self-Assessment 26. A 30-year-old woman comes to the physician for a health maintenance examination. She is training for a marathon and has been running up to 20 miles daily. She states that as long as she has adequate caloric intake, she feels well on long-distance runs of up to 20 miles. Physical examination shows no abnormalities. Her fasting serum glucose concentration is 60 mg/dL. After her glucose stores have been depleted, which of the following organs, in addition to the liver, is most likely to release newly produced glucose in this patient? A) Adrenal glands B) Kidney C) Pancreas D) Stomach E) Thyroid gland

B. The release of glucose into the serum by the kidney and liver, produced via gluconeogenesis and glycogenolysis, likely accounts for this patient's ability to maintain adequate serum glucose concentrations during periods of prolonged strenuous exercise. Depletion of glucose occurs quickly after the start of exercise, so the body must rely upon mechanisms for the continued synthesis and release of glucose into the blood to maintain appropriate glucose concentrations for tissue metabolism. Glycogen is produced during the fed state and stored primarily in the liver and skeletal muscle, but only the liver possesses the ability to release free glucose into the serum during the process of glycogenolysis as it possesses the enzyme glucose-6-phosphatase, which hydrolyzes glucose-6-phosphate to create free glucose molecules. Similarly, the kidney possesses this enzyme, which permits release of free glucose to the serum during gluconeogenesis. The liver participates in both glycogenolysis and gluconeogenesis, whereas the kidney primarily supplies glucose via gluconeogenesis. The free glucose can circulate in the serum, where it may be used by the skeletal muscle for aerobic and/or anaerobic metabolism during exercise. Incorrect Answers: A, C, D, and E. Adrenal glands (Choice A) release epinephrine in response to the stress of exercise and hypoglycemia. Epinephrine acts on hepatocytes to cause the release of intracellular calcium, which activates glycogen phosphorylase allowing for glycogenolysis in the liver. The pancreas (Choice C) releases glucagon from a cells during episodes of hypoglycemia. Glucagon acts on hepatocytes via a cyclic adenosine monophosphate pathway activating protein kinase A that activates glycogenolysis and gluconeogenesis. The stomach (Choice D) is not a site of gluconeogenesis. During periods of prolong

73 Exam Section 2: Item 24 of 50 National Board of Medical Examiners' Comprehensive Basic Science Self-Assessment 24. Which of the following abnormalities is most likely to result from occlusion of the vessel indicated by the arrow in the vertebral angiogram shown? A) Contralateral facial weakness B) Contralateral visual field deficit C) Ipsilateral anesthesia of the face D) Ipsilateral ataxia of extremity movements E) Ipsilateral palsy of lateral gaze

B. The vessel identified in the vertebral angiogram represents the posterior cerebral artery (PCA), the occlusion of which would result in infarction of the ipsilateral occipital lobe and a contralateral homonymous hemianopsia with macular sparing. The PCA originates near the rostral end of the basilar artery and contributes to the circle of Willis. The PCA forms anastomoses with the internal carotid arteries via the posterior communicating arteries. The PCA supplies much of the occipital lobes as well as the inferomedial temporal lobe, the thalamus, and parts of the brainstem. Visual symptoms are a prominent feature of a PCA territory infarction and include cortical blindness, contralateral homonymous hemianopsia with macular sparing, visual agnosia, and prosopagnosia. Incorrect Answers: A, C, D, and E. Contralateral facial weakness (Choice A) and ipsilateral anesthesia of the face (Choice C) are features of infarctions involving the primary motor cortex and primary somatosensory cortex, respectively, which are supplied by the middle cerebral artery. Ipsilateral ataxia of extremity movements (Choice D) can result from infarctions of either the anterior inferior cerebellar artery or the posterior inferior cerebellar artery. In the latter case, ipsilateral cerebellar findings, such as ataxia, form a core feature of lateral medullary syndrome, also known as Wallenberg syndrome. Ipsilateral palsy of lateral gaze (Choice E) is a feature of palsy of the abducens nerve, also known as the sixth cranial nerve. While sixth cranial nerve palsies may originate as a result of nuclear or fascicular ischemic injury in the brainstem, peripheral nerve injury is more common. Most of the blood supply to the abducens nucleus is derived from branches of the basilar artery. Educational Objective: The posterior cerebral artery originates fr

36 Exam Section 1: Item 37 of 50 National Board of Medical Examiners' Comprehensive Basic Science Self-Assessment 37. A 51-year-old woman comes to the physician because of a 6-month history of burning abdominal pain that occurs 1 to 2 hours after eating. She sweats profusely and has light-headedness when she stands. Her blood pressure is 105/70 mm Hg while standing. Physical examination shows epigastric tenderness. A CT scan of the abdomen shows a 2-cm mass on the proximal duodenum. Gastrin released by the tumor cells stimulates which of the following labeled cells in the photomicrograph shown to release hydrogen ions? A B C E A) B) C) D) E)

B. This patient's findings of postprandial abdominal pain, diaphoresis, epigastric tenderness, and orthostatic hypotension in the setting of a duodenal or pancreatic lesion are suggestive of a peptic ulcer secondary to Žollinger-Ellison syndrome (ZÉS). ZES stems from a pancreatic or duodenal gastrin-secreting tumor. Gastrin is typically produced by gastric G cells and stimulates parietal cells to release protons in order to form hydrochloric acid. Parietal cells can be identified by their large size, central, round nucleus, and intensely acidophilic cytoplasm. They are found in the middle region of the gastric gland. Excessive production of acid in the setting of ZES leads to recurrent, chronic duodenal or jejunal ulcers, which can present with abdominal pain, diarrhea secondary to malabsorption, and possible hematemesis, melena, or hematochezia. Pancreatic gastrinoma and ZES may each present as a component of multiple endocrine neoplasia type 1 (MEN1). Incorrect Answers: A, C, D, and E. Choice A indicates a gastric mucous neck cell, which produces gastric mucus and is typically found near the upper region of the gastric gland. They can be identified by a pale, foamy cytoplasm and a basolateral nucleus. Choice C indicates a chief cell. Chief cells can be identified by their basophilic cytoplasm and the presence of numerous cytoplasmic vesicles, which lend the cytoplasm a granular appearance. They produce pepsin. Choice D indicates a vascular endothelial cell, which can be identified by its position lining a capillary. Choice E indicates a perivascular fibroblast, marked by its presence within stromal connective tissue and dark, oblong nucleus. Educational Objective: Gastrinomas are characterized by the excessive production of gastrin, which stimulates parietal cells to produce and release hydrochloric acid. Parietal

11 Exam Section 1: Item 11 of 50 National Board of Medical Examiners' Comprehensive Basic Science Self-Assessment 11. A 22-year-old man is brought to the emergency department 30 minutes after collapsing at home. He has a 3-year history of intravenous drug use. He appears confused. His temperature is 39.5°C (103.1°F). Physical examination shows subungual hemorrhages of the fingers and jugular venous distention. A holosystolic murmur is heard at the lower left sternal border. Blood cultures are positive for Staphylococcus aureus. A chest x-ray shows multifocal airspace opacities. The most likely cause of these findings is infection of which of the following labeled cardiac valves in the CT scans of the chest? -B C A A) B) C) D)

C. Intravenous drug use is a risk factor for infective endocarditis and valvular dysfunction as a result of the nonsterile injection of material into the venous system. Though left-sided infective endocarditis is more common in general, the tricuspid valve is most commonly involved in the setting of intravenous drug use. Tricuspid valve endocarditis is typically associated with Staphylococcus aureus, Pseudomonas aeruginosa, and Candida organisms. The formation of vegetations on the valve and local inflammatory damage can lead to severe tricuspid regurgitation, which presents as a holosystolic murmur best heard in the left lower sternal border. Severe tricuspid regurgitation is a risk factor for the development of right-sided heart failure, which is characterized by increased jugular venous pressure, hepatomegaly, ascites, and peripheral extremity edema. Other complications include septic pulmonary emboli, which present as multifocal peripheral cavitary lung lesions. The tricuspid valve can be identified on CT imaging by its location between the right ventricle and right atrium. The right ventricle is the most anterior and inferior part of the heart, as seen in the axial slices of the patient's CT scan. Incorrect Answers: A, B, and D. Choice A represents the aortic valve, which separates the left ventricle from the ascending thoracic aorta. It is located superior to the left ventricle, anterior to the left atrium, and posterior to the pulmonic valve. It may be, along with the mitral valve, more commonly affected in cases of endocarditis that are unrelated to intravenous drug abuse. Systemic septic emboli are common. Choice B represents the pulmonic valve, which separates the right ventricle from the pulmonary trunk. It is anterior to the aorta. It is the least common valve affected by endocarditis. Choice D represents t

84 Exam Section 2: Item 35 of 50 National Board of Medical Examiners' Comprehensive Basic Science Self-Assessment 35. A 1-year-old boy is admitted to the hospital for antimicrobial treatment of perianal pain and erythema. He has a history of recurrent infections, including an admission to the hospital at the age of 2 weeks because of high-grade fever and swelling and redness around the umbilical cord stump. Physical examination shows perianal erythema and tenderness but no fluctuance. Laboratory studies show: Leukocyte count Segmented neutrophils Bands 74,000/mm3 (N=5000-19,500) 67% 9% Eosinophils Basophils Lymphocytes Monocytes 1% 1% 20% 2% This patient most likely has a genetic deficiency of which of the following immune system components? A) C9 B) IgG C) B2 Integrin D) Interferon alpha E) Tumor necrosis factor-a

C. Leukocyte adhesion deficiency (LAD) results from a defect in the attachment of leukocytes to the vascular endothelium, which consequently results in the impaired recruitment and migration to sites of extravascular inflammation or infection. LAD type 1 is caused by dysfunctional LFA-1 integrin (CD18) protein on the leukocyte surface, which does not allow for the normal attachment of leukocytes to the vascular endothelium. Specifically, it is the absence of the B, integrin subunit that leads to impaired LFA-1 function. LAD is typically characterized by recurrent bacterial infections, impaired wound healing, a delayed detachment of the umbilical cord after birth, and complete lack of leukocytes at sites of infections with no pus. The actions of leukocyte phagocytosis and bacterial killing are not impaired. Laboratory studies in patients with LAD will show increased absolute leukocyte counts in the blood since the leukocytes are unable to maintain their normal marginated position outside of the blood stream against the vessel walls. Incorrect Answers: A, B, D, and E. C9 (Choice A) is a component of the complement system that polymerizes to form the membrane attack complex (along with C5b, C6, C7, and C8) when activated. Pore formation by C9 causes membrane disruption and death of the targeted cell. C9 deficiency is associated with recurrent Neisseria infections. Immunoglobulin G (IgG) (Choice B) deficiency may be present in B- and T-lymphocyte disorders such as X-linked agammaglobulinemia, common variable immunodeficiency, ataxia- telangiectasia, hyper-IgM syndrome, and Wiskott-Áldrich syndrome. Patients are susceptible to recurrent infections. It is not associated with increased absolute leukocyte counts. Interferon alpha (Choice D) is a glycoprotein produced by cells that are infected with a virus. Interferon alpha u

22 Exam Section 1: Item 22 of 50 National Board of Medical Examiners' Comprehensive Basic Science Self-Assessment 22. A31-year-old man who works as a chef comes to the emergency department 30 minutes after accidentally cutting his right thumb with a knife. Examination of the right hand shows a 3-cm-long, 0.6-cm-deep laceration of the thumb. The wound is cleaned, sutured, and heals normally, leaving a faint scar by 1 year after the injury. The remodeling of the scar that occurred during the first year after injury is primarily mediated by which of the following? A) Chemokines B) Interleukin-1 (IL-1) C) Matrix metalloproteinases D) Transforming growth factor-ß E) Tumor necrosis factor

C. Matrix metalloproteinases (MMPS) are enzymes that lyse proteins using a chelated metal ion cofactor. MMPS are necessary for the healing of wounds as well as for the remodeling of bone and cartilage. Wound healing occurs via a staged process. In early wound healing, platelet aggregation and platelet plug formation occur to achieve hemostasis. In the following one to two days, neutrophils and macrophages infiltrate the area and release growth factors and cytokines that stimulate fibroblast proliferation. Around day three, granulation tissue forms as collagen is deposited into the area and angiogenesis begins to occur. During this time, wound edges contract from the action of myofibroblasts, further aiding in healing. Epidermal cells migrate across the newly deposited collagen matrix to reconstitute normal skin appearance. In the following weeks and months, scar formation and remodeling occurs via metalloproteinase-mediated collagen breakdown. Incorrect Answers: A, B, D, and E. Chemokines (Choice A) are cell signaling proteins that specifically promote cellular chemotaxis. An example of a chemokine is IL-8, which induces chemotaxis of neutrophils and granulocytes. Interleukin-1 (IL-1) (Choice B) is a pyrogen secreted by macrophages and monocytes; it also promotes vasodilation, activates osteoclasts, and stimulates the proliferation of granulocytes. IL-1 dysregulation has been implicated in cartilage damage in osteoarthritis. Transforming growth factor-ß (TGF-B) (Choice D) is a cytokine that is secreted by multiple cells including macrophages. TGF-ß has multiple functions that include inhibiting B-cell proliferation and Th17 cells. Tumor necrosis factor (Choice E) is secreted by activated macrophages, and it is key in the formation of granulomas and immune protection against mycobacterial infections. It is increased i

25 Exam Section 1: Item 25 of 50 National Board of Medical Examiners' Comprehensive Basic Science Self-Assessment 25. A 70-year-old woman with osteoporosis comes to the physician for her scheduled injection of ibandronate. She is accompanied by her 75-year-old husband. Physical examination shows no abnormalities. The husband asks the physician, "Why don't I have osteoporosis? For the past 50 years we have done everything together, including eating the same foods and exercising. I even used to smoke and drink. Yet she has osteoporosis, and I don't." Which of the following is the most appropriate response? A) "Drinking alcohol when you were younger was protective against osteoporosis." B) "Exercising is good for both of you, but it is more protective for men." C) "Men start out with a higher peak bone mass compared with women." D) "Your estrogen concentration is lower than your wife's." E) "Your testosterone concentrat

C. Osteoporosis is a common condition that is characterized by the progressive loss of bone mineral density leading to decreased bone strength. This decreased bone mineral density often leads to fragility fractures, which can greatly decrease mobility, and even lead to death in elderly individuals. The disease classically affects postmenopausal women with inflammatory disorders, and individuals with metabolic or endocrine disorders such as hypercortisolism or hyperparathyroidism. Osteoporosis can also be induced by long-term treatment with medications that cause an increase in bone resorption such as corticosteroids. The diagnosis is made using dual energy x-ray absorptiometry via calculation of a T-Score. AT-score of -2.5 or lower, that is a bone density measurement that is less than 2.5 standard deviations below the mean, is diagnostic of osteoporosis. T-scores between -1 and -2.5 are defined as osteopenia.Osteoporosis tends to affect females at a higher rate than males. There are a number of reasons for this. Males generally develop higher peak bone mass during the first three decades of life. Given the chronic and generally linear rate of bone mineral loss, males will therefore tend to retain higher bone mineral density for a longer period of time given the higher bone mineral density starting point than females. Moreover, as estrogen output declines during menopause, bone mineral density loss occurs more rapidly during this phase of life. As both testosterone and estrogen stimulate osteoblasts and increase or maintain bone mineral density, the loss of estrogen leads to an accelerated loss of bone mass in women. Incorrect Answers: A, B, D, and E. Alcohol consumption (Choice A) has a negative effect on bone mineral density; however, this effect is mostly seen with heavy drinking over many years. Alcohol is a diureti

40 Exam Section 1: Item 41 of 50 National Board of Medical Examiners' Comprehensive Basic Science Self-Assessment 41. During an experiment, investigators use an in vitro cell culture system to study leukocyte function. As part of the study, a new drug that prevents polymerization of actin filaments is tested. Which of the following leukocyte functions is most likely to be inhibited by this new drug? A) Binding to mannose receptor B) Internalization into clathrin-coated pits C) Phagocytosis D) Pinocytosis E) Receptor-mediated endocytosis

C. Phagocytosis would be affected by a new drug that prevents polymerization of actin filaments. Actin provides a mechanical superstructure for the phagocyte to maintain shape, acts as a highway to shuttle enzymes and their respective substrates together to augment intracellular signaling, and aids in the endocytosis of large particles. This latter role is a crucial part of phagocytosis. When phagocytes come into contact with an opsonized pathogen at the leading edge, the phagocyte binds the Fc portion of immunoglobulin via its Fc receptor. Grouping of multiple Fc receptors results in the activation of multiple downstream signaling pathways. Some of these include signaling through the Arp2/3 complex, WASP, and Cdc42, which ultimately leads to actin polymerization and formation of the phagocytic cup that surrounds the pathogen. Actin polymerization extends the edges of the plasma membrane in a circumferential fashion around the pathogen, thereby aiding in complete endocytosis. Inhibition at any step of this process could theoretically lead to failed phagocytosis. Incorrect Answers: A, B, D, and E. Binding to mannose receptors (Choice A) on the surface of macrophages, not neutrophils, results in endocytosis of several bacterial pathogens that display mannose residues on their cell surface. Mannose receptors are recycled from the endosome back to the cellular membrane. This process is independent of actin. Internalization into clathrin-coated pits (Choice B), pinocytosis (Choice D), and receptor-mediated endocytosis (Choice E) are all descriptive terms for a similar process. Pinocytosis refers to the endocytosis of small particles from the extracellular matrix and can occur via multiple mechanisms, including via clathrin-mediated endocytosis, caveolin-mediated endocytosis, and micropinocytosis. Clathrin-mediated endocytos

79 Exam Section 2: Item 30 of 50 National Board of Medical Examiners' Comprehensive Basic Science Self-Assessment 30. An investigator is studying the effects of succinylcholine and Drug X on muscle contraction in an experimental animal model. During the study, the force of contraction of abdominal muscles is recorded after the administration of succinylcholine alone. In a separate experiment, Drug X is administered concurrently with succinylcholine. Results show that the phase I neuromuscular blockade induced by succinylcholine is enhanced by concurrent administration of Drug X. However, when Drug X is administered during the succinylcholine-induced phase Il neuromuscular blockade, it is found that Drug X reverses the effects of succinylcholine. Drug X is most likely which of the following? A) Atropine B) Dantrolene C) Neostigmine D) Rocuronium E) Verapamil

C. Succinylcholine is a depolarizing skeletal muscle relaxant commonly used to facilitate rapid sequence tracheal intubation. It is a potent acetylcholine receptor agonist that causes depolarization of the muscle fibers, seen clinically as fasciculations, followed by muscle relaxation. It is structurally similar to acetylcholine but is not metabolized by acetylcholinesterase. Rather, after it diffuses away from the neuromuscular junction, it is hydrolyzed by pseudocholinesterase in the plasma and liver. This alternative metabolism causes it to persist in the synaptic cleft for a longer duration, leading to prolonged depolarization and prevention of further skeletal muscle activation by acetylcholine. Cholinesterase inhibitors, such as neostigmine, reverse neuromuscular blockade when administered after succinylcholine as a result of an increase in acetylcholine concentration available for receptor binding as succinylcholine concentration decreases. However, if neostigmine is administered concurrently with succinylcholine, it initially enhances paralysis provided by the depolarizing neuromuscular blocker. This enhancement is caused by the increased concentration of acetylcholine in the nerve terminal, leading to a more intense depolarization, in addition to neostigmine inhibition of pseudocholinesterase. Incorrect Answer: A, B, D, and E. Atropine (Choice A) is an anticholinergic that acts by competitively binding to muscarinic acetylcholine receptors, particularly in the heart and bronchial smooth muscle. It is most commonly used to treat symptomatic bradycardia. It does not act on nicotinic receptors in the neuromuscular junction and would not affect the potency of succinylcholine. Dantrolene (Choice B) is a ryanodine receptor antagonist, used in the treatment of malignant hyperthermia. It prevents the release of calciu

6 Exam Section 1: Item 6 of 50 National Board of Medical Examiners' Comprehensive Basic Science Self-Assessment 6. The incidence of Haemophilus influenzae type b meningitis in children below age 2 years has decreased since the introduction of the current vaccination for this infection. This vaccine is composed of which of the following? A) Anti-idiotype B) Attenuated bacterial strain OC) Conjugated capsular polysaccharide D) Killed whole cell E) Outer membrane complex F) Peptide G) Recombinant cell protein H) Toxoid

C. The Haemophilus influenzae type b vaccination uses conjugated capsular polysaccharide to provoke an adaptive immune response. Vaccination is a form of active immunity designed to prevent disease by stimulating the immune system to develop and maintain the ability to respond to a foreign antigen. The maintenance of immunity is referred to as immune memory. Vaccinations may contain the whole infectious organism in a killed or a live-attenuated state that is unable to produce virulent disease. They may also be fractional, containing a specific component of a pathogen that the immune system can recognize. Fractional vaccines may be developed from a particular protein or polysaccharide associated with the organism. Bacterial vaccinations commonly employ a capsular or cell-wall polysaccharide, or a denatured toxin that is produced by the bacterium (referred to as a toxoid). Examples of capsular polysaccharide-based vaccines include vaccinations against Haemophilus influenzae type b, Neisseria meningitidis, and Streptococcus pneumoniae. Examples of toxoid-based vaccines include those against Corynebacterium diphtheriae, Clostridium tetani, and Bordetella pertussis. The presence of the polysaccharide alone is usually insufficient to provoke a lasting immune response as the polysaccharide is not usually well- presented to T cells on major histocompatibility complexes (MHC). Conjugation of the pathogen-specific polysaccharide with another antigen that is known to be highly immunogenic (presented on MHC) leads to a robust B- and T-cell response, with lasting immune memory. Toxoids are commonly used as conjugates. Incorrect Answers: A, B, D, E, F, G, and H. Anti-idiotype (Choice A) antibodies are those that bind to the variable region of another antibody. Attenuated bacterial strain (Choice B) is a form of vaccine in which the

74 Exam Section 2: Item 25 of 50 National Board of Medical Examiners' Comprehensive Basic Science Self-Assessment 25. A full-term female newborn is evaluated shortly after birth. Pregnancy and delivery were uncomplicated. During the first 5 minutes after delivery, which of the following is most likely to be increased in this newborn? A) Flow from pulmonary artery to aorta (through ductus arteriosus) B) Pulmonary artery pressure C) Pulmonary blood flow D) Right atrial pressure to left atrial pressure gradient E) Venous return when the umbilical cord is ligated

C. The fetal circulation undergoes important physiologic changes as the neonate transitions to extrauterine life. These changes revolve around adjusting to receiving oxygenated blood from the lungs instead of through the umbilical cord. Fetal blood flow involves three primary physiologic shunts: the ductus venosus, ductus arteriosus, and foramen ovale. Oxygenated blood from the mother flows through the umbilical vein and a portion perfuses the liver. The majority passes through the ductus venosus to enter the inferior vena cava and mix with deoxygenated blood returning from the lower portion of the fetus. As blood enters the right atrium, it is shunted through the foramen ovale into the left atrium because of high pulmonary vascular resistance resulting from hypoxic pulmonary vasoconstriction in the intrauterine environment. Blood that does pass through the right ventricle and into the pulmonary trunk is directed into the aorta via the ductus arteriosus. In the first few minutes of life, the new exposure of the lungs to an oxygen-rich environment results in a dramatic decrease in pulmonary vascular resistance and reversal of the right- to-left shunt. Pulmonary blood flow increases accordingly. Other factors that are essential in establishing gas exchange in the neonatal lungs include clearing of fluid from the alveoli (via cough and crying) and adequate surfactant production to prevent alveolar collapse. Incorrect Answers: A, B, D, and E. Flow from pulmonary artery to aorta (through ductus arteriosus) (Choice A) decreases as the pulmonary vascular resistance drops and the pressure gradient between the pulmonary and systemic circulation reverses. Pulmonary artery pressure (Choice B) decreases secondary to oxygen-induced pulmonary vascular smooth muscle relaxation. The right atrial pressure to left atrial pressure gradie

7 Exam Section 1: Item 7 of 50 National Board of Medical Examiners' Comprehensive Basic Science Self-Assessment 7. A 24-year-old man with AIDS comes to the physician because of a 1-month history of abdominal cramps and watery diarrhea. These symptoms have become increasingly severe during the past 5 days. His pulse is 95/min. Physical examination shows dry mucous membranes and decreased skin turgor. A biopsy specimen of the colon is shown in the photomicrograph. Which of the following is the most likely causal organism? A) Candida albicans B) Cryptococcus neoformans C) Cryptosporidium parvum OD) Cytomegalovirus E) Giardia lamblia OF) Mycobacterium avium complex

C. The photomicrograph demonstrates small, round oocysts on the colonic epithelial surface, consistent with cryptosporidiosis, an AIDS-defining illness. Cryptosporidium is a genus of protozoal pathogens that cause cryptosporidiosis, a diarrheal illness that is mild and self-limited in immunocompetent hosts but which is severe and often life-threatening in immunocompromised hosts. Signs and symptoms include fever, weight loss, symptoms of dehydration and orthostasis, severe watery diarrhea, cramping abdominal pain, nausea, and vomiting. Immunocompromised patients may develop disseminated infection with involvement of the liver and lungs. Diagnosis is made with stool antigen testing or with detection of luminal oocysts by acid-fast staining. Treatment is supportive, with repletion of fluids and electrolytes, as well as antiparasitic therapy. Nitazoxanide is the treatment of choice in immunocompetent patients, whereas immunocompromised patients require antiretroviral therapy to improve their immune status and CD4+ cell counts, as well as combination regimens that involve nitazoxanide and other agents, such as azithromycin or paromomycin. Incorrect Answers: A, B, D, E, and F. Candida albicans (Choice A) can produce oral thrush and esophagitis in immunocompromised patients. Candida esophagitis is an AlIDS-defining illness. Staining with Gomori methenamine silver or periodic acid-Schiff shows pseudohyphae and budding yeasts. Cryptococcus neoformans (Choice B) is a potential cause of meningitis in AIDS patients. Staining with India ink or mucicarmine shows an encapsulated yeast with narrow-based budding. Cytomegalovirus (CMV) (Choice D) is a herpesvirus (human herpesvirus-5) that causes esophagitis, pneumonia, and retinitis in AIDS patients. Light microscopy may show intracellular inclusion bodies known as Cowdry bodies. Diag

20 Exam Section 1: Item 20 of 50 National Board of Medical Examiners' Comprehensive Basic Science Self-Assessment 20. A 33-year-old woman comes to the physician because of a 2-week history of fatigue, difficulty swallowing, and paresthesias of her lips and fingers. She also has had episodes of painful wrist spasms during this period. She has Graves disease and underwent a surgical procedure to remove part of her thyroid gland 3 weeks ago. Her only medication is low-dose propranolol. Vital signs are normal. Physical examination shows no abnormalities except for a well-healing incision on the neck. An ECG shows a prolonged QT interval. Which of the following is the most likely cause of this patient's new symptoms? A) Adverse effect of propranolol OB) Hyperparathyroidism C) Hyperthyroidism OD) Hypoparathyroidism O E) Hypothyroidism F) Laryngeal nerve injury

D. Parathyroid hormone (PTH) is a peptide hormone that increases osteoclast activity and bone resorption in order to maintain calcium homeostasis. It is produced by the four parathyroid glands, which are located on the posterior aspect of the thyroid gland. PTH is decreased in hypoparathyroidism, which may be primary or caused by a PTH-independent increase in serum calcium. Primary hypoparathyroidism can occur following the unintentional resection of the parathyroid glands during thyroidectomy, as in this case, but may also be a result of autoimmune destruction of the parathyroid glands. As a consequence of surgical removal of the parathyroid glands, patients will develop symptoms of hypocalcemia including perioral paresthesia, carpopedal spasm, laryngospasm, and prolonged QT interval. The Chvostek sign, in which tapping of the facial nerve leads to spasm of the facial muscles, and Trousseau sign, in which an inflated blood pressure cuff occluding the brachial artery causes carpopedal spasm, are two physical examination signs that may indicate hypocalcemia. While this patient demonstrates signs of hypocalcemia, PTH may also be decreased when increased concentrations of calcium exert negative feedback on the parathyroid gland. For example, in the case of excessive calcium intake or hypervitaminosis D, calcium will inhibit the production of PTH, causing the hormone concentration to fall. Lytic osseous primary or metastatic cancer, in which calcium is often released from bones that are infiltrated with malignancy, is another potential cause of hypercalcemia and decreased PTH concentrations. Incorrect Answers: A, B, C, E, and F. Major side effects of B-adrenergic blockers, including propranolol (Choice A), are decreased heart rate and contractility, bronchospasm, and hypoglycemia. This patient's symptoms of hypocalcemia po

100 Exam Section 3: Item 1 of 50 National Board of Medical Examiners' Comprehensive Basic Science Self-Assessment 1. A previously healthy 32-year-old woman comes to the physician because of a 3-month history of heavy menstrual bleeding, intermittent nosebleeds, and blood in her urine and stools. She says, "I've had to stop flossing because my gums bleed for an hour afterwards." Physical examination shows numerous ecchymoses and petechiae over the lower extremities. A test result for circulating antiplatelet antibodies is positive. A photomicrograph of a biopsy specimen of bone marrow is shown. Which of the following is the most likely pathogenesis of this patient's disease? A) Inability of the platelets to be released into the blood B) Megakaryocyte dysfunction C) Monosomy 7 D) Platelet destruction O E) Platelet dysfunction

D. Platelet destruction as a result of circulating antiplatelet antibodies accounts for this patient's presentation, which is consistent with immune thrombocytopenic purpura (ITP). Anti-Gpllb/Illa antibodies bind to platelets, which results in their opsonization. Macrophages within the spleen then phagocytose the platelet-antibody complex, resulting in thrombocytopenia. ITP is diagnosed in the setting of a low platelet count in the absence of other explanatory causes for thrombocytopenia. Bone marrow biopsy will classically show an increased number of megakaryocytes indicating adequate platelet production but increased peripheral destruction of platelets. It classically presents with petechiae and purpura, and prolonged bleeding time on laboratory analysis. Severe ITP may lead to uncontrolled hemorrhage. Acute ITP often follows an infection, and generally is self-limited. Chronic ITP is first treated with steroids (which may be required long-term), intravenous immunoglobulin, or immunomodulators. Refractory ITP that is non-responsive to such therapies is treated with splenectomy, which results in remission of thrombocytopenia in the majority of cases because of the spleen's primary role in the underlying pathophysiology. Incorrect Answers: A, B, C, and E. Inability of the platelets to be released into the blood (Choice A) is not a feature of ITP. These patients demonstrate an increased number of megakaryocytes that create and release platelets into the peripheral blood, which then are targeted by antibody-mediated destruction that results in thrombocytopenia. Megakaryocyte dysfunction (Choice B) may occur in patients with ITP but is not hypothesized to be the primary mechanism by which thrombocytopenia occurs. Megakaryocyte dysfunction also occurs in the setting of myelodysplastic syndrome (MDS), but a bone marrow biop

78 Exam Section 2: Item 29 of 50 National Board of Medical Examiners' Comprehensive Basic Science Self-Assessment 29. A 65-year-old man comes to the physician because of a 3-month history of substernal chest pain that occurs after eating. Physical examination shows no abnormalities except for systolic hypertension. Results of an exercise stress test are positive. The table shown depicts the results of an exercise stress test in 1000 patients at similar risk compared with the presence or absence of coronary artery disease (CAD). CAD Absent Present 442 208 650 80 270 522 478 1000 Positive Exercise Stress Test Negative 350 Based on these data, which of the following is the pretest probability of CAD in this patient? A) 35% B) 48% C) 52% D) 65%

D. Pretest probability is defined as the baseline risk for having the condition of interest, in this case coronary artery disease (CAD), in a given population. It is proportionate to the prevalence of the disease in the population. It is determined by taking the number of people in the population with the condition of interest and dividing it by the total number of people in the population. In this case, 650 individuals have coronary artery disease out of a population of 1000. 650/1000 = 0.65 = 65%. Pretest probability is important because it impacts the positive predictive value (PPV) and negative predictive value (NPV) of a test. PPV and NPV are statistical measures that define the proportion of positive test results that are true positives (positive tests in patients who have the condition) and true negatives (negative tests in patients without the condition). Both PPV and NPV vary with disease prevalence; PPV varies directly with prevalence, whereas NPV varies inversely with prevalence. Therefore, the more prevalent a disease, the greater the PPV of a test used in that population and the lower the NPV of a test used in that population. In contrast, pretest probability does not impact the sensitivity or specificity of a test. Incorrect Answers: A, B, and C. 35% (Choice A) is the rate of individuals in the population who do not have CAD. This is equal to 1-pretest probability. 48% (Choice B) is approximately the number of individuals in the population who have a negative exercise stress, and 52% (Choice C) is approximately the number of individuals in the population who have a positive exercise stress test, whether or not they have CAD. Educational Objective: Pretest probability is defined as the baseline risk for having the condition of interest. The pretest probability alters the PPV and NPV of a test but not its s

13 Exam Section 1: Item 13 of 50 National Board of Medical Examiners' Comprehensive Basic Science Self-Assessment 13. An investigator is studying a new hormone, Hormone X. To determine its cell target, a purified, radioactively labeled form of Hormone X is added to tissue cultures. Subsequent analysis shows that the radioactivity is detected only around the region of the plasma membrane. Hormone X is most similar to which of the following hormones? A) Aldosterone B) Cortisol C) Estradiol D) Prolactin O E) Thyroxine (T4)

D. Prolactin is a single-chain polypeptide hormone that is structurally homologous to growth hormone. It is secreted by the basophils in the anterior pituitary gland. Prolactin binds to the prolactin receptor, a member of the Class i ytokine receptor family, which spans the plasma membrane. It has an extracellular binding domain, a transmembrane domain, and an intracellular domain. Thus, a radioactive label on prolactin would concentrate around the region of the plasma membrane where its receptor is located. Prolactin binding to this receptor causes a series of downstream reactions to occur, ultimately utilizing the JAK/STAT signaling pathway. The physiologic effects of prolactin include stimulating milk production by mammary tissue, inhibition of ovulation, and inhibition of spermatogenesis. The regulation of prolactin is controlled by dopamine, which inhibits its production, and thyrotropin-releasing hormone, which stimulates its production. Increased prolactin concentrations cause dopamine production, leading to inhibition of further prolactin secretion. In this way, prolactin inhibits its own secretion. Incorrect Answers: A, B, C, and E. The receptors for aldosterone (Choice A), cortisol (Choice B), estradiol (Choice C), and thyroxine (Choice E) are nuclear receptors that contain DNA-binding domains. Nuclear receptors can initially be located in either the cytosol or nucleus. They bind to hormones that are either lipophilic and therefore capable of diffusion across the outer cell membrane, or which are transported into the cell via carrier-mediated transport. Once nuclear receptors bind their respective hormones, they translocate into the nucleus, if not already there, where they act as DNA transcription factors to regulate the expression of target genes. A similar analysis to that described in the question would d

47 Exam Section 1: Item 48 of 50 National Board of Medical Examiners' Comprehensive Basic Science Self-Assessment 48. A 4-year-old boy is brought to the emergency department by his mother 6 hours after she noticed that his urine was red. He is otherwise feeling well. Fifteen days ago, the patient had a sore throat, fever, and cough. His mother thought he had the flu and treated him symptomatically with rest and analgesics, and his status improved until now. His temperature is 38.5°C (101.3°F), pulse is 110/min, respirations are 22/min, and blood pressure is 100/50 mm Hg. Physical examination shows normal breath and cardiac sounds, no organomegaly, and 1+ lower extremity edema, bilaterally. Laboratory studies show: Serum Urea nitrogen Creatinine 40 mg/dL 2 mg/dL 3.6 g/dL Albumin Urine Color Blood Protein Ketones RBC RBC casts red-brown 3+ 2+ negative 30-50/hpf few Which of the following is the most likely diagnosis?

D. Proliferative glomerulonephritis is a general term that refers to histologic characteristics on kidney biopsy but is commonly used to describe postinfectious glomerulonephritis. In this instance, the patient is a young boy with nephritic syndrome following acute pharyngitis. This is most consistent with poststreptococcal glomerulonephritis, a condition that occurs primarily in children two to four weeks after an episode of pharyngitis with Streptococcus pyogenes, a gram-positive bacterium that also causes skin and soft-tissue infections. Postinfectious glomerulonephritis is a nephritic syndrome presenting with gross or microscopic hematuria and RBC casts in addition to variable degrees of proteinuria. This is in contrast to nephrotic syndromes, which present typically with high concentrations of proteinuria, edema, and hyperlipidemia. It is thought that S. pyogenes contains nephritogenic antigens that lead to a robust inflammatory response within the kidneys, with the deposition of immunoglobulin G and C3 diffusely throughout the mesangium and capillary walls. On laboratory evaluation, low serum C3 concentrations, positive anti-streptolysin O, and anti-DNase titers are noted. On light microscopy, the glomeruli are enlarged and hypercellular, which is termed proliferative glomerulonephritis. Treatment is supportive, as most patients experience complete recovery within the first few weeks of illness. Incorrect Answers: A, B, C, and E. Membranous nephropathy (Choice A) is a nephrotic syndrome that is characterized by proteinuria (more than 3 g/day), edema, hypoalbuminemia, and hyperlipidemia. It is the most common nephrotic syndrome in white adults and can be primary (idiopathic) or secondary to conditions such as systemic lupus erythematosus, autoimmune disease, hepatitis B or C, or medications. Untreated, it may prog

39 Exam Section 1: Item 40 of 50 National Board of Medical Examiners' Comprehensive Basic Science Self-Assessment 40. A study is conducted to assess the effects of aging on cognitive and physiologic function. Participants are recruited via advertisements in the local and regional news media, and all persons who answer the advertisement are selected for study. Which of the following best describes this sampling technique? A) Case-control B) Geographic C) Population-based control D) Self-selection E) Stratified randomization

D. Self-selection occurs when individuals select for themselves to participate in a study. This leads to a potential bias known as self-selection bias. An example of this could be a voluntary, internet- based survey. Such a survey would be limited to persons who use the internet, and then only those who decided to participate. Therefore, non-internet users and persons who decide for whatever reason not to participate would be systematically underrepresented in the study. In this study about aging on cognitive and physiologic function, the sampling technique presented self- selects for individuals who utilize local and regional news media, and who are sufficiently cognitively intact to understand and respond to the request for participation, both potential sources of selection bias. Incorrect Answers: A, B, C, and E. A case-control study (Choice A) investigates an association between an exposure and an outcome. In this study design, a group of patients with the disease (cases) are identified by the investigators and controls are identified from the same population using a matching process. Geographic sampling (Choice B) chooses individuals from a population on the basis of geographic location. For example, sampling of the population of a city may select certain numbers of individuals from each city block. Population-based control (Choice C) refers to when the control group is selected directly from the population of interest. This is often done in case-control studies when the control group is selected directly from the same population from which the cases are selected. Stratified randomization (Choice E) is a process of randomization in which study subjects are enrolled and then stratified into groups on the basis of factors that may influence the outcome of the study. After stratification, the individuals are randomiz

45 Exam Section 1: Item 46 of 50 National Board of Medical Examiners' Comprehensive Basic Science Self-Assessment 46. A7-year-old girl is brought to the physician by her mother because of increasingly severe left knee pain during the past 12 hours. Her temperature is 39.1°C (102.3°F), pulse is 115/min, respirations are 18/min, and blood pressure is 110/60 mm Hg. Physical examination shows a tender, swollen left knee that is unable to be flexed because of pain. Hematologic studies show a leukocyte count of 35,000/mm3 and erythrocyte sedimentation rate of 120 mm/h. Which of the following is the most likely diagnosis? A) Juvenile rheumatoid arthritis B) Lyme disease C) Rheumatic fever D) Septic arthritis E) Viral synovitis

D. Septic arthritis commonly occurs in children. Risk factors include trauma to the joint permitting translocation of cutaneous organisms as well as procedures causing transient bacteremia (eg, dental procedures). The most common organism found on aspiration is Staphylococcus aureus. Septic arthritis classically presents with systemic signs and symptoms of illness such as fever, chills, myalgias, arthralgias, and nausea, along with a painful, erythematous, swollen joint with limited range of motion on examination. Patients may refuse to bear weight. In children, septic arthritis commonly affects the knee and hip. It may occur concomitantly with osteomyelitis of adjacent bone. Serum laboratory values typically disclose leukocytosis, as well as increased erythrocyte sedimentation rate and C-reactive protein concentrations. Synovial fluid aspiration will demonstrate a high concentration of white blood cells and may show organisms on Gram stain with growth on cultures. This patient's presentation is highly consistent with monoarticular septic arthritis. Incorrect Answers: A, B, C, and E. Juvenile rheumatoid arthritis (Choice A) (juvenile idiopathic arthritis) occurs before the age of 16 and classically presents with joint pain and inflammation along with extra-articular findings such as uveitis, lymphadenopathy, and rash. It does not present with signs of sepsis. Lyme disease (Choice B) is transmitted by the bite of the Ixodes scapularis tick and is caused by the pathogen Borrelia burgdorferi. It classically presents with a spreading rash (erythema migrans) at the site of the tick bite. Late-stage manifestations include arthritis, heart block, and cranial nerve palsies. Rheumatic fever (Choice C) is a sequela of untreated Group A Streptococcus infection in which there is an autoimmune reaction against host proteins that ha

90 Exam Section 2: Item 41 of 50 National Board of Medical Examiners' Comprehensive Basic Science Self-Assessment 41. A 45-year-old woman is brought to the emergency department because of a 6-hour history of constant right flank pain, fever, pain with urination, and blood in the urine. Antibiotic treatment was initiated 2 weeks ago for a urinary tract infection caused by Proteus mirabilis. She is writhing on the hospital bed, unable to find a comfortable position. Her temperature is 38.3°C (101°F), pulse is 110/min, respirations are 30/min, and blood pressure is 160/80 mm Hg. Abdominal examination shows no abnormalities except for tenderness of the right flank to palpation. Urinalysis shows 1+ blood and a pH of 7.5. A spiral CT scan of the abdomen shows a staghorn right renal calculus. This renal calculus is most likely composed of which of the following? A) Calcium oxalate B) Calcium phosphate C) Cystine D) Struvi

D. Struvite is the most likely substance implicated in the formation of a staghorn renal calculus, especially in the setting of a recent urinary tract infection with Proteus mirabilis, which is a gram- negative bacterium that produces urease. Urease catalyzes the conversion of urinary urea to ammonia and carbon dioxide, which results in urine alkalinization. The solubility of struvite is decreased under alkaline conditions and precipitates as a crystal. An alkaline urine is present in this patient with a urinary pH of 7.5. Struvite crystals have an orthorhombic configuration under light microscopy. Struvite calculi are radiopaque and can be potentially identified by an x-ray or CT scan. Struvite calculi may adopt a branching morphology that molds itself to the shape of the collecting system (staghorn calculi). Given their large size and often ramified structure, it is rare for struvite calculi to pass through the urinary tract, and surgical removal is often necessary along with treatment of the underlying infection. Incorrect Answers: A, B, C, and E. Calcium oxalate (Choice A) is the most common etiology of nephrolithiasis, which is more frequently seen in patients with malabsorptive syndromes. Citric acid decreases the formation of calcium oxalate crystals by forming soluble complexes with calcium ions, and therefore, calcium oxalate calculi tend to form in the setting of hypocitraturia. Calcium oxalate crystals have an octahedral morphology under light microscopy and form radiopaque calculi. Patients can be treated with thiazide diuretics, a low-sodium diet, and supplementation with potassium citrate. Calcium phosphate (Choice B) stones are found in patients with similar risk factors as those who develop calcium oxalate stones. Most patients have consistently increased urinary pH as a result of distal renal tubular a

93 Exam Section 2: Item 44 of 50 National Board of Medical Examiners' Comprehensive Basic Science Self-Assessment 44. A 34-year-old woman is brought to the emergency department 30 minutes after being involved in a motor vehicle collision. Sensation to pain is decreased over the left lower extremity. Which of the following labeled regions on the diagram of the spinal cord shown is most likely damaged in this patient? BAJ H. C E F Right Left A) B) C) D) E) F) G) H) |) J)

D. The lateral spinothalamic tract carries sensory nerve fibers for pain and temperature. Sensory neurons for pain and temperature synapse in the ipsilateral gray matter in the spinal cord, then the second order neuron decussates in the anterior white commissure of the spinal cord and ascends contralaterally in the lateral spinothalamic tract to synapse in the thalamus. As a result of the immediate decussation in the spinal cord, sensations of pain and temperature from the left side of the body ascend in the lateral spinothalamic tract on the contralateral right side of the spinal cord. Incorrect Answers: A, B, C, E, F, G, H, I, and J. The dorsal column (Choices A, B, I and J) holds ascending sensory nerves transmitting pressure, vibration, proprioception, and fine touch sensation. These sensory neurons ascend ipsilaterally in the dorsal column, so fibers from the right side of the body ascend on the right side of the spinal cord (Choices A and B), whereas fibers from the left side of the body ascend on the left side of the spinal cord (Choices I and J). Sensory nerves from the lower body ascend in the fasciculus gracilis in the medial aspect of the dorsal column (Choices A and J) whereas sensory nerves from the upper body ascend in the fasciculus cuneatus (Choices B and I). Upper motor neurons originating from the motor cortex in the brain descend and decussate at the caudal medulla and descend contralaterally in the lateral corticospinal tract (Choices C and H). Therefore, the lateral corticospinal tract on the right of the spinal cord (Choice C) carries nerves that control the right side of the body, and the left corticospinal tract (Choice H) controls the voluntary movement of the left side of the body. However, these signals originated in the contralateral primary motor cortex of the brain. The anterior corticospi

29 Exam Section 1: Item 30 of 50 National Board of Medical Examiners' Comprehensive Basic Science Self-Assessment 1.0- 30. A study is conducted to measure survival following a diagnosis of lung, colon, or bladder cancer. Data from a statewide cancer registry are used. The graph shows survival plotted against the number of years after diagnosis for each cancer. Which of the following is the most accurate interpretation of these data concerning mortality? 0.9- 0.8- 0.7- 0.6- 0.5- A) 1-Year mortality after a diagnosis of colon cancer is the same as 1-year mortality after a diagnosis of lung cancer B) 3-Year mortality after a diagnosis of lung cancer is lower than 3-year mortality after a diagnosis of bladder cancer C) 3-Year mortality is the same after a diagnosis of lung, bladder, and colon cancer 0.4- D) 5-Year mortality after a diagnosis of bladder cancer is lower than 5-year mortality after a diagnosis of colon 0.3-

D. Survival curves are a useful tool in evaluating the natural history of a disease or the efficacy of treatment. The survival curve in this study plots the number of patients still alive (Y-axis) against time (X-axis). A perfect treatment, in which all patients survived the disease, would be reflected by a horizontal line across the top of the graph, demonstrating a surviving proportion of 1.0 during the entire study duration. Thus, the higher the value on the Y-axis of a survival curve, the greater the chances of survival in that patient group. If patients were to die at a constant rate after the diagnosis of the disease, this would be reflected by a straight line with a constant downward slope. In most cases, however, cancer demonstrates a steep initial decline, followed by a plateau on survival curve analysis. This is because initial disease severity and the success or failure of treatment typically occurs early. After successful treatment, survival rates become more stable for a longer duration of time. This phenomenon is exemplified in the lung cancer survival curve in this study. The bladder cancer group has a proportion surviving of 0.90 at 5 years, meaning that 90% of patients are still alive at 5 years. The colon cancer group has a proportion surviving of 0.65 at 5 years, meaning that 65% of patients are alive at 5 years. This means that the diagnosis of bladder cancer has a lower 5-year mortality rate than does colon cancer. Incorrect Answers: A, B, C, and E. One-year mortality after a diagnosis of colon cancer is approximately 15%, whereas 1-year mortality after lung cancer is approximately 55% on the basis of the survival curves. Therefore, the 1- year mortality rates of these diseases are not the same (Choice A). Three-year mortality after a diagnosis of lung cancer is 70%, whereas the 3-year mortality of

28 Exam Section 1: Item 28 of 50 National Board of Medical Examiners' Comprehensive Basic Science Self-Assessment 28. A 49-year-old man comes to the emergency department because of a persistent and painful erection since he took a drug for erectile dysfunction 4 hours ago. His temperature is 37°C (98.6°F), and blood pressure is 110/65 mm Hg. Physical examination shows flushing of the skin and priapism. The most likely cause of this patient's condition is a lack of input from which of the following neural pathways? A) General sensory fibers from the pudendal nerve B) Parasympathetic fibers from the pelvic plexus C) Somatic motor fibers from the pudendal nerve D) Sympathetic fibers from the prostatic plexus E) Visceral afferent fibers to the pelvic plexus

D. Sympathetic fibers from the prostatic plexus are involved in detumescence of the penile erectile tissue, signifying that a lack of input from these fibers can lead to priapism. From the prostatic plexus, the cavernous and pudendal nerves both carry sympathetic fibers that release norepinephrine to extinguish penile erection. Norepinephrine vasoconstricts the helicine arteries of the corpora cavernosa and thereby abolishes the parasympathetic nervous system-mediated vasodilation that produces erections. Erectile dysfunction medications such as sildenafil are phosphodiesterase inhibitors. Phosphodiesterase inhibition in the case of penile blood flow increases cyclic guanosine monophosphate (CGMP) concentrations, which relaxes the smooth muscle of the vasculature through nitric oxide signaling. This signal occurs downstream from the G protein-coupled receptors at the vascular smooth muscle membrane; therefore, erectile dysfunction medications can override sympathetic nervous system-mediated vasoconstriction. Incorrect Answers: A, B, C, and E. General sensory fibers from the pudendal nerve (Choice A) are activated by touch or pressure on the penile skin, glans, and urethra. Somatic motor fibers from the pudendal nerve (Choice C) respond to this sensory input by causing contraction of the ischiocavernosus and bulbocavernosus muscles, leading to erection. Increased pudendal nerve activity, rather than a lack of activity, promotes erections. Parasympathetic fibers from the pelvic plexus (Choice B) contribute to the cavernous nerves, which travel to the corpora cavernosa of the penis and release acetylcholine that causes vasodilation of the helicine arteries, resulting in penile erection. Increased activity of the parasympathetic fibers of the cavernous nerves promotes erections. Visceral afferent fibers to the pelvic plexu

21 Exam Section 1: Item 21 of 50 National Board of Medical Examiners' Comprehensive Basic Science Self-Assessment 21. A 48-year-old man with renal artery stenosis undergoes stent placement. Femoral access is used to place the stent. After entrance into the aorta, the guide wire should be advanced superiorly just beyond which of the following structures to approach the right renal artery in this patient? A) Celiac trunk B) Internal iliac artery C) Superior mesenteric artery D) Testicular artery

D. The abdominal aorta has numerous branches that arise to supply the entirety of the gastrointestinal system, spleen, kidneys, spine, and testicles before dividing into the left and right iliac arteries. The testicular arteries are paired and arise just inferior to the takeoff of the renal arteries at the level of the second lumbar vertebrae, before they pass into the retroperitoneum and inferiorly through the deep inguinal ring to supply the testes. In this patient, the guidewire should be advanced just superior to the testicular artery orifices, placing it at the level of the renal arteries at approximately L1-2, to access the right renal artery. Incorrect Answers: A, B, and C. Celiac trunk (Choice A) gives rise to the common hepatic artery, left gastric artery, and splenic artery at the level of T12. It supplies blood to the liver, stomach, proximal duodenum, pancreas, inferior esophagus, and spleen. It lies above the level of the renal arteries. Internal iliac arteries (Choice B) are paired vessels that arise at the level of L5-S1 and are branches of the common iliac arteries. The internal iliac arteries supply blood to the buttocks and the pelvic organs, including the bladder and inferior part of the ureters, but do not supply blood to the kidneys. They lie below the level of the renal arteries. Superior mesenteric artery (Choice C) is a branch of the abdominal aorta that arises just inferior to the celiac trunk and supplies blood to the duodenum, jejunum, ileum, and proximal two-thirds of the colon. It arises just above the level of the renal arteries. Educational Objective: To access the renal arteries from the femoral artery approach, the catheter should be advanced to the area just superior to the takeoff of the testicular artery. %D Previous Next Score Report Lab Values Calculator Help Pause

33 Exam Section 1: Item 34 of 50 National Board of Medical Examiners' Comprehensive Basic Science Self-Assessment 34. A 12-year-old girl is brought to the physician by her parents because of a 2-day history of dizziness and headache. She is intelligent and does well in school. Her mother had a cerebellar tumor removed during childhood and is concerned that her daughter might have a similar condition. A normal reaction to which of the following physical examination maneuvers is most likely to rule out a cerebellar tumor in this patient? A) Babinski sign B) Doll's eye (oculocephalic) maneuver C) Glabellar tap D) Rapid alternating movements E) Tinel sign

D. The inability to perform rapid alternating movements, also known as dysdiadochokinesia, indicates cerebellar dysfunction. Rapid alternating movements involve quickly alternating between clapping the palms and the dorsum of one hand. The cerebellum mediates motor learning and calculates movement course changes in response to sensory input. Therefore, cerebellar dysfunction results in a decreased ability to alternate between opposite movements as in the rapid alternating movements physical examination maneuver. This patient may have a pilocytic astrocytoma or medulloblastoma, two of the most common cerebellar tumors in children. Cerebellar tumors may present with dizziness and symptoms of increased intracranial pressure (as a result of compression of the fourth ventricle by the tumor and consequent obstructive hydrocephalus) such as headache as well as gait ataxia and nystagmus (for midline tumors) or dysmetria on finger-to-nose testing and dysdiadochokinesia (for lateral tumors). Incorrect Answers: A, B, C, and E. The Babinski sign (Choice A), or extensor plantar response, is an upper motor neuron sign that indicates damage to the corticospinal tract, a descending motor tract located in the central nervous system. The Babinski sign is present when the lateral part of the plantar surface of the foot is firmly stroked and the hallux extends (moves rostrally) while the other toes abduct. The cerebellum does not contain the corticospinal tract, so cerebellar dysfunction would not cause the Babinski sign. The doll's eye (oculocephalic) maneuver (Choice B) refers to the brainstem reflex that occurs in normal adults when the examiner moves the patient's head and the eyes move in the opposite direction to hold the patient's gaze on an object. In a negative oculocephalic reflex, the patient's eyes remain midline, which indica

5 Exam Section 1: Item 5 of 50 National Board of Medical Examiners' Comprehensive Basic Science Self-Assessment 5. A 26-year-old woman is brought to the emergency department by her husband after she ingested an overdose of narcotics. She is arousable only with deep painful stimuli. Her pulse is 110/min, respirations are 10/min, and blood pressure is 110/70 mm Hg. Cerebral blood flow is increased. Which of the following is the most likely cause of the increase in cerebral blood flow? A) Decreased blood pressure B) Decreased hydrogen ion concentration in the blood C) Decreased oxygen concentration in the blood D) Increased carbon dioxide concentration in the blood E) Increased heart rate

D. The intracranial compartment is fixed, surrounded by a rigid skull containing brain parenchyma, cerebrospinal fluid, and intravascular blood. Serum carbon dioxide concentration directly increases or decreases cerebral blood flow (CBF). Hypocapnia induces cerebral vasoconstriction, whereas hypercapnia induces cerebral vasodilation, increasing the amount of CBF and the volume of intracranial blood. In a patient with decreased respirations secondary to narcotic-induced respiratory depression, decreased air exchange and ventilation will lead to an increased amount of carbon dioxide in the blood. The resulting hypercapnia leads to vasodilation and increased CBF. Incorrect Answers: A, B, C, and E. Decreased blood pressure (Choice A) would not increase CBF. Mean arterial pressure (MAP) can have an effect on CBF, but as a result of cerebral autoregulation, CBF is maintained at a constant state over a wide range of MAP. Small changes or decreases in blood pressure have no effect on CBF; only blood pressure below or above the autoregulatory range will affect CBF. Decreased hydrogen ion concentration in the blood (Choice B) is the opposite of what would occur with opioid-induced respiratory depression, which leads to hypercapnia and increased hydrogen ion concentration. Decreased oxygen concentration in the blood (Choice C) results from hypoventilation, but CBF is more directly affected by concentrations of carbon dioxide. Increased heart rate (Choice E), just like blood pressure and MAP, has little effect on CBF provided that MAP remains within the autoregulatory range. Extreme bradycardia or tachycardia leading to severe hypotension could decrease CBF. Educational Objective: As a result of cerebral autoregulation, changes in MAP have minimal impact on CBF. Serum carbon dioxide concentration elicits a reversible effect on CBF

82 Exam Section 2: Item 33 of 50 National Board of Medical Examiners' Comprehensive Basic Science Self-Assessment 33. A 34-year-old woman comes to the office because of a 6-month history of progressive swelling of her hands and feet. Her blood pressure is 140/89 mm Hg. Physical examination shows 1+ edema of the hands and 2+ edema of the feet. Serum studies show a potassium concentration of 3.3 mEq/L, decreased aldosterone concentration and plasma renin activity, and increased cortisol:cortisone ratio. After questioning, it is found that the patient has been consuming a nutritional supplement she obtained from the Internet during the past 10 months. Serum studies obtained after the patient stops using the supplement are within the reference ranges. The supplement this patient consumed most likely inhibited the activity of which of the following enzymes? O A) A4-5a-Dehydrogenase B) 1a-Hydroxylase C) 21-Hydroxylase D) 1

D. The patient likely consumed a supplement containing licorice, the active ingredient of which is glycyrrhetinic acid, or enoxolone. This compound inhibits a variety of enzymes, including the enzyme 11B-hydroxysteroid dehydrogenase type lI, which catalyzes the conversion of cortisol to cortisone in the zona fasciculata of the adrenal gland. Inhibition of this reaction causes an increased cortisol to cortisone ratio. Cortisol demonstrates a weak mineralocorticoid effect, which becomes more pronounced as the concentration of cortisol increases. It inserts ENAC channels into the renal collecting tubule, which cause sodium and water reabsorption in exchange for potassium excretion, leading to hypertension and a mild hypokalemia. Secondary to this hypertension, the juxtaglomerular apparatus decreases its production of renin and subsequently decreases the production of angiotensin and aldosterone. This accounts for the decreased plasma renin and aldosterone concentrations seen in this patient. The enzyme also inhibits the metabolism of prostaglandins E2 and F2g to their respective inactive metabolites, which causes a downstream effect of inhibiting gastric acid secretion by the stomach. Incorrect Answers: A, B, C, and E. 4-5a-Dehydrogenase (Choice A) is a subtype of 5a-reductase and participates in the conversion of testosterone to dihydrotestosterone. This enzyme is inhibited by finasteride and does not account for the changes in glucocorticoid metabolism or renin-angiotensin-aldosterone pathway seen in this patient. 1a-Hydroxylase (Choice B) catalyzes the reaction converting 25-OH vitamin D3 to its active form, 1,25-(OH)2 vitamin D3 (calcitriol). This enzyme resides in the kidney and is not involved in glucocorticoid metabolism. 21-Hydroxylase deficiency (Choice C) is the cause of the most common form of congenital adrena

23 Exam Section 1: Item 23 of 50 National Board of Medical Examiners' Comprehensive Basic Science Self-Assessment 23. A 51-year-old man has a cardiac output of 5 L/min and a right atrial pressure of 0 mm Hg at rest. The solid curve in the graph shows the relationship between right atrial pressure and cardiac output in a normal heart. Which of the following lettered points best represents the change in this relationship 5 seconds after a large arteriovenous fistula opens? 15- 10 AO PE Boo¯OF 0- -4 -2 0 2 4 6. 8 10 Right atrial pressure (mm Hg) A) B) C) D) E) F)

D. The sudden surge of arterial blood into venous return that results from a large arteriovenous fistula (AVF) opening would result in increased right atrial pressure and increased cardiac output caused by the increase in preload. The AVF results in an abnormal communication between an artery and vein, leading to low-resistance, high-volume flow of blood between the artery and vein. An AVF decreases systemic vascular resistance through the creation of a shunt to the high-capacitance, low-pressure venous system. The increase in preload causes a greater distension in the cardiomyocyte fibers at the end of diastole, which results in increased cardiac contractility per the Frank-Starling relationship. This is represented by point D on the cardiac function curve in this patient with a normal heart that can respond to changes in preload and generate the increased contractile force. Patients with an AVF present with a palpable thrill, diminished distal pulses, and a continuous machine-like murmur at the site of the fistula. Over time, this can lead to the development of high-output cardiac failure. The size of the AVF is most predictive of the risk for developing heart failure, as a larger AVF results in higher flow between the artery and vein, requiring the heart to compensate with greater output to supply the remainder of the systemic circulation. Incorrect Answers: A, B, C, E, and F. Point B on the cardiac function curve represents the cardiac output relative to a decrease in preload from the patient's heart at rest. This may result from positive intrathoracic pressure during expiration (which decreases venous return to the right atrium), hypovolemia, or exposure to a venous vasodilator. Point A represents a shift upward from this state of decreased preload onto a new curve as a result of enhanced cardiac performance, whic

91 Exam Section 2: Item 42 of 50 National Board of Medical Examiners' Comprehensive Basic Science Self-Assessment 42. A 1-year-old girl is admitted to the hospital because of failure to thrive. She has had frequent, loose stools for 6 months. She has not gained weight satisfactorily during this period. She is at the 3rd percentile for length and weight. Physical examination shows facial edema. Laboratory studies show hypoalbuminemia. Examination of the contents of the duodenum postprandially show: Trypsinogen Chymotrypsinogen Proelastase increased increased increased Lipase Amylase normal normal A deficiency of which of the following enzymes is the most likely cause of these findings? A) Aminopeptidase B) Carboxypeptidase A OC) Chymotrypsin D) Enteropeptidase E) Trypsin

D. This infant's findings of failure to thrive, diarrhea, edema, hypoalbuminemia, and increased concentrations of luminal proenzymes are suggestive of enteropeptidase deficiency. This rare disorder is caused by an inactivating mutation in the gene encoding enteropeptidase. In the absence of functional enteropeptidase, pancreatic trypsinogen fails to be cleaved to active trypsin, and therefore fails to catalyze the conversion of other pancreatic proenzymes, such as chymotrypsinogen and proelastase, to their active forms. The normal activity of pancreatic enzymes, including lipase and amylase, suggests that the quantity of pancreatic secretions is adequate and that the patient's defect is rather one of luminal enzymatic conversion of proenzymes to their active form. Failure to digest proteins and fats leads to osmotic and malabsorptive diarrhea and failure to thrive during infancy. Treatment is with oral replacement of enteropeptidase. Incorrect Answers: A, B, C, and E. Aminopeptidase (Choice A) is important for the digestion of dietary peptides at the brush border. However, deficiency of this enzyme would not account for the increased concentrations of several proenzymes that are observed in this patient. Carboxypeptidase A (Choice B) is a pancreatic enzyme that is important for the digestion of dietary peptides. However, deficiency of this enzyme would not account for the increased concentrations of several proenzymes that are observed in this patient. Chymotrypsin (Choice C) is produced as the proenzyme chymotrypsinogen by pancreatic acinar cells and is converted to its active form by trypsin. A deficiency of chymotrypsin would not account for the increased concentrations of other proenzymes, such as proelastase and trypsinogen, which is suggestive of deficiency of enteropeptidase. Similarly, trypsin (Choice E) is pro

80 Exam Section 2: Item 31 of 50 National Board of Medical Examiners' Comprehensive Basic Science Self-Assessment 31. A 10-year-old girl is brought to the physician by her mother because of increased hair growth on her face and increased muscle mass during the past 3 months. Breast development is Tanner stage 1, and pubic hair development is Tanner stage 5. Physical examination shows clitorimegaly. Pelvic examination shows a normal-appearing vagina. Serum studies show: Luteinizing hormone Dehydroepiandrosterone Dehydroepiandrosterone sulfate Testosterone 0.8 mlU/mL (N=1.0-5.9) 6.2 nmol/L (N=5.2-19.8) 0.9 umol/L (N=0.9-3.4) 14 nmol/L (N=0.2-0.7) An unregulated increase in hormone production by which of the following types of cells is the most likely cause of this patient's hirsutism? A) Adrenal zona fasciculata cells B) Adrenal zona glomerulosa cells C) Ovarian follicle cells D) Ovarian Sertoli-Leydig cells E) Pituita

D. This patient is presenting with clinical signs and symptoms of virilization, the condition in which females develop male features a a result of increased concentrations of androgens. These features include hirsutism (the presence of unwanted hair in a male distribution), increased muscle mass, pubic hair, and clitoromegaly and are confirmed by her increased testosterone concentration. Testosterone may be increased in the setting of a hormone-secreting ovarian tumor, polycystic ovarian syndrome, hypersecretion of androgens by the adrenal gland, or an adrenal tumor. An increased testosterone concentration causes negative feedback on the pituitary gland, thereby lowering the luteinizing hormone (LH) concentration. When evaluating virilization and hirsutism, it is critical to measure the concentrations of testosterone, dehydroepiandrosterone (DHEA), and dehydroepiandrosterone sulfate (DHEA-S); the latter two are made by the adrenal gland and help to narrow the differential diagnosis. In this case, because the testosterone is increased while DHEA and DHEA-S are within normal limits, the culprit is likely of an ovarian source. À Sertoli-Leydig cell tumor is a rare androgen-producing, ovarian sex-cord stromal tumor. It is a neoplastic proliferation of the cells that give rise to the specialized gonadal stroma surrounding the granulosa and theca cells in women and the Sertoli and Leydig cells in men. These proliferating cells have the capacity for differentiation into many cell types, including Sertoli-Leydig cells, which secrete testosterone. As in this case, the presence of a Sertoli-Leydig cell tumor may cause precocious puberty in boys or virilization in girls. Incorrect Answers: A, B, C, E, and F Though not in this case, virilization may also be caused by the increased adrenal secretion of androgens, which are made in

31 Exam Section 1: Item 32 of 50 National Board of Medical Examiners' Comprehensive Basic Science Self-Assessment 32. An autopsy is done on a 50-year-old man who died of pneumonia despite 5 days of antibiotic therapy in the intensive care unit. He had a 15-year history of alcoholism. A photograph of a sagittal section of the brain is shown. Based on this pathology, which of the following findings was most likely present on neurologic examination of the patient prior to his death? A) Dysdiadochokinesia B) Dysmetria on finger-nose testing C) Essential tremor D) Gait ataxia E) Present Romberg sign

D. This patient's cerebellar vermis (pictured in cross section) appears atrophic, which could present with gait ataxia. The cerebellar vermis is in the medial zone of the cerebellum (smaller in cross- sectional surface area than the cerebellar hemisphere behind the vermis). The cerebellar vermis controls the motor function of the axial and proximal limb muscles and mediates body posture and balance. Lesions of the cerebellar vermis lead to severe gait ataxia (from truncal instability), titubation (bobbing of the head or axial body), and nystagmus. The cerebellar hemispheres control ipsilateral motor functions of the limb muscles (generally distal compared with muscles controlled by the vermis). Dysfunction of the cerebellar hemispheres leads to dysdiadochokinesia, dysmetria on finger-nose testing, intention tremor, and gait ataxia (from lower extremity instability). Alcohol has direct toxic effects on the dendrites of cerebellar neurons, which may mediate cerebellar atrophy from chronic alcohol use. Cerebellar damage can persist after abstinence from drinking. Incorrect Answers: A, B, C, and E. Dysdiadochokinesia (Choice A) and dysmetria on finger-nose testing (Choice B) occur when the cerebellar hemispheres, as opposed to the cerebellar vermis, are damaged. Cerebellar hemisphere damage also causes an intention tremor and gait ataxia. Essential tremor (Choice C) is a common adult action tremor that can present as a bilateral intention tremor of the upper extremities and is commonly heritable. Essential tremor has been associated with cerebellar lesions in various locations but is not specifically correlated with lesions of the cerebellar vermis. A present Romberg sign (Choice E) signifies that a patient maintains station without wobble or sway when the eyes are open but sways or falls when the eyes are closed. A presen

44 Exam Section 1: Item 45 of 50 National Board of Medical Examiners' Comprehensive Basic Science Self-Assessment 45. A 30-year-old man is brought to the physician by his wife because of a 1-day history of irrational behavior and severe abdominal pain. He began treatment with trimethoprim- sulfamethoxazole for a urinary tract infection 3 days ago. His urine turned a light burgundy color during a similar episode 1 year ago. His mother and his maternal grandfather have had similar symptoms. He appears anxious and restless. His pulse is 96/min. Physical examination shows diaphoresis. Serum studies show increased concentrations of 5-aminolevulinic acid (0-ALA) and porphobilinogen. The physician suspects that a mutant allele is causing decreased activity of an enzyme involved in heme biosynthesis. This enzyme is most likely which of the following? A) õ-ALA dehydratase B) 0-ALA synthase C) Ferrochelatase D) Porphobilinoge

D. This patient's presenting signs and symptoms, including psychosis, abdominal pain, and burgundy-colored urine following medication exposure is consistent with a diagnosis of acute intermittent porphyria (AIP). AIP is an autosomal dominant disorder caused by inactivating mutations of the enzyme porphobilinogen deaminase, resulting in deficient synthesis of heme and accumulation of porphobilinogen. Porphobilinogen has a neurotoxic effect, leading to symptoms of abdominal pain, psychosis, and peripheral polyneuropathy. In contrast to other porphyria disorders, cutaneous photosensitivity and blistering is not observed in patients with AlP. Acute exacerbations of AIP are commonly precipitated by metabolic stressors such as starvation, alcohol ingestion, or ingestion of medications. Commonly offending medications include sulfonamides (including trimethoprim-sulfamethoxazole), barbiturates, oral contraceptives, and rifampin. Treatment includes discontinuation of the offending agent as well as intravenous infusion of glucose and heme, which inhibit the upstream enzyme ALA synthase, thereby inhibiting synthesis of porphobilinogen. Incorrect Answers: A, B, C, and E. Ö-ALA dehydratase (Choice A) catalyzes the conversion of ō-aminolevulinic acid to porphobilinogen. Defects in this enzyme cause a rare form of porphyria that may resemble AIP but which would not display increased serum concentrations of porphobilinogen. Ō-ALA synthase (Choice B) catalyzes the conversion of succinyl-CoA and glycine to õ-aminolevulinic acid. Defects in this enzyme result in X-linked sideroblastic anemia. Ferrochelatase (Choice C) catalyzes insertion of ferrous iron into protoporphyrin to form heme. Ferrochelatase is inhibited by lead poisoning. Defects in ferrochelatase can cause erythropoietic protoporphyria. However, photosensitivity is usuall

27 Exam Section 1: Item 27 of 50 National Board of Medical Examiners' Comprehensive Basic Science Self-Assessment 27. A 56-year-old woman with type 2 diabetes mellitus is being treated with pioglitazone to improve her sensitivity to insulin and with gemfibrozil to decrease her serum triglyceride concentration. These drugs both initiate their therapeutic effects by binding to which of the following receptor groups? A) B-Adrenergic receptors B) Farnesoid X receptors C) Insulin-like growth factor receptors D) Peroxisome proliferator-activated receptors E) Toll-like receptors

D. Type 2 diabetes mellitus is a common disorder in resource-rich nations. Initial treatment consists of diet and activity modification, weight loss, and metformin monotherapy. If these measures fail to achieve adequate blood glucose control, additional oral antihyperglycemics may be considered. Common options for second-line pharmacotherapy include sodium-glucose co-transporter 2 (SGLT2) inhibitors, glucagon-like peptide-1 (GLP-1) analogs, dipeptidyl peptidase-4 (DPP-4) inhibitors, thiazolidinediones, and sulfonylureas. Pioglitazone (a thiazolidinedione) achieves its effect by binding to peroxisome proliferator-activated receptors (PPARS), which are expressed in most cell types including adipose cells, pancreatic cells, myocytes, and hepatocytes. PPARS are nuclear receptors that act as transcription factors to regulate metabolism. Activation results in increased cell sensitivity to insulin and adiponectin. Insulin has multiple effects, including increased glycogen, triglyceride, and protein synthesis and storage, as well as glucose uptake by adipose tissue and myocytes. Adiponectin is secreted by adipocytes and helps regulate fatty acid and glucose metabolism. Gemfibrozil is a fibrate lipid-lowering agent that also activates PPARS and upregulates lipoprotein lipase, resulting in decreased serum LDL, increased HDL, and decreased triglyceride concentrations. Incorrect Answers: A, B, C, and E. B-Adrenergic receptors (Choice A) are ubiquitous in the body. B-Adrenergic receptor activation in adipocytes results in increased lipolysis, in pancreatic cells results in increased insulin release, and in hepatocytes stimulates gluconeogenesis and glycogenolysis. Farnesoid X receptors (Choice B) are nuclear receptors expressed in hepatocytes and intestinal cells. Activation results in inhibition of cholesterol 7-a-hydroxylase whic

69 Exam Section 2: Item 20 of 50 National Board of Medical Examiners' Comprehensive Basic Science Self-Assessment 20. A 5-year-old boy who had intrauterine growth restriction has continued to grow slowly. Psychomotor development is normal. His parents are of average stature. Genetic studies show that he has uniparental maternal heterodisomy for chromosome 7. Which of the following mechanisms best explains his slow growth? A) Expansion of an unstable trinucleotide repeat B) Expression of an autosomal dominant disorder C) Expression of an autosomal recessive disorder D) Genomic imprinting of growth genes E) Somatic mosaicism

D. Uniparental heterodisomy is the inheritance of a pair of nonidentical chromosomes from a single parent that results from an error during meiosis I. In contrast, uniparental isodisomy is the inheritance of a pair of identical chromosomes caused by errors during meiosis Il or postzygotic duplication of the chromosome. Uniparental maternal heterodisomy for chromosome 7 is associated with intrauterine growth restriction and short stature without impairment of psychomotor development. This is caused by genomic imprinting of growth genes. Genomic imprinting describes epigenetic modification in which one gene copy is silenced by methylation depending on the parent-of-origin. Genomic imprinting may be complete or partial, with the methylated allele suppressed rather than silenced. Other disorders resulting from uniparental disomy include Angelman syndrome and Prader-Willi syndrome. Incorrect Answers: A, B, C, and E. Expansion of an unstable trinucleotide repeat (Choice A) is the mechanism of disorders such as Huntington disease, myotonic dystrophy, fragile X syndrome, and Friedreich ataxia. These disorders may display anticipation, the phenomenon in which a genetic condition tends to increase in severity or present at an earlier age as it passes through multiple generations. Expression of an autosomal dominant disorder (Choice B) occurs when the disease phenotype is expressed with only a single copy of the defective allele (heterozygous) or two copies of the defective allele (homozygous). Autosomal dominant genetics follow Mendelian patterns of inheritance. Expression of an autosomal recessive disorder (Choice C) occurs when the disease phenotype is expressed only if two copies of the defective allele are present. Autosomal recessive genetics follow Mendelian patterns of inheritance. Somatic mosaicism (Choice E) occurs when

34 Exam Section 1: Item 35 of 50 National Board of Medical Examiners' Comprehensive Basic Science Self-Assessment 35. A 40-year-old man comes to the physician for a routine examination. He has smoked 2 packs of cigarettes daily for 20 years. Which of the following initial statements by the physician is likely to be most effective in encouraging this patient to stop smoking? A) "Do you have any relatives who died of lung cancer?" B) "Let me show you a picture of what lungs look like after a lifetime of smoking." C) "Please tell me how I can help you to stop smoking." D) "Smoking causes lung cancer and many other illnesses." E) "What thoughts have you had about quitting smoking?" F) "Why do you continue to smoke when it is so unhealthy?"

E. Asking an open-ended question about the patient's thoughts about quitting smoking is the best way to engage and motivate the patient and can also assess the patient's readiness for behavioral change. The stages of behavioral change are used to define a patient's readiness to change a health-related behavior such as smoking. Physicians aim to move patients through these stages over time with an interview technique called motivational interviewing. For instance, physicians can utilize motivational interviewing to move a patient who is precontemplative, or not interested in changing, to the contemplative stage, or indecisive about changing. Motivational interviewing involves using open-ended, nonjudgmental questions to help the patient explore their reasons for wanting to change or maintain the habit. Motivational interviewing is a collaborative, nondirective process that capitalizes on the patient's autonomy rather than striking a paternalistic tone. Physicians should refrain from providing advice or education initially, as this advice is unlikely to be effective (especially for patients who are in the precontemplative stage). Incorrect Answers: A, B, C, D, and F. Educating the patient about lung cancer or other health concerns related to smoking (Choices A, B, D, and F) strikes a tone that is too directive and may make this patient feel defensive about his smoking habit. Questioning why the patient continues to smoke despite the health risks (Choice F) would likely make him feel judged. An effective discussion is more likely to result from an open- ended question about the patient's thoughts about his habit. Education about the health risks of smoking may be appropriate later in the discussion after the physician has established that the patient has surpassed the precontemplative stage and allowed the patient to expl

10 Exam Section 1: Item 10 of 50 National Board of Medical Examiners' Comprehensive Basic Science Self-Assessment 10. A 28-year-old man comes to the physician for a routine health maintenance examination. He is from Jamaica. He has practiced Rastafarianism all his life and follows a diet consisting only of natural juices, fruits, and grains. He also regularly smokes marijuana as part of his religion. Physical examination shows no abnormalities. His hematocrit is 32%, and serum vitamin B12 (cobalamin) concentration is within the reference range. Which of the following best describes the erythrocytes in this patient? Color Size (in Diameter) Shape A) Normal small normal B) Normal small 10% sickle, 90% normal C) Normal large normal D) Normal large 10% sickle, 90% normal E) Pale small normal F) Pale small 10% sickle, 90% normal G) Pale large normal H) Pale large 10% sickle, 90% normal

E. Choice E corresponds with a diagnosis of iron deficiency anemia (IDA). Erythrocytes in the setting of IDA are pale (hypochromic) and small in size (microcytic) but are normal in shape. Iron is required for the synthesis of heme, which is a necessary component of the hemoglobin molecule and, thus, of erythrocytes. It functions to shuttle oxygen to and from peripheral tissues. In individuals who do not have an adequate intake of dietary iron in the form of heme obtained from animal meat, deficiency can develop. IDA may also develop as the result of chronic blood loss from menstruation, colorectal bleeding, or can occur as a result of malabsorption syndromes such as celiac disease and in patients who have undergone gastric bypass surgery. Erythrocytes on the peripheral blood smear are hypochromic as a result of deficient hemoglobin concentration. It is thought that erythrocytes are microcytic as a result of continuing erythrocyte division in order to reach an adequate hemoglobin concentration; because hemoglobin stores are inadequate, cell division continues beyond what would normally occur and causes the cells to be smaller than normal. Treatment in the case of poor dietary intake is with oral iron supplementation. If severe, intravenous iron can be given. Incorrect Answers: A, B, C, D, F, G, and H. Normal color, small size, and normal shape (Choice A) describes several other causes of microcytosis without hypochromia, including thalassemia and anemia of chronic disease. While thalassemia may present with target cells on peripheral smear, the majority of erythrocytes will appear as microcytic. The degree of hypochromia is dependent on the severity of decreased hemoglobin concentration and mean corpuscular volume. Normal color and small size with 10% sickle and 90% normal shape (Choice B) can be seen in patients with s

98 Exam Section 2: Item 49 of 50 National Board of Medical Examiners' Comprehensive Basic Science Self-Assessment 49. A 58-year-old man with chronic pancreatitis comes to the physician because of a 3-week history of greasy, foul-smelling stools. The stools are difficult to flush. There is no associated blood or mucus. He is 178 cm (5 ft 10 in) tall and weighs 65 kg (143 Ib); BMI is 21 kg/m2. Vital signs are normal. Physical examination shows no abnormalities; there is no jaundice. Which of the following recommendations is most appropriate for this patient? A) B vitamin supplementation B) Decreased carbohydrate intake C) Decreased protein intake D) Decreased sodium intake E) Fat-soluble vitamin supplementation F) Iron supplementation

E. Chronic pancreatitis is a progressive disease process caused by a response to recurrent pancreatic injury that results in pancreatic fibrosis, regeneration, and loss of acinar cells, islet cells, and ducts. In addition to chronic abdominal pain as a result of inflammation, patients may develop pancreatic insufficiency as a result of decreased pancreatic secretions. Absence of pancreatic lipase results in steatorrhea and an inability to absorb fat-soluble vitamins including vitamins A, D, E, and K. Patients may also develop diabetes mellitus as a result of decreased pancreatic insulin production. Patients with chronic pancreatitis can be treated with pancreatic enzyme repletion and with supplementation of fat-soluble vitamins. Incorrect Answers: A, B, C, D, and F. B vitamin supplementation (Choice A) may include supplementation of vitamin B1 (thiamine), B6 (pyridoxine) or B12 (cobalamin) and is useful for the treatment of various disorders such as Wernicke encephalopathy or sideroblastic anemia, or in patients being treated with isoniazid. While pancreatic insufficiency may lead to deficiency of vitamin B12 (cobalamin), it does not affect the absorption of most B vitamins, which are water-soluble. Decreased carbohydrate intake (Choice B) may be useful for patients with diabetes mellitus. While diabetes mellitus may develop in the setting of chronic pancreatitis, the patient's symptoms of steatorrhea are more concerning for the development of fat-soluble vitamin deficiencies. Decreased protein intake (Choice C) may be useful in the treatment of patients with uric acid disorders such as gout or ornithine transcarbamylase deficiency. Decreasing protein intake does not address the patient's steatorrhea and subsequent inability to absorb fat-soluble vitamins. Decreased sodium intake (Choice D) may be useful in the treatme

51 Exam Section 2: Item 2 of 50 National Board of Medical Examiners' Comprehensive Basic Science Self-Assessment 2. A female newborn delivered at 26 weeks' gestation is found to have hyaline membrane disease. She is intubated and mechanically ventilated. Her vital signs are continuously monitored, and inspired oxygen is maintained at a level that will sustain an oxygen saturation between 92% and 95%. The primary goal of this treatment is protection of which of the following structures? A) Basal ganglia B) Choroid plexus C) Cochlea D) Ductus arteriosus E) Retina

E. Fetal retinal vascularization is not complete until term. Fetal retinal blood vessels continue to grow outward from the optic nerve and posterior pole towards the ora serrata throughout the third trimester in a process that is dependent on a chemical gradient of vascular endothelial growth factor (VEGF). Premature birth may interrupt this process, especially in the setting of high concentrations of supplemental oxygen, which are often used to treat neonatal respiratory distress syndrome. High concentrations of supplemental oxygen may result in the abolition of the VEGF chemical gradient that promotes normal retinal vascularization, and may be directly toxic to capillaries through increased formation of free radicals. Abnormal retinal vascularization leads to avascularity of the peripheral retina with subsequent abnormal neovascularization, a condition known as retinopathy of prematurity (ROP). Neovascular vessels in the peripheral retina may cause hemorrhage and induce traction that results in retinal detachment. Proper regulation of fetal supplemental oxygen administration helps to prevent the development of ROP. Neonates who are born prematurely (less than 30 weeks gestational age) or with a low birth weight (less than 1500 g) should undergo dilated examination to screen for ROP. Neonates who develop ROP can be treated with retinal laser photocoagulation to decrease the production of VEGF and prevent further complications such as retinal detachment and blindness. Incorrect Answers: A, B, C, and D. Basal ganglia (Choice A) developmental abnormalities may occur in premature infants, but their outcome is not directly affected by neonatal hyperoxygenation. Deposition of bilirubin in the brain (kernicterus) is a known pathology in premature neonates. The choroid plexus (Choice B) lies within the cerebral ventricles and

48 Exam Section 1: Item 49 of 50 National Board of Medical Examiners' Comprehensive Basic Science Self-Assessment 49. A9-year-old girl is brought to the clinic by her mother because of areas of hair loss on her scalp that the mother first noticed 1 week ago. The patient otherwise has been well. There is no personal or family history of serious illness. She receives no medications. The patient transferred from her neighborhood elementary school to a private school for gifted children 3 months ago. The mother says that no one in the house has used new shampoos, soaps, or detergents recently. The patient's growth and development have been normal. The child appears healthy. Vital signs are within normal limits. Examination of the scalp shows two irregularly shaped, quarter-sized areas of alopecia on the right side of the head. Short, broken hairs are noted within these areas, but there is no scaling or erythema. Which of

E. Focal, irregular patches of hair loss with broken hairs is consistent with trichotillomania. Trichotillomania is a psychiatric impulse control disorder that presents with repetitive hair pulling resulting in noticeable hair loss and several unsuccessful attempts to stop hair pulling. Patients also commonly experience tension prior to hair pulling and relief after hair pulling. Stressors can exacerbate hair pulling behavior. Physical examination typically demonstrates bizarrely shaped patches of hair loss on the scalp or loss of eyelashes or eyebrows without significant underlying skin changes. Broken hairs are usually different lengths. Trichotillomania is managed with habit reversal therapy, which helps patients identify triggers for hair pulling and develop alternative ways to resolve the tension that precedes hair pulling behavior. Incorrect Answers: A, B, C, and D. Alopecia areata (Choice A) is a chronic, immune-mediated disorder of hair loss that can present with diffuse or focal hair loss. Areas may demonstrate smooth or irregular borders. Hairs are typically narrower proximally than distally and are prone to breaking. The broken hairs are usually the same length, while in trichotillomania, the broken hairs are different lengths. In this child with a known stressor, trichotillomania is more likely. Hypothyroidism (Choice B) commonly presents with diffuse alopecia instead of the focal alopecia of trichotillomania. Thyroid hormone helps maintain hair follicles. Menkes disease (Choice C) is a congenital X-linked disorder that causes a defect in intestinal copper absorption, leading to copper deficiency. The disease presents with progressive neurologic symptoms, hypopigmentation of the skin, bony abnormalities, and kinked hair instead of alopecia. Further, most children with Menkes disease have a life expectancy o

63 Exam Section 2: Item 14 of 50 National Board of Medical Examiners' Comprehensive Basic Science Self-Assessment 14. A 69-year-old woman is brought to the emergency department because of confusion and light-headedness for 45 minutes. She has a 5-year history of type 2 diabetes mellitus treated with glipizide. Recently, the dose of glipizide was increased. Her vital signs are normal. Physical examination shows no abnormalities. Her blood glucose concentration is 43 mg/dL. Intravenous infusion of glucose is begun. Which of the following is the most likely explanation for hypoglycemia in this patient? A) Decreased a-glucosidase activity in the gastrointestinal tract B) Decreased hepatic production of glucose C) Increased agonist activity with insulin receptor sites D) Increased conversion of glucose to glycogen E) Increased release of insulin from the pancreas

E. Glipizide is a second-generation sulfonylurea. The mechanism of action of sulfonylureas involves closing a potassium channel in a pancreatic islet B-cell membrane, resulting in depolarization of the cell. Depolarization leads to opening of voltage-gated calcium channels, calcium influx, and endogenous insulin release from the pancreas. Sulfonylureas are used as treatment in type 2 diabetes mellitus, in which the pancreatic islet cells retain function and are able to release insulin, in contrast to type 1 diabetes mellitus where the islet cells are destroyed and no functional insulin is produced. Insulin induces glucose uptake by peripheral tissues, such as skeletal muscle and adipose tissue, and promotes glycogen, triglyceride, and protein synthesis. Through these mechanisms, insulin decreases the concentration of serum glucose. The patient's recently increased dose of glipizide increases the risk for inadvertent iatrogenic hypoglycemia. Hypoglycemia that results from sulfonylurea use often persists for 12-24 hours, as sulfonylureas are long-acting agents. Treatment in the short term includes administration of dextrose. The dose of sulfonylurea should also be decreased, or an alternative agent should be selected for glycemic control. Incorrect Answers: A, B, C, and D. Decreased a-glucosidase activity in the gastrointestinal tract (Choice A) is the result of administration of acarbose. Acarbose inhibits brush border enzymes in the small intestine, preventing digestion of carbohydrates and therefore limiting the total amount of dietary carbohydrate absorbed. Side effects include gastrointestinal upset (eg, diarrhea, flatulence). Decreased hepatic production of glucose (Choice B) is incorrect, as gluconeogenesis is not directly affected by sulfonylureas. Metformin acts in this manner. Increased agonist activity with in

64 Exam Section 2: Item 15 of 50 National Board of Medical Examiners' Comprehensive Basic Science Self-Assessment 15. A 44-year-old man comes to the physician for a follow-up examination 1 month after receiving a cadaveric kidney transplant. He says he feels well. Current medications include mycophenolate and prednisone. Physical examination shows an arteriovenous fistula in the left upper extremity and a well-healing surgical wound over the right lower quadrant of the abdomen. Serum studies show a urea nitrogen concentration of 44 mg/dL and creatinine concentration of 2.6 mg/dL, compared with 21 mg/dL and 1.8 mg/dL, respectively, 2 weeks ago. Examination of a biopsy specimen of the transplanted kidney shows an inflammatory infiltrate affecting the small arterioles. These changes are most likely caused by which of the following mechanisms? A) Donor CD4+T lymphocytes recognize class II MHC molecules on host dendritic

E. Host CD8+ T lymphocytes recognize class I MHC molecules on allograft cells. This leads to acute T-lymphocyte-mediated transplant rejection (TCMR), typified by an asymptomatic rise in creatinine and biopsy showing an inflammatory infiltrate affecting the small arterioles. This most commonly occurs within the first six months after transplantation. TCMR is caused by host CD8+ T lymphocytes reacting to donor MHC I complexes present on cells within the glomeruli, tubules, interstitium, and blood vessels of the allograft. The recognition of the foreign MHC I complex induces a robust immune response against the allograft. Biopsy is required for the evaluation of graft dysfunction because it can distinguish between acute rejection and other causes of renal injury, including acute tubular necrosis, interstitial nephritis, or viral infections (eg, cytomegalovirus or BK virus). Primary histologic changes include renal interstitial infiltration with lymphocytic cells, in addition to the obliteration of the tubular basement membrane. Treatment is based on the degree of rejection and allograft dysfunction but typically includes steroids and lymphocyte- depleting agents. Incorrect Answers: A, B, C, and D. Donor CD4+ T lymphocytes recognize class II MHC molecules on host dendritic cells (Choice A) describes one pathologic mechanism in acute graft versus host disease (GVHD). Host dendritic cells present host antigens on their surface continuously via MHC II molecules, and these are recognized by donor CD4+ T cells, which induce a robust inflammatory response resulting in GVHD. These CD4+ cells help recruit donor CD8+ T cells, which recognize class I MHC molecules on host dendritic cells (Choice C) and augment the immune response against host tissues. Common manifestations of acute GVHD include rash, hepatitis, and diarrhea. Donor C

60 Exam Section 2: Item 11 of 50 National Board of Medical Examiners' Comprehensive Basic Science Self-Assessment 11. A 20-year-old man with sickle cell disease has chronic hemolytic anemia and has frequent painful crises that require transfusions. Hydroxyurea treatment significantly decreases the painful crises and transfusion requirements. Which of the following is the most likely mechanism of the beneficial effects of this treatment? A) Decreased erythrocyte cellular dehydration B) Decreased hemoglobin S concentration C) Increased affinity of hemoglobin S for oxygen D) Increased bone marrow production of erythrocytes E) Increased hemoglobin F concentration

E. Increased hemoglobin F concentration is the most likely mechanism by which hydroxyurea decreases the frequency of sickle cell pain crises and transfusion requirements. Hydroxyurea is a myelosuppressive drug used in a number of conditions such as essential thrombocytosis but is commonly used for the treatment of sickle cell disease. It inhibits ribonucleotide reductase, which results in the inhibition of DNA synthesis within erythrocytes leading to apoptosis. This is hypothesized to lead to recruitment of earlier erythroid stem cells that contain a higher concentration of hemoglobin F (HbF), which has a higher affinity for oxygen but does not result in sickling of the erythrocyte. Beyond inhibiting the production of erythrocytes containing hemoglobin S (HbS), hydroxyurea also inhibits other cell lines, including granulocytes and platelets. Incorrect Answers: A, B, C, and D. Decreased erythrocyte cellular dehydration (Choice A) is not the mechanism of action of hydroxyurea. Erythrocyte hydration is maintained through ion pumps with regulation of intracellular potassium, sodium, and water concentration. There exists a rare group of disorders known as hereditary hydrocytosis and xerocytosis resulting in abnormalities of erythrocyte hydration. Decreased HbS concentration (Choice B) is an indirect effect of hydroxyurea, but the primary mechanism is by increasing HbF concentrations. Increased affinity of HbS for oxygen (Choice C) is not a direct effect of hydroxyurea. Blood in patients with sickle cell disease overall has a lower oxygen-carrying capacity. HbS unto itself has the same affinity for oxygen as hemoglobin A when HbS is not polymerized, but an increased amount of 2,3-BPG in erythrocytes containing HbS alters the oxygen dissociation curve. Increased bone marrow production of erythrocytes (Choice D) is not the cor

54 Exam Section 2: Item 5 of 50 National Board of Medical Examiners' Comprehensive Basic Science Self-Assessment 5. A 24-year-old woman at 28 weeks' gestation is brought to the emergency department because of a 3-hour history of shortness of breath. She has asthma, but she currently takes no medications. Her pulse is 100/min, respirations are 32/min, and blood pressure is 120/83 mm Hg. Physical examination shows the use of accessory muscles of respiration. Diffuse inspiratory and expiratory wheezes are heard. An inhaled Bradrenergic agonist is administered. Which of the following findings is most likely in this patient after this therapy? A) Bradycardia B) Diaphoresis C) Dry mouth D) Pallor E) Tremor

E. Inhaled B2-adrenergic agonists, such as albuterol, formoterol, and salmeterol are used for the treatment of bronchoconstriction in asthma and in chronic obstructive pulmonary disease. Drugs in this class act preferentially on the B2-adrenergic receptor, which is found on bronchial smooth muscle, leading to bronchodilation. B2-adrenergic agonists cause a variety of side effects that are mediated by the metabolic actions of the B2- and also the B1-adrenergic receptor, and include tremor, increased activity of the sodium-potassium ATPase with subsequent hypokalemia, glycogenolysis with subsequent hyperglycemia, tachycardia, hypertension, and headache. Recognizing the effect of B-agonists on tremor forms the basis of using B-adrenergic antagonists, such as propranolol, in the treatment of benign essential tremor. Incorrect Answers: A, B, C, and D. Bradycardia (Choice A) is a side effect of some a2-adrenergic agonists, such as clonidine, which are used for the treatment of hypertension, hypertensive emergency, and opiate withdrawal. Diaphoresis (Choice B) is mediated by sympathetic stimulation from acetylcholine on muscarinic, not adrenergic receptors. This is one of few cases when the sympathetic nervous system signals via acetylcholine, whereas its effects are typically mediated by norepinephrine, epinephrine, and dopamine. Dry mouth (Choice C) is a side effect of muscarinic receptor antagonists, such as atropine, which are used for the treatment of bradycardia. Pallor (Choice D) is a potential side effect of B2-adrenergic antagonists, such as propranolol. Peripheral blood vessels dilate in response to B2-agonism. Blocking this effect promotes a net vasoconstriction. Educational Objective: Inhaled B2-adrenergic agonists, such as albuterol, formoterol, and salmeterol are used for the treatment of bronchoconstriction. Si

15 Exam Section 1: Item 15 of 50 National Board of Medical Examiners' Comprehensive Basic Science Self-Assessment 15. A 34-year-old man who is seropositive for HIV has a neoplasm manifested as violaceous raised cutaneous plaques (erythematous cutaneous plaques). Examination of tissue obtained on biopsy of the neoplasm is most likely to show which of the following? A) Dense, relatively acellular fibrous tissue with few typical fibroblasts B) Large polygonal and giant cells with myofibrils C) Malignant gland-forming cells with mucin D) Poorly differentiated malignant squamous cells E) Proliferating spindle cells forming slit-like spaces filled with blood

E. Kaposi sarcoma is a neoplasm of vascular endothelial cells, which may be induced by human herpesvirus 8 (HHV-8) in patients with poorly controlled HIV or who are otherwise immunosuppressed. Histologically, proliferating spindle cells forming slit-like spaces filled with blood confirms the diagnosis. Kaposi sarcoma also exhibits a classic form, which occurs spontaneously in men of Eastern European descent, and an endemic form, seen in sub-Saharan Africans. Clinically, Kaposi sarcoma progresses from patch, to plaque, and then to tumor stages. In the plaque phase, as described in this patient, reddish-purple or violaceous, nonblanching plaques are seen. It commonly affects the lower extremities, face, oral mucosa, and genitalia. It may also affect the gastrointestinal and pulmonary systems. For this reason, screening for occult gastrointestinal bleeding using a fecal occult blood test and screening for pulmonic lesions using a chest x-ray is recommended. Overall, Kaposi sarcoma is a low-risk tumor that demonstrates a good prognosis, provided the underlying HIV infection is treated. Local treatments including intralesional chemotherapy, radiation therapy, and topical retinoids may help manage existing lesions, though they do not prevent new ones from forming. For those with advanced disease, systemic therapy is warranted and can include anthracyclines (doxorubicin, daunorubicin) and/or paclitaxel, though treatment regimens are broad. Incorrect Answers: A, B, C, and D. Dense, relatively acellular fibrous tissue with few typical fibroblasts (Choice A) is characteristic of a scar. Dermal collagen fibers increase as a result of scarring. While the majority of collagen fibers in healthy skin are collagen, type I, scar is initially created by collagen, type III. Large polygonal and giant cells with myofibrils (Choice B) are c

66 Exam Section 2: Item 17 of 50 National Board of Medical Examiners' Comprehensive Basic Science Self-Assessment 17. A 15-month-old boy is brought to the physician by his mother because of a 9-month history of recurrent bacterial infections. The patient has not had recurrent viral infections. He is at the 10th percentile for length and weight. Physical examination shows multiple areas of honey yellow, crusted lesions over the lower extremities. A Gram stain of one of the lesions shows many gram-positive cocci in clusters but no leukocytes. Laboratory studies show no abnormalities except for a leukocyte count of 30,000/mm3. This patient most likely has a rare autosomal recessive disease leading to a lack of CD18 expression. The leukocytes would be deficient in which of the following characteristics? A) Cytokine production B) Helper T-cell function C) Immunoglobulin gene rearrangement D) Killing of intracellular bacte

E. Leukocyte adhesion deficiency (LAD) results from a defect in the attachment of leukocytes to the vascular endothelium, which consequently results in the impaired recruitment and migration to sites of extravascular inflammation or infection. LAD type 1 is caused by a dysfunctional LFA-1 integrin (CD18) protein on the leukocyte surface, which does not allow for normal attachment of the cell to the vascular endothelium. It is typically characterized by recurrent bacterial infections, impaired wound healing, a delayed detachment of the umbilical cord after birth, and lack of leukocytes at sites of infection with an absence of pus. The actions of leukocyte phagocytosis and bacterial killing are otherwise unimpaired. Laboratory studies in patients with LAD will show increased leukocyte concentrations in the blood. Incorrect Answers: A, B, C, and D. Cytokine production (Choice A) is not impaired in leukocyte adhesion deficiency. Cytokines are signaling proteins that modulate cellular response to inflammation. Proinflammatory cytokines include IL-1, 6, 8, 12, and 18, interferons, and tumor necrosis factor. Anti-inflammatory cytokines include İL-4, 10, 11, and 13. Helper T-cell function (Choice B) remains intact in LAD. Helper T lymphocytes are an essential component of the adaptive immune response to infection. They activate CD8+ effector T lymphocytes, promote humoral immunity, activate macrophages and neutrophils, and recruit eosinophils. Impaired helper T-lymphocyte function is the underlying mechanism that allows opportunistic infections to occur in HIV infection and AIDS. Immunoglobulin gene rearrangement (Choice C), also called V(D)J recombination, is the mechanism by which lymphocytes are able to generate the enormous range of the adaptive immune response to foreign antigens. Deficiency results in severe combined or

41 Exam Section 1: Item 42 of 50 National Board of Medical Examiners' Comprehensive Basic Science Self-Assessment 42. A 1-year-old girl is brought to the physician by her parents because of developmental delay. Physical examination shows a coarse facial appearance, clouded corneas in both eyes, hepatosplenomegaly, stiff joints, and developmental delay. Urine studies show an increased concentration of dermatan sulfate and heparan sulfate. Which of the following enzymes is most likely impaired in this patient? A) a-L-Fucosidase B) B-Galactosidase C) Glucose 6-phosphatase D) B-Hexosaminidase A E) a-lduronidase F) Neuraminidase G) Steroid sulfatase

E. Mucopolysaccharidoses are caused by an enzyme deficiency that prevents the degradation of glycosaminoglycans. The buildup of atypical substrates leads to varying levels of cellular dysfunction on the basis of the enzyme affected. The diagnosis is not typically clinically apparent at birth, as the abnormal deposition of metabolites has not had time to occur. The two primary mucopolysaccharidoses are Hurler syndrome and Hunter syndrome. Hurler syndrome and Hunter syndrome are caused by deficiencies in a-iduronidase and iduronate-2-sulfatase, respectively, with both resulting in the deposition of heparan sulfate and dermatan sulfate. Hurler Syndrome is an autosomal recessive disorder and presents in the first year of life with coarse facial features, corneal clouding, hepatosplenomegaly, joint contractures, intellectual disability, and developmental delay, as seen in this patient, as well as skeletal abnormalities, heart failure, hydrocephalus, hearing loss, and recurrent respiratory tract infections. Hunter syndrome is an X-linked recessive disorder and presents with milder features of Hurler syndrome with the absence of corneal clouding, as well as aggressive behavior and pearly papular skin lesions. Diagnosis is made with urinary glycosaminoglycans and an analysis of enzyme activity; treatment is largely supportive. Average lifespan is five years for Hurler syndrome and 13 to 20 years for Hunter syndrome. Incorrect Answers: A, B, C, D, F, and G. a-l-Fucosidase (Choice A) deficiency leads to a rare glycoproteinosis called fucosidosis secondary to the accumulation of glycoproteins, glycolipids, and oligosaccharides. Fucosidosis is an autosomal recessive disorder that presents with coarse facial features, hepatosplenomegaly, intellectual disability, seizures, skeletal abnormalities, alterations in perspiration, and neu

56 Exam Section 2: Item 7 of 50 National Board of Medical Examiners' Comprehensive Basic Science Self-Assessment 7. A 3-year-old boy is brought to the emergency department by his father because of bleeding from a cut on his left arm after he fell off his tricycle 1 hour ago. Physical examination shows a 2-cm shallow laceration on the left upper extremity. The laceration has stopped bleeding and is covered with a clot. This finding results from hepatic synthesis of which of the following? A) Enzymes that increase production of platelets B) Inhibitors of naturally occurring anticoagulants C) Plasmin from plasminogen D) Protein C and protein S E) Proteins that result in the production of fibrin strands

E. Proteins that result in the production of fibrin strands are made by hepatocytes in the liver and aid in the formation of blood clots. Two common pathways of activating the clotting cascade are the intrinsic pathway, whereby exposure to endothelial collagen initiates clotting, and the extrinsic pathway, whereby tissue factor release from endothelial cells initiates the cascade. Regardless of the inciting event, both pathways converge on the common pathway, which begins with the activation of factor X to factor Xa. Activated factor X induces the conversion of prothrombin to thrombin, which is subsequently required for the conversion of fibrinogen to fibrin. Fibrin is an essential component of clot formation, polymerizing to form fibrin strands via the assistance of factor XIlla. This fibrin meshwork then incorporates platelets to form a mature clot over the site of injury, which stops bleeding. Fibrinogen is produced in the liver. Incorrect Answers: A, B, C, and D. Enzymes that increase production of platelets (Choice A) are not produced in the liver. Certain mutations in genes such as JAK2, MPL, and CALR can cause unregulated production of platelets in the bone marrow and are a cause of disorders such as essential thrombocytosis, polycythemia vera, and myelodysplastic syndrome, but these are not hepatically produced enzymes. Inhibitors of naturally occurring anticoagulants (Choice B) such as protein C inhibitor, a plasma serine protease inhibitor, exist naturally in the body. They aid in maintaining balance between clotting and bleeding. Plasmin from plasminogen (Choice C) describes the process of activating plasminogen to become plasmin, which subsequently degrades fibrin clots leading to clot dissolution. While plasminogen is made in the liver, its role is to degrade and prevent clot formation. Common medications

59 Exam Section 2: Item 10 of 50 National Board of Medical Examiners' Comprehensive Basic Science Self-Assessment 10. A 35-year-old woman comes to the physician because of fever and left flank pain for 2 days. She has had several similar episodes during the past 4 years, all of which resolved with antibiotic therapy. Her temperature is 38.6°C (101.5°F). Physical examination shows exquisite left costovertebral angle tenderness. Urinalysis shows small numbers of WBCS and WBC casts, and numerous triple phosphate crystals. A course of antibiotic therapy does not resolve the pain and the patient undergoes resection of the left kidney. A photograph of the patient's resected kidney is shown. Which of the following was the most likely causal organism in this patient's disease process? A) Enterobacter cloacae B) Enterococcus faecalis C) Escherichia coli D) Klebsiella pneumoniae E) Proteus mirabilis F) Pseudomonas aeruginosa

E. Proteus mirabilis is the most likely causal organism. The gross photograph shows a large staghorn calculus occupying the near-entirety of the renal pelvis and branching into multiple calyces. Staghorn calculi are made of struvite, also known as triple phosphate or magnesium ammonium phosphate. Bacteria such as P. mirabilis or K. pneumoniae are urease-producing organisms. Urease catalyzes the conversion of urinary urea to ammonia and carbon dioxide, which results in urine alkalinization. The solubility of struvite is decreased under alkaline conditions and precipitates as a crystal. An alkaline urine is present in this patient with a urinary pH of 7.5. Struvite crystals have an orthorhombic configuration under light microscopy. Struvite calculi are radiopaque and can be potentially identified by an x-ray or CT scan. Struvite calculi may adopt a branching morphology that molds itself to the shape of the collecting system (staghorn calculi). Given their large size and often ramified structure, it is rare for struvite calculi to pass through the urinary tract, and surgical removal is often necessary along with treatment of the underlying infection. The presence of calculi in the collecting system predisposes to recurrent episodes of pyelonephritis. Treatment is usually with a third-generation cephalosporin, although indole-positive P. mirabilis may have inducible resistance to B-lactams mediated by the AmpC gene. Incorrect Answers: A, B, C, D, and F. Enterobacter cloacae (Choice A) is an AmpC inducible gram-negative rod and intrinsically resistant to B-lactam antibiotics. It can cause urinary tract infections, pneumonia, bacteremia, and rarely meningitis. Treatment is based on resistance detected by standard microbiologic techniques. E. cloacae is not known to predispose to struvite stones although it can produce urease

55 Exam Section 2: Item 6 of 50 National Board of Medical Examiners' Comprehensive Basic Science Self-Assessment 6. A 54-year-old woman comes to the physician because of a 3-month history of fatigue and a persistent cough productive of colorless mucoid sputum. She also has had a 14-kg (30-lb) weight loss during this period. She has smoked 2 packs of cigarettes daily for 25 years. She is 168 cm (5 ft 6 in) tall and now weighs 52 kg (115 lb); BMI is 19 kg/m2. Percussion of the chest shows hyperresonance. Her serum calcium concentration is 12.8 mg/dL. A chest x-ray shows a mass in the right middle lobe of the lung near the hilum. Examination of aspirate from the mass shows malignant cells. The neoplasm in this patient is most likely which of the following? A) Adenocarcinoma B) Giant cell carcinoma C) Large cell undifferentiated carcinoma D) Small cell carcinoma E) Squamous cell carcinoma

E. Squamous cell carcinoma of the lung is the second most common type of primary lung cancer after adenocarcinoma. Risk factors for all major types of lung cancer include tobacco use, secondhand smoke, asbestos, or radon exposure, and a family history of lung cancer. Features associated with squamous cell carcinoma of the lung include pulmonary cavitations, central location, and hypercalcemia as a result of paraneoplastic parathyroid hormone-related peptide (PTHPP) production. Histologic characteristics include polygonal cells with intercellular bridges, eosinophilic cytoplasm, keratin pearls, and necrosis. Lung cancer typically presents with cough, unintentional weight loss, hemoptysis, chest pain, dyspnea, and hoarseness when symptomatic; occasionally, wheezing, focal rhonchi, or hypertrophic osteoarthropathy may be noted on examination. Diagnosis is made by chest imaging and biopsy. Prognosis is a function of the cancer type along with grading and staging of the disease. It is often detected late in stage, once metastatic, at which point the prognosis is poor. Incorrect Answers: A, B, C, and D. Adenocarcinoma (Choice A) of the lung is the most common overall primary lung cancer and the most common among nonsmokers. It typically presents in the periphery of the lung rather than centrally. It is more common in women than men. A glandular pattern is classically seen on histology with mucin-positive staining. Giant cell carcinoma (Choice B) is a rare carcinoma of the lung that contains pleomorphic giant, multinucleated cells on histology. It most commonly involves the upper lobes and the lung periphery. Large cell undifferentiated carcinoma (Choice C) is a rare, poorly differentiated lung neoplasm that displays large atypical cells on histology. It most commonly involves the lung periphery. Small cell carcinoma (Choice

68 Exam Section 2: Item 19 of 50 National Board of Medical Examiners' Comprehensive Basic Science Self-Assessment 19. An 82-year-old woman with type 2 diabetes mellitus and painful neuropathy comes to the physician for a follow-up examination. At her last visit 3 months ago, the physician prescribed a topical cream and recommended use of acetaminophen as needed for pain. The patient reports that the topical therapy helped more than she expected and did decrease her need for acetaminophen. Physical examination shows no change in her condition. The prescribed cream most likely depletes which of the following substances from sensory nerve endings? A) Arachidonic acid B) Bradykinin C) Histamine D) Serotonin E) Substance P

E. Substance P has likely been depleted by the use of capsaicin cream, an agonist of transient receptor potential vanilloid 1 receptor (TRPV1) on nociceptive nerves. This initially results in a painful stimulus; however, the continued use of capsaicin completely depletes these nerve fibers of substance P. As substance P release is implicated in nociception, depletion of this substance results in gradual pain relief. It has applications in the treatment of diabetic neuropathy and neuropathic pain and has been used off-label to treat cannabis hyperemesis syndrome. While this patient's neuropathy is irreversible as a result of damage to nerve fibers over many years of hyperglycemia, her pain is effectively controlled by depleting the nociceptive neurotransmitter substance P. Incorrect Answers: A, B, C, and D. Arachidonic acid (Choice A) is produced using phosphatidylinositol and this production is inhibited by exogenous corticosteroids. Additionally, arachidonic acid is required for the production of prostanoids via cyclooxygenase. Prostaglandins and leukotrienes play a role in inflammation, and this pathway is effectively inhibited by nonsteroidal anti-inflammatory drugs, which are cyclooxygenase inhibitors. Bradykinin (Choice B) is made from kininogen via the action of kallikrein, which can be blocked by kallikrein inhibitors such as ecallantide. Bradykinin is a vasodilator that is implicated in hereditary angioedema. It does not modulate pain. Histamine (Choice C) is blocked by antihistamine medications. Histamine release causes vasodilation and increased vascular permeability through its action on four separate histamine receptors; it has been implicated as a contributor to conditions such as environmental allergies. Antihistamine medications are broken into first and second generations, with first-generation medicati

35 Exam Section 1: Item 36 of 50 National Board of Medical Examiners' Comprehensive Basic Science Self-Assessment 36. A 46-year-old man comes to the office because of a 1-year history of difficulty reading small print. He has no other history of vision difficulty. He has hypertension treated with hydrochlorothiazide. Pulse is 72/min, and blood pressure is 142/86 mm Hg; other vital signs are within normal limits. The pupils are 2 mm, equally round, and reactive to light and accommodation. Ocular movements are intact. On wall chart testing, visual acuity is 20/20 bilaterally. On handheld chart testing, he can read the 20/20 line only if he holds the card more than 14 inches away. Disc margins are sharp; there are no hemorrhages, exudates, or arteriolar narrowing. No other abnormalities are noted. Which of the following is the most likely diagnosis? O A) Hyperopia B) Hypertensive retinopathy C) Муорia D) Nuclear scl

E. The Helmholtz theory of accommodation holds that contraction of the ciliary muscle relaxes tension of the zonular fibers upon the lens, allowing the lens to become rounder, thereby increasing its dioptric power and allowing for focusing on objects at close distances. The accommodative ability of the lens is dependent upon its ability to distend its shape. Age-related hardening of the lens and loss of accommodative ability is known as presbyopia and is a normal age-related change. Patients present with blurry vision at near-distance and notice that objects must be held at further working distances to be seen clearly. Normal pupillary reflexes to accommodation are not affected by presbyopia. Treatment is with reading glasses with positive dioptric power. Incorrect Answers: A, B, C, and D. Hyperopia (Choice A) occurs when the focal point of the eye is found at a point behind the retina and may result from short axial length or diminished converging power of the cornea and lens. Patients present with blurry vision both at a distance and when near. The condition usually presents in childhood. Treatment is with corrective lenses with a positive dioptric power. Hypertensive retinopathy (Choice B) is often asymptomatic but may cause blurry vision. Fundus examination discloses optic disc edema, retinal nerve fiber layer infarcts, retinal edema, and retinal arteriolar attenuation. While this patient has hypertension, the absence of characteristic fundus findings makes hypertensive retinopathy an unlikely diagnosis. Myopia (Choice C) occurs when the focal point of the eye is found at a point anterior to the retina and may result from long axial length or increased converging power of the cornea and lens. Patients present with blurry vision at distance. The condition usually presents in childhood. Pathologic forms of myopia may

65 Exam Section 2: Item 16 of 50 National Board of Medical Examiners' Comprehensive Basic Science Self-Assessment 16. A 25-year-old man comes to the physician because of a 6-week history of daytime fatigue, irritability, and emotional lability; he has had a 4.5-kg (10-lb) weight loss during this period. He says that he is sleep-deprived, but he is unable to fall asleep readily. He is a medical student and has recently finished an intensive care unit rotation where he often worked days and nights. He is currently on an outpatient rotation and will be starting vacation next week. Physical examination shows no abnormalities. Which of the following recommendations by the physician is most appropriate to improve this patient's sleep hygiene? A) "Have a glass of wine before bedtime." B) "Make sure you sleep late whenever you can to catch up on your sleep." C) "Stay in bed, even if you can't fall asleep." D) "Take it easy.

E. This patient likely has a circadian rhythm sleep disorder caused by the long and irregular hours of his intensive care unit rotation and should adopt sleep hygiene habits such as a regular wake- up time. Circadian rhythm sleep disorders feature misalignment between environmental light-dark phases and an individual's internal circadian rhythm. This misalignment may derive from an extrinsic source such as shift work, which is called shift work sleep disorder. Sequelae can include fatigue, irritability, and mood lability. The diagnosis is made by history and sleep diaries. Management relies primarily on sleep hygiene. Sleep hygiene refers to sleep-promoting habits such as a regular bedtime and wake time, avoiding screen time before bedtime, avoiding coffee and other substances that may interfere with sleep, exercising regularly, getting out of bed if unable to sleep (to avoid a mental association between wakefulness and the bed), avoiding watching the clock, and maintaining a quiet and comfortable bedroom environment. By regularly waking at the same time each morning, patients can reset their circadian rhythm and take advantage of accumulated sleep pressure as a result of sleep restriction to fall asleep more quickly. Incorrect Answers: A, B, C, and D. Wine (Choice A) and other alcoholic beverages decrease the duration of rapid-eye movement and delta-wave sleep and would therefore exacerbate this patient's fatigue. Wine is not an effective long-term solution for sleep-onset insomnia. Encouraging a patient to sleep late to catch up on sleep (Choice B) would lead to a pattern of going to sleep and waking up later and may exacerbate circadian rhythm sleep disorder. By instead waking up at the same time each morning, patients can more easily fall asleep at bedtime and reestablish a normal circadian pattern. Staying in bed


Kaugnay na mga set ng pag-aaral

Chapter 39: Nursing Management: Patients With Rheumatic Disorders

View Set

ANS and Special Senses Laboratory

View Set

Medical-Surgical RN A Prophecy Relias

View Set

Tort 3: Negligence: duty of care and breach of duty

View Set

Chapter 15: Tools of Monetary Policy

View Set

Module 5 Capstone/ Transition to Practice PRACTICE QUIZ

View Set

Peds Neurologic Alterations 3 from Mom

View Set

quiz 1 Ch.1 & Ch.2 operating systems

View Set